Why atheism is self-defeating.

Posted on July 9, 2012 By

Whoever it was that first said, “The hardest part of my job is getting other people to do my job for me” apparently didn’t have the job of refuting naturalism, which is the belief (upon which atheism is based) that the natural world is self-existent, and therefore does not require an intelligent cause (God). This is because naturalism is a self-defeating belief system. By creating this self-defeating belief system, naturalists have left little work for the theist to do, other than to point out this self-defeating nature. The easiest way for one to see that naturalism is self-defeating is to realize that, under naturalism, we have no reason to believe that ANY of our beliefs are true…let alone a belief in naturalism. In fact, under the naturalist belief system, we have no reason to think that we even have the ability to reason accurately. David Wood writes in his essay The Explanatory Emptiness of Naturalism (as it appears in True Reason: Christian Responses to the Challenge of Atheism):

“According to naturalists, our ability to reason is the product of natural selection acting on random mutation. Natural selection, of course, favors traits that help organisms survive and reproduce. So if human reasoning evolved naturally, it’s because it helped human beings survive and reproduce. Does this give us any basis for trusting our reasoning ability when it comes to questions of cosmology, or quantum mechanics, or neuroscience? Not at all. At best, our cognitive faculties would be reliable when it comes to finding berries, or using a spear against an enemy, or doing something to attract a mate. Interestingly, Darwin himself noticed this problem. He once admitted:

‘[W]ith me the horrid doubt always arises whether the convictions of man’s mind, which has been developed from the mind of the lower animals, are of any value or at all trustworthy. Would anyone trust in the convictions of a monkey’s mind, if there are any convictions in such a mind?’

“In other words, our reasoning ability serves the same evolutionary purpose as the traits of other animals (e.g., the claws of a lion, the song of a canary, or the colorful buttocks of a baboon). We wouldn’t trust the traits of animals to lead us to the truth, because they weren’t developed for that purpose. Why, then, would we trust our own convictions, which are the result of the same evolutionary process? There’s no way around this problem for naturalists, for in order to escape the Problem of Reason, they would need to construct an argument. But this argument would presuppose the reliability of human reason, which is precisely the issue under investigation. Hence, if we take Naturalism seriously, we cannot take our reasoning ability seriously, and science falls apart.”

Renowned philosopher of neuroscience Patricia Churchland, despite being a staunch naturalist, admits to this problem with naturalism in her article Epistemology in the Age of Neuroscience:

“The principal chore of nervous systems is to get the body parts where they should be in order that the organism may survive. . . Improvements in sensorimotor control confer an evolutionary advantage: a fancier style of representing [the world] is advantageous so long as it . . . enhances the organism’s chances for survival. Truth, whatever that is, definitely takes the hindmost.”

Prominent atheist philosopher Thomas Nagel admits to the same in his book Mind and Cosmos, and devotes much of the rest of the book trying to wriggle free from theism. He writes:

“Evolutionary naturalism implies that we shouldn’t take any of our convictions seriously, including the scientific world picture on which evolutionary naturalism itself depends.”

Naturalism, simply put, leaves us no reason whatsoever to think that any of our beliefs are reliable…such as a belief in naturalism.

Naturalism, simply put, leaves us no reason whatsoever to think that any of our beliefs are reliable…such as a belief in naturalism. Please recall that naturalism insists that the evolutionary mechanism of the mutation of genes is mindless and random. Also recall that natural selection selects for survivability, not for truth. And, if one stops to think, there is no reason to think that certain false beliefs could not provide just as much survival value as a corresponding true belief. For example, the belief that eating a particular plant should be avoided because doing so would cause one to turn into a werewolf provides just as much survival value as the belief that eating that plant should be avoided because doing so puts poison into one’s body. Philosopher Alvin Plantinga sums up this point in his book Where the Conflict Really Lies: Science, Religion, and Naturalism:

“…neurology causes adaptive behavior and also causes or determines belief content [according to naturalism]: but there is no reason to suppose that the belief content thus determined is true. All that’s required for survival and fitness is that the neurology cause adaptive behavior; this neurology also determines belief content, but whether or not that content is true makes no difference to fitness. Certain NP [neuro-physiological] properties are selected for, because they contribute to fitness. These NP properties also cause or determine belief content; they associate a content or proposition with each belief. The NP properties are selected, however, not because they cause the content they do, but because they cause adaptive behavior. If the content, the proposition determined by the neurology (the NP properties of the belief) is true, fine. But if it is false, that’s no problem as far as fitness goes.”

Naturalism leaves us no reason to think that we can rely on our reason. By dismissing God, naturalists have stripped away any reason to think that human reason can lead to truth. We should therefore dismiss naturalism as having no more value than the empty “convictions of a monkey’s mind” (as Charles Darwin put it).

Christianity is responsible for the rise of science

What reason, then, does the theist provide for believing that human reason can lead to truth? Stanley Jaki, a leading philosopher and historian of science (and a physicist), provides perhaps the best articulated answer to this question. An article titled The Origin of Science describes how Christian theism anchors man’s rationality in God’s rationality and how Christianity was responsible for the rise of science. Below is an excerpt:

In Christ and Science (p. 23), Jaki gives four reasons for modern science’s unique birth in Christian Western Europe: “Once more the Christian belief in the Creator allowed a break-through in thinking about nature. Only a truly transcendental Creator could be thought of as being powerful enough to create a nature with autonomous laws without his power over nature being thereby diminished. Once the basic among those laws were formulated science could develop on its own terms.”

“The Christian idea of creation made still another crucially important contribution to the future of science. It consisted in putting all material beings on the same level as being mere creatures. Unlike in the pagan Greek cosmos, there could be no divine bodies in the Christian cosmos. All bodies, heavenly and terrestrial, were now on the same footing, on the same level. This made it eventually possible to assume that the motion of the moon and the fall of a body on earth could be governed by the same law of gravitation. The assumption would have been a sacrilege in the eyes of anyone in the Greek pantheistic tradition, or in any similar tradition in any of the ancient cultures.”

“Finally, man figured in the Christian dogma of creation as a being specially created in the image of God. This image consisted both in man’s rationality as somehow sharing in God’s own rationality and in man’s condition as an ethical being with eternal responsibility for his actions. Man’s reflection on his own rationality had therefore to give him confidence that his created mind could fathom the rationality of the created realm.” “At the same time, the very createdness could caution man to guard against the ever-present temptation to dictate to nature what it ought to be. The eventual rise of the experimental method owes much to that Christian matrix.”

Atheists insist that naturalism is the worldview most compatible with science and that theism is unscientific. And this viewpoint certainly gets plenty of play in the media, in academia, and in popular thought. But, if naturalism does not provide us with any reason to trust our reason, why should we accept any naturalist reasoning? And if Christian theism was responsible for the rise of science (as I detail in Without Christianity, There Would Be No Science), why does naturalism so often get the credit for being the more scientific worldview? Perhaps Vladimir Lenin was onto something when he said, “A lie told often enough becomes the truth.” 


  1. Natural selection is most certainly NOT “mindless and random” (you may be thinking about mutation).

    I respond to Plantinga’s Evolutionary Argument Against Naturalism (which I believe is the same as your argument) in detail at the link below. In short, natural selection does a pretty good job of matching our beliefs with reality. If they weren’t well-matched, those genes would have been selected against.

    http://crossexaminedblog.com/2011/12/07/plantingas-nutty-evolutionary-argument-against-naturalism/

    • Bob:

      I have reviewed your response to Plantinga’s argument, and I am going to be very frank. Please do not take personal offense, but in this situation frankness is needed:

      Your reply to Plantinga’s argument is an absolutely textbook example of responding to a straw-man caricature of an argument, rather than the actual argument that was presented. For example, your chart claims that Plantinga is suggesting that “I’m hungry” leads to the maladaptive neuro-physiological [NP] response of “get to fresh air.” Your chart also claims that Plantinga is suggesting, for example, that “I’m sleepy” leads to the maladaptive neuro-physiological response of “eat palatable food.”

      The problem is that there is nothing in Plantinga’s argument that suggests that physiological needs lead to maladaptive neuro-physiological responses. This is something that you have injected into Plantinga’s argument so as to (perhaps deliberately) mischaracterize his argument. If Plantinga actually suggested this, then you should be able to cite him as doing so. But guess what…you won’t, because you can’t, because he never made such an argument. I challenge you to prove me wrong.

      Your reply to Plantinga’s argument first creates a straw-man caricature of his argument and then proceeds to attack that straw man rather than his actual argument. I strongly suspect that you did not read his actual argument. Below is a copy and paste of some of his ACTUAL ARGUMENT that you need to respond to:

      “Natural selection selects for adaptive NP [neuro-physiological] properties; those NP properties determine content; but natural selection just has to take pot-luck with respect to the propositions or content determined by those adaptive NP properties. It does not get to influence or modify the function from NP properties to content properties: that’s just a matter of logic or causal law, and natural selection can’t modify either. Indeed, the content generated by the NP properties of this structure, on this occasion, need have nothing to do with that predator, or with anything else in the environment. True: the structure is correlated with the presence of a predator and indicates that presence; but indication is not belief. Indication is one thing; belief content is something else altogether, and we know of no reason (given materialism) why the one should follow the other. We know of no reason why the content of a belief should match what that belief (together, perhaps, with other structures) indicates. Content simply arises upon the appearance of neural structures of sufficient complexity; there is no reason why that content need be related to what the structures indicates, if anything. Indeed, the proposition constituting that content need not be so much as about that predator; it certainly need not be true.”

      Please first note the first sentence in the above citation from Plantinga’s argument: “Natural selection selects for adaptive NP [neuro-physiological] properties.” [italics added by me]

      Now Bob, please tell us what it is in Platinga’s above presented ACTUAL ARGUMENT that suggests that physiological needs will lead to maladaptive neuro-physiological responses, such as “I’m sleepy” leading to the maladaptive neuro-physiological (NP) response of “eat palatable food” (etc). One of the key points in Plantinga’s above argument that you need to respond to is: “Indication is one thing; belief content is something else altogether, and we know of no reason (given materialism) why the one should follow the other.” All that natural selection is concerned with is that the NP response meet the NP need. It is not concerned with whether the belief leading to that NP response is true or not. A false belief leading to an adaptive NP response is just as good as a true belief leading to an adaptive NP response.

      Bob, with all due respect, once again, you need to respond to Plantinga’s ACTUAL ARGUMENT rather than a straw-man mischaracterization of it. You call his argument “nutty,” but the only thing here that is “nutty” is the extent of your mischaracterizations of his argument.

      • “The problem is that there is nothing in Plantinga’s argument that suggests that physiological needs lead to maladaptive neuro-physiological responses.”

        Then I must completely misunderstand the argument.

        As I stated in the post, Plantinga (apparently) is giving the example that our hominid Paul has, by dumb luck, a survival-promoting response to seeing a tiger. A response in accord with reality would be “Tigers are dangerous, and my proper response is to run away,” but he (again, just by luck) responds instead: “Tigers are cuddly and huggable, and my proper response is to run away.”

        Have I got it right?

        “Now please tell us what it is in Platinga’s above presented ACTUAL ARGUMENT that suggests that physiological needs will lead to maladaptive neuro-physiological responses, such as “I’m sleepy” leading to the maladaptive neuro-physiological (NP) response of “eat palatable food” (etc).”

        Are you asking, “Where in Plantinga’s argument did he even mention sleepiness leading to eat food?”? He doesn’t. I never said he did. I only said that Plantinga’s model of actions bearing no relationship with reality (the dumb luck approach to survival) is not what natural selection would cause.

        That was the point of the jumbled table connecting motivations with bizarre instinctive actions–evolution would simply not select for that. That creature would be too dumb to live.

        “It is not concerned with whether the belief leading to that NP response is true or not.”

        Right–natural selection doesn’t lead to beliefs that are correct. We’re both familiar with loads of mental biases and failings that humans are saddled with. But that doesn’t mean that natural selection can’t encourage traits that actually happen to mesh nicely with reality.

        “With all due respect, once again, you need to respond to Plantinga’s ACTUAL ARGUMENT rather than a straw-man mischaracterization of it.”

        Obviously.

        • Bob:

          Thanks for getting back to me. Plantinga is not in any way suggesting that survival promoting responses are dumb luck. As I cited Plantinga in my previous reply, Plantinga says (without any qualification) that “natural selection selects for adaptive NP [neuro-physiological] properties.”

          So, referencing your example of Paul and the tiger, Plantinga would not deny that natural selection would select the appropriate neuro-physiological [NP] response of running away or hiding. A NP response of running or hiding would be the “adaptive NP property” in such a case. Plantinga’s key point here is that “natural selection just has to take pot-luck with respect to the propositions or content determined by those adaptive NP properties.”

          So the “propositions or content” in your Paul and the tiger example could be anything from “I should run and hide from the tiger because doing so will make me rich and famous” to “I should run and hide from the tiger because it will make grow 2 inches taller” to the most accurate proposition (or content) of “I should run and hide from the tiger because it will prevent him from eating me.”

          In summary, natural selection will select adaptive NP properties (such as running and hiding from a tiger), but it CANNOT determine the propositions or content determined by those adaptive NP properties. This is why naturalism leaves us with no reason to think that the propositions or content of naturalism (or any other belief) contain any truth. It just leaves us with Charles Darwin’s “horrid doubt” that our beliefs can contain anything more than the “convictions of a monkey’s mind.”

          • (Trying again)

            As I said in my other comment, the readers of these comments are intelligent enough to recognize that you are trying to use semantic maneuvering to obscure the weakness of your stance.

            I love it! More scolding!

            Since you’ve failed on every attempt to read my internal motives, I suggest you stop and focus on, y’know, the argument.

            The definition of specific words is an irrelevant semantic issue…a smokescreen.

            I hate having to focus on definitions. When we are not using the same ones, or when you conflate two terms that are not synonymous, we must return to the dictionary. I’m just trying to avoid confusion—seems like a noble goal.

            The pertinent question regarding the existence of God is whether or not there exists morality above and beyond subjective human opinion.

            Yes and, yet again, I’ll state that I eagerly await your evidence that (1) this exists and (2) that we can reliably access it.

            If there is such a morality, then there needs to be a source for it. This is true regardless of what definition of “objective” one uses…Wm Lane Craig’s definition, or any other definition.

            Then let’s define “objective” as “shared.” The morality that is common to all humans (shared morality) that we get from evolution is then objective, and its existence proves God.

            Sorry—I’m not buying that flabby argument.

            You gave the example of the civil rights movement in your other comment. Prior to the civil rights movement, the subjective opinions of the white majority regarding racial discrimination were in worse compliance with the objective moral truth that racial discrimination is wrong.

            No more reference to objective morality until you show that it exists.

            The leaders of the civil rights movement appealed to a higher objective moral standard

            Handwave all you want that a shared or viscerally felt moral claim is objective, but that’s no argument that objective morality exists.

            If the leaders of the civil rights movement did not succeed by appealing to a higher objective moral truth, then how did they succeed in changing the subjective opinions of so many people in our society?

            Oh, please. You act like it was easy.

            Black pastor: “Hey, white folks—did you ever consider the fact that we’re all the same, regardless of skin color?”

            Racist person: “By golly … I never did. But I see it now! Thanks, black pastor, for setting me straight! And apologies for that whole Jim Crow thing.”

            How did they get so many white people to change their subjective opinions about race considering that there was no personal benefit for them to do so?

            Did you never pay attention to U.S. History back in 8th grade? Slavery wasn’t overturned by pointing out objective truths, it was won by a long civil war. Civil rights didn’t happen overnight, but it was a long process. Citizens who weren’t on board either died out (replaced by younger people who accepted the new ideas) or (mostly) mellowed as they saw the writing on the wall.

            If that’s the power of objective morality, I don’t think much of it.

            • Scott Youngren says:

              Bob,

              I have bolded your comments, and responded below:

              Then let’s define “objective” as “shared.” The morality that is common to all humans (shared morality) that we get from evolution is then objective, and its existence proves God.

              Ok, we are getting somewhere. You agree that humans share a common moral sense. For example, most humans share the moral belief that it is wrong to 1) try to exterminate entire ethnic groups…as with the Nazis 2) rape 3) force people into slavery based upon skin color 4) torture babies, etc. etc.

              But you feel that this common moral sense does not point to a morality which transcends mere human opinion. Rather, it just points to common “viscerally felt” reactions which we have developed from evolution. Do I have this right, Bob? I don’t want to misrepresent you.

              So, by your logic, it follows that the disagreement over slavery between northerners and southerners (which led the south secede from the union, thereby causing the Civil War) was caused by different “viscerally felt” reactions to people being forced into slavery based upon skin color. By your logic, there is nothing really morally wrong with forcing people into slavery based upon skin color (in a sense that transcends mere human opinion). Rather, southerners evolved different “viscerally felt” reactions to slavery than northerners, and there is no higher moral truth that makes slavery morally inferior to racial equality. Or did the southerners merely ignore their “viscerally felt” reactions which told them that slavery is wrong?

              By your logic, the disagreement over the morality of trying to exterminate the Jews (between the Nazis and the allied nations) was really just caused by the fact that the Nazis evolved different “viscerally felt” reactions to genocide than people in the allied nations. There is nothing morally inferior about genocide, in a sense that transcends subjective human opinion. Rather, the Nazis evolved different “viscerally felt” reactions to genocide than people in, say, England. Either that, or the Nazis evolved the same “viscerally felt” reactions regarding genocide, but just chose to ignore them.

              Which is it Bob? Or is there a third option which I failed to mention?

              If we merely evolved “viscerally felt” reactions to such things as slavery, rape, burglary, and genocide, then what was the evolutionary purpose of these common “viscerally felt” reactions? And why did the Nazis evolve a different viscerally felt reaction to people in the allied nations? Or if the Nazis had the same viscerally felt reactions to genocide as people in England, what evolutionary purpose did it serve for them to ignore their commonly felt viscerally felt reactions?

              What evidence do you provide to support your stance that these “visceral reactions” are the result of evolution? What “simple test” do you cite to back up your stance with regards to this issue?

              Would you agree with the famous atheist Richard Dawkins that we are really just “survival machines, blind robot vehicles” with no purpose other than to pass on our genes? Would you agree with the atheist stance that we are ultimately just organisms without a transcendent soul? Are we are ultimately just bundles of molecules which are completely specified by physical and chemical properties? Or do we have something which transcends our physical bodies…like a soul? Please let me know your stance with regards to this issue, Bob.

              Lastly, I need to do a copy and paste since you failed to respond to one of my previous comments:

              You say that beliefs need to be “reliably verifiable.” You believe that beliefs must be able to “pass simple tests” to see if they are objectively true. But your view is self-refuting, since it cannot pass any such “simple tests.” This is the self-refuting incoherence in which atheism is caught. Please describe for me the “simple tests” which you would use to verify the truth of the following statements you have made (in bold):

              1)If you mean a reliably verifiable belief, then “Bob has a yellow car” would be objectively true. There are simple tests that you could use to find this to be a true statement. And with this definition, these shared moral beliefs you list would be “objective.”

              What “simple test” did you apply to verify that “there are simple tests” which one could use to find out if a statement is objectively true? Perhaps it was a chemistry experiment involving a bunsen burner and test tubes? Did you just assume that your methods for verifying objective truth are objectively true, or did you verify them with some sort of empirical observation (“simple test”)?

              2)”…your morality is flawed. Indeed, anywhere it diverges from mine, it’s flawed.”

              What simple test did you use to verify that my morality is flawed where it diverges from yours? A biology experiment involving observation under a microscope, perhaps?

              3)“objectively true” = viscerally felt.

              By your own standards, if people share viscerally felt reactions, and there is no objective morality, then you should be able to cite the simple tests which you used to verify the objectivity of this belief. How did you observe these reactions inside of people’s minds? What “simple test” did you use to verify that humans share “viscerally felt” reactions, and that these are not really just robot-like reactions with no feeling behind them.

              4)”…you should be looking for the standard we both use, the dictionary.”

              Please describe the “simple test” (or empirical observation) which you used to verify the objective truth of this statement. What test did you use to verify the objective truth of your belief that I should be using the dictionary definition?

              Scott

              • Scott:

                Ok, we are getting somewhere. You agree that humans share a common moral sense.

                If this is progress, I’m surprised that we didn’t make this breakthrough earlier. I wish you’d asked.

                We’re the same species. Yes, we have a similar take on moral issues.

                For example, most humans share the moral belief that it is wrong to 1) try to exterminate entire ethnic groups…as with the Nazis

                Yes, they do. At some point, you’ll have to respond to God’s demand for genocide in the OT.

                But you feel that this common moral sense does not point to a morality which transcends mere human opinion. Rather, it just points to common “viscerally felt” reactions which we have developed from evolution. Do I have this right, Bob?

                More or less. “Transcends” is fuzzy. I’d rather say that I reject Craig’s definition, “moral values that are valid and binding whether anybody believes in them or not,” for lack of evidence.

                By your logic, there is nothing really morally wrong with forcing people into slavery based upon skin color (in a sense that transcends mere human opinion).

                What does “really” mean in this case? Sounds like “really morally wrong” means “objectively morally wrong.” If you mean “objectively,” then say so. This conversation suffers from too many ill-defined words already.

                To your point: no, I don’t see evidence for objective moral truths. I’ve already made that clear.

                If we merely evolved “viscerally felt” reactions to such things as slavery, rape, burglary, and genocide, then what was the evolutionary purpose of these common “viscerally felt” reactions?

                I’m trying to help you out here. If you don’t think that these moral beliefs are viscerally felt, then say so. I’m trying to nail down this squishy word “objective morality.”

                Would you agree with the famous atheist Richard Dawkins that we are really just “survival machines, blind robot vehicles” with no purpose other than to pass on our genes?

                Every person creates his own purpose.

                I see no evidence for absolute or objective purpose at all. You disagree? Then show me the evidence.

                And speaking of which, I’ve yet to see you defend your claim of objective morality. I’m still waiting.

                Would you agree with the atheist stance that we are ultimately just organisms without a transcendent soul?

                Obviously.

                Are you going to do anything but ask questions?

                You say that beliefs need to be “reliably verifiable.” You believe that beliefs must be able to “pass simple tests” to see if they are objectively true. But your view is self-refuting, since it cannot pass any such “simple tests.” This is the self-refuting incoherence in which atheism is caught.

                First, let me congratulate you for making an insulting meta point in your last sentence. You get bonus points for that.

                Your question is nonsense. Here’s what I said: “If you mean a reliably verifiable belief, then “Bob has a yellow car” would be objectively true. There are simple tests that you could use to find this to be a true statement.”

                Can we declare this pointless busywork completed? Or are you still confused?

                Please describe for me the “simple tests” which you would use to verify the truth of the following statements you have made (in bold):
                1)If you mean a reliably verifiable belief, then “Bob has a yellow car” would be objectively true. There are simple tests that you could use to find this to be a true statement. And with this definition, these shared moral beliefs you list would be “objective.”
                What “simple test” did you apply to verify that “there are simple tests” which one could use to find out if a statement is objectively true?

                No—I guess you’re still confused.

                I’m making no universal statement. I’m saying that there are simple tests by which you could verify “Bob has a yellow car” is a true statement. Why is this hard?

                Did you just assume that your methods for verifying objective truth are objectively true, or did you verify them with some sort of empirical observation (“simple test”)?

                I was desperately trying to nail down your use of “objective.” Does it mean “reliably verifiable belief”? If you don’t like this definition, then give me a better one.

                Are you always this difficult to converse with or are you putting in special effort?

                • Scott Youngren says:

                  Bob,

                  I don’t think that there are any third party readers of these comments who are fooled by your evasion tactics.

                  It is thoroughly obvious that you are trying to divert attention from the weakness of your stance by using red-herring arguments. As the preceding Wikipedia link mentions, “A red herring is something that misleads or distracts from a relevant or important issue. It may be either a logical fallacy or a literary device that leads readers or audiences towards a false conclusion.” This is the same semantic maneuvering which I referred to in an earlier comment.

                  For example, Bob, do you really think that you are convincing third party observers of this thread by switching the discussion to the definition of the words really, objective, and transcend, when I ask you a question that you cannot coherently respond to?!

                  I just don’t think that there is anyone reading these comments who cannot clearly perceive that you are trying to use diversionary sleight-of-hand (related to the definitions of words) when I ask you a question that you cannot coherently respond to from the framework of your worldview. The only “squishy” thing going on here is your attempt to evade my arguments by switching the discussion to the definitions of words.

                  For example, I made it clear that I was using the word objective in the sense of transcending mere human opinion. Then you switch the discussion to the definition of the word transcend. At some point, Bob, you are going to have to engage in the discussion and stop trying to create distractions.

                  I will rephrase my previous questions in hope that you will answer them. You can try to distract attention yet again by questioning the definitions of words, Bob, but if you do this, you are just further advertising your need to evade my questions.

                  Bob, is there anything about slavery that is morally inferior to racial equality…in a sense that is true independent of (or beyond) human opinion? (If you question my definition of a word such as independent or beyond in your reply, I just don’t think that you are going to fool anyone with this evasion tactic).

                  Bob, is there anything about burglary that is morally inferior to the respect of property rights…in a sense that is true independent of (or beyond) human opinion? (If you question my definition of a word such as independent or beyond in your reply, I just don’t think that you are going to fool anyone with this evasion tactic).

                  Please also answer my previous questions, which I will copy and paste below:

                  If we merely evolved “viscerally felt” reactions to such things as slavery, rape, burglary, and genocide, then what was the evolutionary purpose of these common “viscerally felt” reactions? And why did the Nazis evolve a different viscerally felt reaction to people in the allied nations? Or if the Nazis had the same viscerally felt reactions to genocide as people in England, what evolutionary purpose did it serve for them to ignore their commonly felt viscerally felt reactions?

                  What evidence do you provide to support your stance that these “visceral reactions” are the result of evolution? What “simple test” do you cite to back up your stance with regards to this issue?

                  Would you agree with the famous atheist Richard Dawkins that we are really just “survival machines, blind robot vehicles” with no purpose other than to pass on our genes? Would you agree with the atheist stance that we are ultimately just organisms without a transcendent soul? Are we are ultimately just bundles of molecules which are completely specified by physical and chemical properties? Or do we have something which transcends our physical bodies…like a soul? Please let me know your stance with regards to this issue, Bob.

                  Lastly, I will YET AGAIN copy and paste my previous questions below. You YET AGAIN failed to respond. In an earlier comment, you very clearly said that beliefs need to be “reliably verifiable.” You believe that beliefs must be able to “pass simple tests” to see if they are objectively true. But your views are self-refuting, since they cannot pass any such “simple tests.” This is the self-refuting incoherence in which atheism is caught. Please describe for me the “simple tests” which you would use to verify the truth of the following statements you have made (in bold):

                  1)If you mean a reliably verifiable belief, then “Bob has a yellow car” would be objectively true. There are simple tests that you could use to find this to be a true statement. And with this definition, these shared moral beliefs you list would be “objective.”

                  What “simple test” did you apply to verify that “there are simple tests” which one could use to find out if a statement is objectively true? Perhaps it was a chemistry experiment involving a bunsen burner and test tubes? Did you just assume that your methods for verifying objective truth are objectively true, or did you verify them with some sort of empirical observation (“simple test”)?

                  2)”…your morality is flawed. Indeed, anywhere it diverges from mine, it’s flawed.”

                  What simple test did you use to verify that my morality is flawed where it diverges from yours? A biology experiment involving observation under a microscope, perhaps?

                  3)“objectively true” = viscerally felt.

                  By your own standards, if people share viscerally felt reactions, and there is no objective morality, then you should be able to cite the tests which you used to verify the objectivity of this belief. How did you observe these reactions inside of people’s minds? What “simple test” did you use to verify that these “visceral reactions” are not just the mechanistic and mindless behavior of “survival machines – robot vehicles blindly programmed to preserve the selfish molecules known as genes”…as the famous atheist biologist Richard Dawkins describes human beings?

                  4)”…you should be looking for the standard we both use, the dictionary.”

                  Please describe the “simple test” (or empirical observation) which you used to verify the objective truth of this statement. What test did you use to verify the objective truth of your belief that I should be using the dictionary definition?

                  I really prefer not to use all caps, but when an atheist continues to evade it sometimes becomes necessary: BOB, HOW CAN YOUR OWN BELIEFS BE SAID TO BE “RELIABLY VERIFIABLE” IF YOU CANNOT PROVIDE THE “SIMPLE TESTS” WHICH YOU USED TO ESTABLISH THEIR RELIABILITY? YOUR BELIEFS DO NOT MEET THEIR OWN STANDARDS, AND ARE THEREFORE SELF-REFUTING.

      • Terry S says:

        Scott: that the natural world is self-existent

        Terry S: What is self-existent? If you mean did it create itself the answer is no, it didn’t. It has always existed. There was no beginning.

        Scott: Atheists insist that naturalism is the worldview most compatible with science

        Terry S: Atheist insist no such thing. I am an atheist and science has nothing to do with my theological position. Simply put, the theist claim is that god exists. The atheist claim is that theists haven’t supported their claim. It has nothing to do with science.

        • Except that the universe very clearly did have a beginning, as I demonstrate in Isn’t the Universe Eternal (Thus doing away with the need for a creator).

          Further, you assert that theists do not support their claim for God. But this is an utterly remarkable statement considering that you have not responded to the support for belief in God that I provide in such essays as Why Life Could Not Have Emerged Without God and The Ultimate Cart Before the Horse (Why Atheism is Illogical), and Why God? Why Not Just Plain Luck? and God Is Real: Why Modern Physics Has Discredited Atheism, etc., etc.

          • Alex says:

            I am no Christian. But I found this article as a result of a sudden insight I had last night. If naturalist evolution is to be beileved, then the conclusion is that evolution produced humans, then credited “creation” to a god(s) figure. Almost all humans in known history have believed in the supernatural. These beliefs have to have been a product of evolution (if we believe naturalism.) Evolution (speaking of it as a thing) produced people, gave them intelligence, and then lied to them about who made them, as can be observed by the number of people who have believed they were created by the supernatural. This is a glaring hole in the idea that our beliefs generally match up with the truth under naturalism. Said once more, taking the naturalist position results in believing that evolution produced the finest intellect ever, then created an imaginary thing called the supernatural, and gave it credit for creation. Hence, evolution lied to us, in a manner speaking. The problem with the naturalist position is that we may never be able to know what else evolution is tricking us into believing, for we know it tricked us into believing in god under their worldview.

            I see this as a fatal flaw. It is proof that evolution will, at least at times, select for completely untrue beliefs – beliefs which have a huge impact on people’s lives. Also if this is true, and we have somehow come to understand it, then shouldn’t naturalists wish to continue to teach god – considering it was an evolutionary adapatation which worked? Why rock the boat naturalists? The experiment of history selected for belief in the supernatural. That seems like a good enough reason to stick with it, instead of gambling on changing the world-wide belief structure to naturalism, which has not been tested for its survival benefits. I think pure naturalists just don’t think things through.

            I am no Christian. I don’t think the “god did it” explanation is any better. It may be worse. But naturalists often try to shame god believing people – as if they were less thoughtful. The truth is that naturalists have not presented a well-enough thought out argument for making the switch. I personally don’t think either system holds all the answers. Naturalism and supernaturalism both suffer form the recognition that the human mind is faulty. So, both have their valid internal reasons to only take themselves so seriously, and to never rule the other out completely. Because in the end we just don’t know for sure.

            • Terry S says:

              @ Alex on July 18, 2013 at 3:34 am

              You are making the mistake of thinking that a belief in the supernatural is a step in evolution. It’s not. The ability to form a belief might be but a belief in anything specific has nothing to do the evolutionary process.

              Alex: “Because in the end we just don’t know for sure.”

              Terry S: I know, So you must be mistaken.

            • Alex:

              In reply to your comment, there is one item I need to point out: You write that “evolution will, at least at times, select for completely untrue beliefs.” This is not quite accurate. It is not merely that evolution will, at times, select for untrue beliefs. Rather, it is that evolution does not select for ANY beliefs. Evolution can only select for neuro-physiological (NP) responses, NOT for the beliefs attached to those NP responses.

              So, regarding the survival-enhancing NP response of running and hiding from tigers, the belief “I should run and hide from tigers because it will make me rich and famous” provides JUST AS MUCH SURVIVAL VALUE as the belief “I should run and hide from tigers because it will prevent me from being eaten.” Both beliefs are attached to the same NP response. But evolution only selects for survival, NOT FOR TRUTH. This is because truth does not enhance survivability…only appropriate NP (neuro-physiological) responses enhance survivability.

              Therefore, the beliefs attached to a NP response have to be selected “pot luck” (in Plantinga’s words).

              Thus naturalism provides us with NO WAY of giving us access to truth. Under naturalism, we must therefore reject ALL human beliefs…such as belief in naturalism (hence the title of this essay).

              Alex, you characterize theism as a “God did it” explanation. I cannot put words in your mouth, but it seems to me like you are here suggesting that theism is an “argument from ignorance.” But nothing could be further from the truth.

              There are numerous reasons why the theistic stance is an argument from knowledge. I will present just a few introductory reasons below:

              1) The mind-boggling specificity which was necessary to produce a universe capable of producing intelligent life (as I describe in Is there a God? (What is the chance that our world is the result of chance?) and OK…I want numbers. What is the probability that our world is the result of chance?

              In fact, the majority of astrophysicists accept theistic or deistic explanations for the origin of our universe for these reasons. A citation from the first above essay:

              Anyone who probes into the origins of our universe soon becomes overwhelmed by the evidence that “the universe is a put-up job” (or, the product of deliberate, conscious intent), in the words of Cambridge University physicist and mathematician Fred Hoyle.

              So compelling, in fact, has become the case for the universe as the product of a conscious creator that astrophysicist Hugh Ross, a former post-doctoral fellow at the California Institute of Technology observes (in his book The Creator and the Cosmos: How the Greatest Scientific Discoveries of the Century Reveal God) that:

              “Astronomers who do not draw theistic or deistic conclusions are becoming rare, and even the few dissenters hint that the tide is against them. Geoffrey Burbidge, of the University of California at San Diego, complains that his fellow astronomers are rushing off to join ‘The First Church of Christ of the Big Bang.’”

              2) The evidence from modern physics, as presented in <em>God is real…Why modern physics has discredited atheism. A couple excerpts:

              Virtually everyone is familiar with the popular conundrum, “Which came first…the chicken or the egg?” But probably very few people realize that the question of God’s existence, in a very real sense, boils down to what is likely the ultimate chicken-or-the-egg conundrum: Which came first, mind or matter? In other words, is mind (or “consciousness”) the product of matter, or is matter the product of mind? Is our universe—at its core—a material universe, or is it a mental (or spiritual) universe?

              Well…when Max Planck (the Nobel Prize winning physicist who founded quantum theory) says…

              “As a man who has devoted his whole life to the most clear headed science, to the study of matter, I can tell you as a result of my research about atoms this much: There is no matter as such. All matter originates and exists only by virtue of a force which brings the particle of an atom to vibration and holds this most minute solar system of the atom together. We must assume behind this force the existence of a conscious and intelligent mind. This mind is the matrix of all matter.”

              and Albert Einstein says…

              “Everyone who is seriously involved in the pursuit of science becomes convinced that a spirit is manifest in the laws of the Universe–a spirit vastly superior to that of man, and one in the face of which we with our modest powers must feel humble.”

              and the Nobel Prize winning physicist Eugene Wigner says…

              “When the province of physical theory was extended to encompass microscopic phenomena, through the creation of quantum mechanics, the concept of consciousness came to the fore again; it was not possible to formulate the laws of quantum mechanics in a fully consistent way without reference to the consciousness,”

              and

              “The content of consciousness is an ultimate reality.”

              and the great physicist Sir Arthur Eddington says…

              “The idea of a universal mind or Logos would be, I think, a fairly plausible inference from the present state of scientific theory.” [“Logos” is defined as “the word of God, or principle of divine reason and creative order.”]

              and the knighted mathematician, physicist and astronomer Sir James Jeans says (in his book The Mysterious Universe)…

              “There is a wide measure of agreement which, on the physical side of science approaches almost unanimity, that the stream of knowledge is heading towards a non-mechanical reality; the universe begins to look more like a great thought than a great machine. Mind no longer appears as an accidental intruder into the realm of matter. We are beginning to suspect that we ought rather to hail mind as the creator and governor of the realm of matter.” (italics added)

              …there can be no question on which side of this debate modern physics falls.

              3) The evidence from experience, as presented (in part) in Has anyone met God and returned to tell about it?. Albert Einstein said that “the only source of knowledge is experience.” Even scientific knowledge is really just an accumulation of experiential knowledge. A couple excerpts from the above essay:

              An entire field of research has sprung up to analyze the near-death experience (NDE) phenomenon. Researchers from the fields of medicine and psychology have come together to form the International Association of Near- Death Studies (IANDS, website iands.org) and the Near Death Experience Research Foundation (NDERF, website: nderf.org).

              In 2005, IANDS released The Handbook of Near-Death Experiences to summarize the conclusions of 30 years of research in this field. Some of the revelations featured in this book (which appear below) should come as a wake-up call to those inclined to doubt the existence of the Deity:

              “NDErs often believe that they have survived because God willed it and had a divine purpose in bringing them back…They have experienced the love of God and been changed by it (Grosso 1981). Many have come face-to-face with a personal God with whom they continue to maintain a loving relationship.”

              “…for most the result appears to be a spiritual awakening. The NDE often brings with it a spiritual certainty and intense desire to conform one’s life to divine will. The new relationship with what is often a personal God becomes central to the NDErs’ lives.”

              In part because of the sheer volume of NDE accounts, it has become a phenomenon that is difficult to ignore. And, as Patrick Glynn notes in his book God: The Evidence, “the majority of researchers who have investigated the phenomenon, generally professionals with medical, psychological, or other scientific training—many of whom started out as skeptics—have concluded that these experiences are authentic.”

              Evidence for the Afterlife by Jeffrey Long, MD cites, “Reunion with deceased loved ones and with God, angels, Jesus” as one of the most frequently reported phenomena from NDEs.

              Scott

        • Alex says:

          @Terry. I could not reply below so I will reply here.
          According to naturalism all things are physical/natural, even ideas or beliefs. You suggest that evolution did not select for beliefs which would help the organism survive. You state that you “know” that.

          Alex: Evolution gave man the ability to believe and know, but then selected for a lie (the supernatural.) This shows evolution can select for falsehoods.

          Terry: Evolution may have given us the ability to believe, but it did not affect the content of our beliefs.

          Alex: Then what you are saying right now could be faulty right?

          Terry: Nah. Even though my belief system is primitive because evolution never improved on the original design, I’m pretty sure that I am right.

          Alex: Terry, you have thoroughly proven your point. You win.

      • doops says:

        Naturalism is based in evolution [strict] and evolution is random and mindless. You are incorrect. There is only a binary choice. Either Evolution (mindless, which is uncontrolled, which is “random” or better “unguided”) or Creation.

        Pick.
        1 – Evolution > Naturalism > > Random > Atheism > …
        OR
        2. Creationism > Guided > Non Random > Theism > …

        You’re looking at the branches, but you got to look at the root.

        • God Evidence says:

          Doops,

          No, this is a deeply entrenched cultural myth, and a false dichotomy. Indeed, the real conflict is not between Christianity and evolution. Rather, the real conflict (as the Christian philosopher Alvin Plantinga points out in his book Where the Conflict Really Lies) is between Christianity and atheistic philosophical add-ons to evolutionary theory. Despite atheist rhetoric to the contrary, Christians do not deny evolution since the term evolution only means change over time…when stripped of philosophical add-ons such as the insistence that this change over time is driven by random and purposeless (rather than intelligent and goal driven) processes. Most Christians objecting to evolution are really objecting to evolution with the atheistic philosophical add-ons of randomness and purposelessness.

          It is just that an enormous amount of distortion (and outright denial) is necessary to apply the terms random and purposeless to evolution…especially in light of the science which has emerged since the time of Charles Darwin.

          Atheists such as Dawkins forget that randomness can never be verified

          First of all, as Perry Marshall points out in his book Evolution 2.0, it is possible to prove that a pattern is non-random, but there is no mathematical procedure for proving that a pattern is random. Atheists can only assume that genes mutate randomly…because their worldview demands it. But the random mutation hypothesis can never be verified, and therefore it stands in opposition to the scientific method itself.

          Marshall cites the renowned mathematician Gregory Chaitin from his paper Randomness and Mathematical Proof:

          “Although randomness can be precisely defined and can even be measured, a given number cannot be proved to be random. This enigma establishes a limit to what is possible in mathematics.”

          Evolution is directed, not random…but Dawkins leaves this out.

          But the fundamentally unscientific nature of the random mutation evolutionary hypothesis is only its first problem. Marshall continues by calling attention to the research which has proven that evolution is clearly NOT random. Scientists such as the famous evolutionary biologist and geneticist Theodosius Dobzhansky conducted six decades of research in which fruit flies were exposed to radiation in order to induce the mutation of genes, with the intent of accelerating evolution. But after 60 years of research, and despite the fact that a new generation of fruit flies occurs every 11 days, no new species emerged, or even a new enzyme. Rather, the only results are what amount to frankenflies, including mutant fruit flies with legs growing out of their heads where their antennae belong.

          So, if it is not random, how does change over time (the definition of evolution stripped of its atheistic philosophical add-ons) really happen?

          Marshall answers:

          “Remember the fruit fly experiments? [Nobel Prize-winning biologist Barbara] McClintock’s experiments were similar. She too used organisms damaged by radiation. She discovered that radiation broke chromosomes and triggered editing systems in real time. Cells would reconstruct the damaged chromosome with another section of radiation-broken genetic material.”

          “…Barbara McClintock had discovered that plants possess the ability to recognize that data has been corrupted. Then they repair it with newly activated genome elements, and in the process of repairing the data, the plants can develop new features!”

          Random mutation and natural selection is not what drives evolution (as Darwinism insists). Rather, directed processes drive evolution. The directed process mentioned above is known as transposition, and amounts to a cut/copy/paste of genetic information within a cell. The discovery of transposition won Barbara McClintock the Nobel Prize in biology, and her face on a U.S. postage stamp.

          And despite the fact that no legitimate biologist denies transposition, Marshall notes, it is noticeably absent from popular presentations of evolution, such as in books by atheistic evolution promoters Richard Dawkins and Jerry Coyne. Scientists with an atheistic agenda do not wish to call attention to directed evolutionary processes such as transposition.

          Physicist Amit Goswami echoes Marshall’s point about the directed (as opposed to random and mindless) nature of evolution in his book Creative Evolution: A Physicist’s Resolution Between Darwinism and Intelligent Design. Atheistic scientists argue in favor of upward causation, in which elementary particles make atoms, which make molecules, which make living cells, which make the brain, which produces consciousness. According to the upward causation model, then, everything begins with elementary particles, and winds up with consciousness (in human brains), as the result of mindless and random processes working over millions of years. But, as Goswami points out, downward causation (in which a consciousness comes first) is the actual state of affairs:

          “The new evidence suggests that certain bacteria, when threatened with mass starvation, accelerate their own mutation rate to evolve to a new species that can survive on the available food (Cairns, Overbaugh, and Miller 1988). This behavior is called directed mutation. Critics of directed mutation point out that under starvation perhaps the mutation rate of all the genes is enhanced, not just the one needed for survival. But even so, the question remains: What enhances the mutation rates? The correct explanation is to see this phenomenon as direct evidence in favor of downward causation (Goswami and Todd 1997) and the causal efficacy of organisms, as also propounded by organismic biologists.”

          Mind comes first, matter comes from mind

          So what (or rather who) is responsible for this downward causation? Goswami responds that the only answer can be God, in part because an immaterial conscious mind is required to explain the famous “observer effect” in physics. The “observer effect” refers to the conclusion of modern physics that, prior to observation by a conscious observer, particles exist only in an immaterial form known as a possibility wave (or probability wave). It is only after an observation is made by a conscious observer that these possibilities “collapse into actuality,” thereby taking on material form. Readers who find this bizarre or difficult to understand are in good company. Even the world’s most elite physicists are amazed and puzzled by the observer effect. However, it has been repeatedly scientifically verified. [Please click here to watch a video explaining the observer effect.] Goswami writes:

          “If the idea of downward causation were an isolated idea invented to solve the special problems of fast-tempo evolution and purposiveness of life, if it were needed nowhere else in science, then it could not be called a scientific idea, end of story. But the intriguing situation is this: The idea of a God as an agent of downward causation has emerged in quantum physics (Goswami 1989, 1991, 1993, 2000, 2002; Stapp 1993; Blood 1993, 2001) as the only legitimate explanation of the famous observer effect. (Readers skeptical about this statement should see these original references, especially Goswami 2002.)”

          Downward causation (in which a conscious agent comes first) is no doubt a bizarre (even mind-bending) concept for persons raised in a culture which has deeply entrenched assumptions supporting the upward causation model. But, far from being a fringe concept, downward causation is a virtually undeniable conclusion of modern physics, as Goswami notes.

          Physicist Richard Conn Henry from Johns Hopkins University agrees with Goswami that downward causation by God is the only reasonable conclusion one can draw from modern physics:

          “Why do people cling with such ferocity to belief in a mind-independent reality? It is surely because if there is no such reality, then ultimately (as far as we can know) mind alone exists. And if mind is not a product of real matter, but rather is the creator of the illusion of material reality (which has, in fact, despite the materialists, been known to be the case since the discovery of quantum mechanics in 1925), then a theistic view of our existence becomes the only rational alternative to solipsism.” [“Solipsism” is defined as “the view or theory that the self is all that can be known to exist.”]

          Indeed, the founder of quantum physics himself, the Nobel Prize-winning physicist Max Planck, was referring to downward causation, in which a conscious mind (read: God) comes first, and produces matter, when he wrote:

          “I regard consciousness as fundamental. I regard matter as derivative from consciousness. We cannot get behind consciousness. Everything that we talk about, everything that we regard as existing, postulates consciousness.”

          Planck also wrote:

          “As a man who has devoted his whole life to the most clear headed science, to the study of matter, I can tell you as a result of my research about atoms this much: There is no matter as such. All matter originates and exists only by virtue of a force which brings the particle of an atom to vibration and holds this most minute solar system of the atom together. We must assume behind this force the existence of a conscious and intelligent mind. This mind is the matrix of all matter.”

  2. “Plantinga says (without any qualification) that “natural selection selects for adaptive NP [neuro-physiological] properties.””

    Right. And that’s why I tried to rephrase it so we had a common understanding. I have no idea what Plantinga’s trying to say.

    I usually get my information about biology from biologists, so I don’t feel bound by Plantinga. Let’s just drop Plantinga. I’d rather discuss your understanding of the argument rather than have to wade through Plantinga’s and worry that I’m misunderstanding somehow.

    We seem to be going around in circles. Yes, I understand that you’re saying that we hypothesize that Paul has internal programming that says, “I should run and hide from the tiger because X,” where X could be anything. And if X could be any of, say, 1000 things (only one being “because the tiger would eat me, and that would be bad”), then it’s just dumb luck that the correct one would be chosen.

    As I said earlier, your challenge is to show that these 1000 justifications would be equally likely, since this isn’t what evolution says. Evolution says that there’s no guarantee that the justification would be accurate but that that an accurate justification is more likely than any other option. Your imaginary Paul who thinks, “I should run and hide from tigers because that’ll make me rich and famous” (and similarly, “I should avoid quicksand because it’s bad for my complexion” and “I should eat cooked food instead of raw because it’ll make the sun come up tomorrow”) is just too dumb to live.

    (And speaking of personalities, let’s leave Charles Darwin out of it as well. What Darwin wrote is interesting only for historians and is irrelevant to evolution today.)

    Let me say, however, that I do now see the distinction you’re making between the action and the justification for that action. My post was focused on actions, and if I were to tackle this topic again, I’d need to make that correction. Thanks for that.

    • Bob:

      You say, “Evolution says that there’s no guarantee that the justification would be accurate but that an accurate justification is more likely than any other option.” But how does evolution demonstrate that an accurate justification is more likely than any other option? You take this for granted and you ignore Plantinga’s point that natural selection selects for NP properties that improve an organisms chances of survival, but does not select for the content or propositions of those NP properties. This is because the content or propositions of NP properties have absolutely nothing to do with improving or harming an organisms chances of survival….only the NP properties themselves (such as the NP property of having an inclination to run from tigers) effect survivability. A NP property of running and hiding from tigers with the content or proposition of “doing so will make me rich and famous” has just as much survival value as a NP property of running and hiding from tigers with the content or proposition of “doing so will protect me from being eaten.”

      And when you assert that “Your imaginary Paul who thinks ‘I should run and hide from tigers because that’ll make me rich and famous’ is just too dumb to live,” you are again overlooking the fact that the content or proposition of the NP property has absolutely nothing to do with improving or harming survival value. The content or proposition of “running from tigers will make me rich and famous” is plenty absurd to you and I, but, from an evolutionary perspective, there is nothing about this proposition that will either harm or improve survivability.

      • You take this for granted and you ignore Plantinga’s point that natural selection selects for NP properties that improve an organisms chances of survival, but does not select for the content or propositions of those NP properties.

        In the domain of biology, I have little interest in what Plantinga thinks. He’s not a biologist, right? What I’d like to see is a biologist confirming your claim that any justification (answer to “why?”) is as likely as any other.

        • So you are suggesting that the content of a person’s thoughts lies within the scope of biology? How can the content of a person’s thoughts lie within the scope of biology? Can you please explain? Can the thought “I want to visit the zoo today,” for example, be studied by a biologist? How about the thought, “I prefer living in New York to Los Angeles”?

          But since you have asked to “see a biologist confirming [my] claim,” I will humor you. Sean McDowell notes in his essay Are Science and Christianity at Odds?:

          “Toward the end of The God Delusion, [atheist biologist Richard] Dawkins admits that since we are the product of natural selection, our senses cannot be fully trusted. …Since a human being has been cobbled together through the blind process of natural selection acting on random mutation, says Dawkins, it’s unlikely that our views of the world are completely true.” (Please see pages 411-417 of The God Delusion).

          Also, did you forget that I furnished Charles Darwin’s views on this subject matter (in the essay)?

          Further, when you say that you have little interest in what Plantinga thinks, you are trying to reframe his argument as an opinion. Everyone is entitled to their opinion, but what Plantinga has done is construct a rational argument. Can you respond to Plantinga’s argument with a counter-argument? That is the question.

          Please also note that an argument is not made valid or invalid by the person who presents it. Rather, an argument is made valid or invalid by the strength or weakness of its premises. For example, the argument “it is in the best interest of our society to prevent contagious diseases” is just as valid when presented by Bozo the Clown as when presented by a public health expert. And even when such an argument is presented by a public health expert, his or her premises would still be subject to examination and validation by non-public health experts.

          So even if the content of a person’s thoughts fell within the scope of biology, the arguments made by a biologist on this subject matter would still be subject to logical analysis.

          • So you are suggesting that the content of a person’s thoughts lies within the scope of biology?

            I’m saying that, if we’re critiquing evolution, we should look to Biology to see what it says.

            since we are the product of natural selection, our senses cannot be fully trusted. …Since a human being has been cobbled together through the blind process of natural selection acting on random mutation, says Dawkins, it’s unlikely that our views of the world are completely true.”

            Yep. I’ve already said and agreed to that.

            when you say that you have little interest in what Plantinga thinks, you are trying to reframe his argument as an opinion.

            When I say that I have little interest in what Plantinga things, I’m saying that, when the topic is biology, I won’t get a critique from anything less than a biologist.

            Am I confused here? It seems like Plantinga sketches out one aspect of how evolution works: that the justification behind an action is immune to natural selection. To our example, the correct justification of Paul’s action (tigers are dangerous and it sucks to be eaten) is just one of 1000 equally likely justifications for natural selection to select.

            OK, he’s made his claim about evolution. Now I want to see if that claim stands up to scrutiny within biology.

            (And no, my points don’t fail due to the genetic fallacy.)

            • Bob:

              Sorry about the delayed reply, I have been busy with work.

              I’m saying that, if we’re critiquing evolution, we should look to Biology to see what it says.

              So you apparently ARE saying that the contents and propositions of one’s thoughts is something that fits within the realm of biology? What do you suppose biology would have to say about the content of my thought “I prefer living in New York to living in Los Angeles.” Or my thought “I would like to visit the zoo today”?

              Either the contents of one’s thoughts fits within the scope of biology or it does not. Which is it? When you say “if we are critiquing evolution, we should look to biology to see what it says,” it very much sounds like you are implying that the content and propositions of one’s thoughts fits within the realm of biology.

              Yep. I’ve already said and agreed to that.

              OK, you have agreed to what Dawkins says about our inability to trust our views of the world. Then what is your basis for trusting the naturalist view of the world (in which you appear to believe) which says that the universe is self-existent and therefore does not require a transcendent cause? How about any of the beliefs that you or any other naturalist holds about evolution? What would be the basis for trusting any of those beliefs? This is the crux of what I mean when I say that naturalism (and by extension, atheism) is self-defeating.

              When I say that I have little interest in what Plantinga things, I’m saying that, when the topic is biology, I won’t get a critique from anything less than a biologist.

              Am I confused here? It seems like Plantinga sketches out one aspect of how evolution works: that the justification behind an action is immune to natural selection. To our example, the correct justification of Paul’s action (tigers are dangerous and it sucks to be eaten) is just one of 1000 equally likely justifications for natural selection to select.

              OK, he’s made his claim about evolution. Now I want to see if that claim stands up to scrutiny within biology.

              Once again, we get into the question of whether or not the content and propositions of one’s thoughts fits within the realm of biology. It would be impossible to construct a reasonable argument to support such a thesis. If you think you can do so, by all means please feel free to do so.

              When you make a statement such as “Plantinga sketches out one aspect of how evolution works,” you are clearly implying that the content and propositions of one’s thoughts fall within the realm of biology. What Plantinga is suggesting is that the content and propositions of one’s thoughts are a factor that are NOT influenced by evolutionary considerations. You seem to have missed this.

              What I suspect is going on here is that you participate in the worldview known as “scientism,” which I discuss in my essay titled I Believe in Science: Why Do I Need Religion? As I discuss in this essay, scientism proposes that the only kind of knowledge that we can have is scientific knowledge. The problem with this proposition is that this very proposition can never be verified scientifically.

              An excerpt from that essay:

              How could a statement such as, “The only kind of knowledge that we can have is scientific knowledge,” be verified scientifically? With a chemistry experiment utilizing a bunsen burner and test tubes? With a physics experiment utilizing a particle accelerator? Because the belief that, “The only kind of knowledge that we can have is scientific knowledge” CAN NEVER ITSELF BE SCIENTIFIC KNOWLEDGE, it is a self-refuting belief.

              So you seem to be engaging in the following scientistic (not to be confused with “scientific”) circular argument:

              1) You start from the pre-supposition that human beings are exclusively biological entities.

              2) From this pre-supposition, you reason that all of human experience can be subjected to biological study.

              3) Because (supposedly) all of human experience can be subjected to biological study, you reason right back to the pre-supposition that you started with…human beings are exclusively biological entities.

  3. Scott:

    So you apparently ARE saying that the contents and propositions of one’s thoughts is something that fits within the realm of biology?

    I’m saying that “evolution is limited in way X” is a claim within biology and should be evaluated by biologists.

    What do you suppose biology would have to say about the content of my thought “I prefer living in New York to living in Los Angeles.”

    That you could analyze it biochemically (poorly today, better in the future), but that that’s a clumsy way to do so.

    OK, you have agreed to what Dawkins says about our inability to trust our views of the world.

    We can’t trust it 100%. What our brains tell us about the world is somewhat accurate. Science is a tool that can help us work together to improve the accuracy of our understanding.

    Then what is your basis for trusting the naturalist view of the world (in which you appear to believe) which says that the universe is self-existent and therefore does not require a transcendent cause?

    Suppose we have the hypothesis 1 + 1 = 2. We try it on pebbles and it seems to work in any case. We try it on goats and it still seems to work. Our confidence grows in our hypothesis, but we’re still looking out for the one counterexample that would demolish the claim “1 + 1 = 2 in all cases.”

    That’s how it is with naturalism. Primitive man imagined agency where there was one. Not surprising. A skittish ancestor was likelier to survive than one who thought, “Nah, that’s not a dangerous anim—.“

    We haven’t ruled out the possibility that there is a transcendent cause, just like we haven’t ruled out the possibility that 1 + 1 = 2 could be false in some cases. But we go where the facts lead, and there is no evidence to support a transcendent cause.

    How about any of the beliefs that you or any other naturalist holds about evolution? What would be the basis for trusting any of those beliefs?

    Science is pretty reliable; evolution is the scientific consensus; therefore, evolution is the best working theory we have at the moment and we should proceed accordingly.

    If you think you can do so, by all means please feel free to do so.

    It’s not me who’s making the claim, it’s you (correct me if I’m wrong). You are talking about limitations to natural selection. Heck, maybe you’re right. So let’s ask the guys who can actually evaluate this claim (biologists) rather than two lay knuckleheads like us.

    What Plantinga is suggesting is that the content and propositions of one’s thoughts are a factor that are NOT influenced by evolutionary considerations. You seem to have missed this.

    Uh, no, this is precisely what I’m saying. Plantinga is claiming limitations to how evolution works. That’s not enough for me; I don’t get my biology from philosophers, sorry. But I don’t want to dismiss this claim either, so let’s get an evaluation from biologists.

    The problem with this proposition is that this very proposition is something that science cannot tell us.

    Arg! The dreaded self-refuting statement. The naturalist is hoist by his own petard, eh?

    My own view is that science appears to be the way to truth, so I use that. Doesn’t prove that science doesn’t have blind spots, but since it’s delivering on its claims in spades (unlike fields like, say, theology), I’ll go with that. There’s no absolutism in my thinking; if something is shown to be limited, we accept those limitations and adapt accordingly. When we find limitations to science, great, let’s incorporate that into our worldview. But we adjust our worldview based on evidence, not wishful thinking.

    • I’m saying that “evolution is limited in way X” is a claim within biology and should be evaluated by biologists.

      So the statement that “all of human beliefs are biologically determined” is a biological statement rather than a philosophical statement? Is that correct? If so, how could the validity of such a statement be verified biologically? With an experiment involving a petri dish and a microscope?

      You fail to see the self-defeating nature of your argument. The premise that “all human beliefs have a biological basis” is itself a premise that CAN NEVER HAVE A BIOLOGICAL BASIS. Unless you can provide a scientific basis for your views, your views are necessarily extra-scientific and therefore philosophical/religious. The view that “science will eventually provide an explanation for everything” is not a scientific view because it does not have a scientific basis. It is therefore necessarily an extra-scientific and therefore philosophical/religious belief. Furthermore, it is a philosophical/religious belief that can NEVER be scientifically verified because science cannot verify the presuppositions upon which science is based.

      That you could analyze it [the content an propositions of thougths] biochemically (poorly today, better in the future), but that that’s a clumsy way to do so.

      Bob, you seem to be blind to the problem with the view that human thoughts are nothing but electrical and chemical activity in the brain that are determined biologically to maximize fitness. If this is so, then the belief that human thoughts are nothing but electrical and chemical activity in the brain that are determined biologically in order to maximize fitness is itself nothing but electrical and chemical activity in the brain that was determined biologically in order to maximize fitness.

      We can’t trust it 100%. What our brains tell us about the world is somewhat accurate. Science is a tool that can help us work together to improve the accuracy of our understanding.

      You agree with Dawkins that we can’t trust our beliefs 100%. But what basis have you provided that we can trust our beliefs (such as a belief in naturalism) even 1/2 of 1%? If the content and propositions of a belief cannot be biologically determined, but are just selected from a pot-luck (as Plantinga explains), then how can we trust them even 1/2 of 1%? If the content and propositions of a belief CAN be determined by biological processes, then feel free to cite any biologist as to how this can be so. If this is a question for biology, than please provide a biological explanation. The explanation that “biology will explain it in the future” is not a biological explanation. Rather, it is an extra-scientific and therefore philosophical/religious explanation. And it is a self-defeating philosophical/religious explanation because if human beliefs are nothing but chemical and electrical activity in the brain, then the belief that biology will explain it in the future is nothing but chemical and electrical activity in the brain.

      Suppose we have the hypothesis 1 + 1 = 2. We try it on pebbles and it seems to work in any case. We try it on goats and it still seems to work. Our confidence grows in our hypothesis, but we’re still looking out for the one counterexample that would demolish the claim “1 + 1 = 2 in all cases.”

      That’s how it is with naturalism. Primitive man imagined agency where there was one. Not surprising. A skittish ancestor was likelier to survive than one who thought, “Nah, that’s not a dangerous anim—.“

      We haven’t ruled out the possibility that there is a transcendent cause, just like we haven’t ruled out the possibility that 1 + 1 = 2 could be false in some cases. But we go where the facts lead, and there is no evidence to support a transcendent cause.

      This is a great example of how you conflate science with the PHILOSOPHICAL belief known as naturalism. If naturalism were a scientific rather than philosophical view, you would be able to provide scientific validation for its premises. What scientific validation can you provide for the naturalistic premise that “the universe is self-existent and therefore does not require a transcendent cause”? What scientific validation could you provide for the naturalistic premise that “the mechanisms which drive biological evolution are self-existent and therefore do not require a source or an explanation”? Even the view that “science will one day provide answers to such questions” is itself an extra-scientific and therefore philosophical/religious view.

      You state that “there is no evidence to support a transcendent cause.” And yet, there is VAST evidence to support a transcendent cause. I touch upon this evidence in the following essays:

      1) Is There A God: (What is the chance that our world is the result of chance?)

      2) God is Real: What It All Boils Down To

      3) Why Life Could Not Have Emerged Without God

      Science is pretty reliable; evolution is the scientific consensus; therefore, evolution is the best working theory we have at the moment and we should proceed accordingly.

      Yes, but here, again, you are conflating science with the philosophical belief system of naturalism. If evolution is the best scientific explanation for the diversification of life, then we need an ontological explanation of where the mechanisms that drive evolution came from. I get into detail regarding this topic in the “short take” titled Why Evolution Cannot Be Used to Rationalize Atheism. Explaining the diversification of life from a putative common ancestor through the mechanism of random mutation and natural selection does nothing to provide any final ontological answers. Rather, it just delays the question. We are left with the question of where these mechanisms came from.

      Uh, no, this is precisely what I’m saying. Plantinga is claiming limitations to how evolution works. That’s not enough for me; I don’t get my biology from philosophers, sorry. But I don’t want to dismiss this claim either, so let’s get an evaluation from biologists.

      Once again, you are reasoning from the presupposition that the content and propositions of one’s thoughts are biologically based. But you have provided us with no biological basis for this. Feel free to cite any explanation from any biologist you can find for how the content and proposition of one’s thoughts are biologically determined. You have failed to do this.

      The explanation that “biology will explain it in the future” is not a scientific explanation because this proposition itself does not have a scientific basis. It is therefore an extra-scientific, and therefore philosophical/religious premise.

      Arg! The dreaded self-refuting statement. The naturalist is hoist by his own petard, eh?

      My own view is that science appears to be the way to truth, so I use that. Doesn’t prove that science doesn’t have blind spots, but since it’s delivering on its claims in spades (unlike fields like, say, theology), I’ll go with that. There’s no absolutism in my thinking; if something is shown to be limited, we accept those limitations and adapt accordingly. When we find limitations to science, great, let’s incorporate that into our worldview. But we adjust our worldview based on evidence, not wishful thinking.

      You say that “your own view is that science appears to be the way to truth”. Since you say “the way,” I am going to infer that you mean that the only kind of knowledge we can have is scientific knowledge. What I am looking for is a scientific validation for the premise that “the only kind of knowledge we can have is scientific knowledge.” How could such a premise be verified scientifically? With a chemistry experiment involving a bunsen burner and test tubes? With a biology experiment involving a petri dish and a microscope? Please explain.

      Since you insist that the theistic worldview has no evidence, please respond to the evidence presented in the three essays that I mentioned above (to start).

      Please also detail what “evidence” you used to arrive at your naturalistic worldview. For example, what evidence do you have that the universe emerged during the Big Bang purely as the result of unintelligent natural mechanisms? Can you even describe what these natural mechanisms were that caused the Big Bang to happen, let alone provide evidence for them? How about the emergence of life from non-life? What was the mechanism that allowed that to happen? Can you even cite the mechanism, let alone cite evidence for it? As I detail in Why Life Could Not Have Emerged Without God, the mechanisms cited by various highly prominent atheist biologists include but are not limited to 1) Intervention from space aliens 2) Life came to earth from space without aliens 3) A piggyback ride on crystals.

      • I think we’re going in circles here. You said earlier, “natural selection selects for NP properties that improve an organisms chances of survival, but does not select for the content or propositions of those NP properties.” That’s a claim about natural selection; that’s a claim within the field of biology. Again, I’m asking that we have biologists evaluate that claim.

        What scientific validation can you provide for the naturalistic premise that “the universe is self-existent and therefore does not require a transcendent cause”?

        Haven’t we been over this? I don’t make such a claim. I simply say that the claim that the universe can only be explained with a transcendent cause has insufficient evidence in favor of it; therefore, I must (provisionally) reject it. Show me sufficient evidence, and I’ll change my mind. Until that point, I would be an idiot to believe such a monumental claim with insufficient evidence.

        You delight in trying to show my argument as self-contradicting (it’s not, because I’m not saying what you’re claiming I’m saying) without trying to shore up your own argument. Even if you imagine that I have nothing, what have you got? For me to accept the incredible claim that there was a transcendent cause for the universe, I gotta have that evidence. Simply showing that my argument is in tatters does zilch to support yours.

        Sorry, I haven’t had time to read your long posts. Can you summarize?

        (And just a metacomment: you make yourself look ridiculous when deny evolution. It’s hard to then take anything else you say seriously. I realize that your saying idiotic thing X says nothing about the validity of thing Y that you also say, but help me out here. You’re not making yourself look credible. I’m just sayin’.)

        If evolution is the best scientific explanation for the diversification of life, then we need an ontological explanation of where the mechanisms that drive evolution came from.

        And what out if we agree that God did it? Do we still need an ontological explanation of that??

        “Oh, don’t worry about that; God’s always existed” doesn’t do it for me, I’m afraid.

        Why Evolution Cannot Be Used to Rationalize Atheism.

        And I don’t do so. Destroy the theory of evolution and you’ve done nothing to support the hypothesis that God exists.

        We are left with the question of where these mechanisms came from.

        With evolution as an explanation, you’re left tossing and turning at night, wondering about what underlies that … but with “God dun it!” you’re sleeping like a baby?

        The God explanation is simply a relabeling of the questions at the frontier of science.

        The explanation that “biology will explain it in the future”

        Huh??

        What I am looking for is a scientific validation for the premise that “the only kind of knowledge we can have is scientific knowledge.”

        Drop the self-refuting angle, will you? Doesn’t apply. Getting tedious.

        Say we have the hypothesis that scientific knowledge is reliable. We test that hypothesis, and we adapt accordingly. If science doesn’t give us reliable knowledge, we dial back our confidence. If something besides science gives us reliable knowledge (I don’t know—history, maybe, if the techniques within history wouldn’t be considered scientific), then we broaden the sources of reliable knowledge in our hypothesis.

        We won’t be able to prove this hypothesis. Indeed, we never prove a scientific hypothesis, law, or theory. Everything within science is provisional. We adapt.

        • I think we’re going in circles here. You said earlier, “natural selection selects for NP properties that improve an organisms chances of survival, but does not select for the content or propositions of those NP properties.” That’s a claim about natural selection; that’s a claim within the field of biology. Again, I’m asking that we have biologists evaluate that claim.

          Yes, we are going in circles, Bob, because you are not responding to the points that I am making. You say, “That’s a claim about natural selection; that’s a claim within the field of biology.” But you:

          1) Repeatedly ignore the fact that Plantinga is arguing that the content and propositions of one’s thoughts does NOT fit within the realm of biology.

          2) Repeatedly ask “that we have biologists evaluate that claim” while simultaneously failing to provide the evaluation by a biologist that I repeatedly ask for.

          But since you have not, will not, and CANNOT provide any evaluation by a biologist that establishes how the content and propositions of one’s thoughts could be biologically determined, what choice do we have but to assume that this is a question THAT DOES NOT FALL WITHIN THE REALM OF BIOLOGY?

          Here is the argument, again, for why the content and propositions of one’s thoughts do not fall within the realm of biology. This is the argument that YOU NEED TO RESPOND TO:

          1) If our beliefs are biologically determined by evolution to maximize fitness, then the belief that our beliefs are biologically determined by evolution to maximize fitness is ITSELF a belief that was biologically determined by evolution to maximize fitness. And what evolutionary advantage would such a belief provide? And if beliefs evolve to provide fitness, what basis do we have for assuming that they also provide truth? None whatsoever, because a false belief can provide just as much survival value as a corresponding true belief. For example, the belief that running from a lion will make one rich and famous provides just as much survival value as the belief that running from a lion will protect one from being eaten.

          Further, since science biology is only a few hundred years old, how could any beliefs about biology evolve in only a few hundred years? Did our 18th century ancestor who did not believe that beliefs are biologically determined die off for some reason?

          2) Biology only examines the biological workings of the brain (chemical and electrical activity), not the content of one’s thoughts. If the content of one’s thoughts is nothing but chemical and electrical activity in the brain, then this very belief (that the contents of one’s thoughts is nothing but the chemical and electrical activity in the brain) is nothing but the chemical and electrical activity in a naturalist’s brain. But if thoughts and beliefs ARE more than just the chemical and electrical workings of the brain, then thoughts and beliefs do not fall within the scope of biology. Chemical/electrical activity can neither be true nor false. How would one distinguish between a true and false electrical impulse? By the wattage of that electrical impulse? What chemical property would a true belief have that would distinguish it from a false belief? A higher PH level? The assertion that beliefs fall within the scope of biology is patently absurd.

          These 2 above points are what you need to respond to rather than continuing to assert that the content and propositions of one’s thoughts fall within the scope of biology without providing a rational basis for your assertion. You suggest that it falls within the scope of biology because the view that it does not fall within the scope of biology places a limitation on biology, and only biologists can determine the limitations on biology. But can only dentists determine the limitations of dentistry? Can only doctors determine the limits of medicine? Can only chemists determine the limitations on chemistry? Your reasoning evades me.

          Even if you imagine that I have nothing, what have you got? For me to accept the incredible claim that there was a transcendent cause for the universe, I gotta have that evidence. Simply showing that my argument is in tatters does zilch to support yours.

          Sorry, I haven’t had time to read your long posts. Can you summarize?

          Sorry, but I am going to have to call your bluff. You clearly have the time to invest in writing long replies to my comments, so I have every reason to assume that you have time to read a roughly 2 page essay.

          So lets just go one essay at a time in order to conserve your time. The first essay which provides compelling evidence for theism is titled Is There A God (What Is the Chance That Our World Is the Result of Chance)? I am going to assert that if, as this essay demonstrates, the majority of astronomers/astrophysicists have come to theistic/deistic conclusions, then there is sufficient evidence that you need to respond to with a point-by-point rebuttal, rather than a sweeping and facile dismissal as “insufficient evidence.” Is this too bold of a claim?

          (And just a metacomment: you make yourself look ridiculous when deny evolution. It’s hard to then take anything else you say seriously. I realize that your saying idiotic thing X says nothing about the validity of thing Y that you also say, but help me out here. You’re not making yourself look credible. I’m just sayin’.)

          Except that I never have denied evolution. That is something that you made up. If I am wrong, then show me where I did so. You can’t and you won’t because I never did. Period.

          What other conclusion should we derive from your false allegation other than the conclusion that you are trying to distract attention from the bankruptcy of your arguments?

          “Oh, don’t worry about that; God’s always existed” doesn’t do it for me, I’m afraid.

          Then please provide your own counter ontological explanation.

          Since time, like matter and energy, began with the Big Bang, whatever brought the universe into existence clearly exists outside of time.

          And I don’t do so. Destroy the theory of evolution and you’ve done nothing to support the hypothesis that God exists.

          Who ever said anything about destroying the theory of evolution? You clearly did not read the post titled Why Evolution Cannot Be Used to Rationalize Atheism in the “short takes” section. Don’t worry, it is not a very long essay. I am not in any way trying to destroy the theory of evolution. Rather, I am demonstrating that the theory of evolution, whatever truth it may hold, cannot be used to rationalize atheism. And as the essay demonstrates, Charles Darwin agreed with me.

          With evolution as an explanation, you’re left tossing and turning at night, wondering about what underlies that … but with “God dun it!” you’re sleeping like a baby?

          The God explanation is simply a relabeling of the questions at the frontier of science.

          Here, again, you 1) conflate science with the philosophical/religious belief system of naturalism and 2) you confuse science with ontology. “The God explanation” is an ontological explanation, not a scientific explanation. Ontological explanations underlie scientific reasoning, they do not compete with scientific reasoning. For example, no matter what natural mechanisms are discovered by science in the future, the ontological question of where these mechanisms came from will always be there. Please provide a counter ontological explanation of where natural mechanisms come from.

          Drop the self-refuting angle, will you? Doesn’t apply. Getting tedious.

          Say we have the hypothesis that scientific knowledge is reliable. We test that hypothesis, and we adapt accordingly. If science doesn’t give us reliable knowledge, we dial back our confidence. If something besides science gives us reliable knowledge (I don’t know—history, maybe, if the techniques within history wouldn’t be considered scientific), then we broaden the sources of reliable knowledge in our hypothesis.

          We won’t be able to prove this hypothesis. Indeed, we never prove a scientific hypothesis, law, or theory. Everything within science is provisional. We adapt.

          I’m afraid I cannot drop the self-refuting angle. Your statement that it “doesn’t apply” needs some sort of rational validation. Whether or not it it tedious is an entirely different matter from whether or not it is true.

          You write “Say we have the hypothesis that scientific knowledge is reliable. We test that hypothesis, and we adapt accordingly.” But what we are looking for here is an explanation of HOW such an hypothesis could ever be tested scientifically. Could you test the hypothesis “scientific knowledge is reliable” with the above mentioned chemistry experiment utilizing a bunsen burner and test tubes? Or would it be a biology experiment involving a petri dish and a microscope? Some other scientific experiment that I have overlooked?

          You have to re-evaluate whether the premise that “scientific knowledge is reliable” is a scientific or philosophical premise.

  4. I’ve gently and repeatedly pointed out that your claim “natural selection selects for NP properties that improve an organisms chances of survival, but does not select for the content or propositions of those NP properties” is a biological claim. I guess it’s inconvenient for you to respond to this, so you focus your attention elsewhere and argue that, you don’t want to follow this line of reasoning anymore and some other question is the actual focus.

    If failing to play your game means that you win, so be it. Declare victory–I give up.

    I never have denied evolution.

    Terrific!

    you are trying to distract attention from the bankruptcy of your arguments?

    Yeah, that must be it. Ouch!

    You clearly did not read the post titled Why Evolution Cannot Be Used to Rationalize Atheism

    And I clearly made that clear.

    Charles Darwin agreed with me.

    That’s nice, but no one cares what Darwin said.

    What is your scientific evidence for the belief that “the only kind of knowledge we can have is scientific knowledge”?

    Have none. And that doesn’t bother me as I explained before.

    • I’ve gently and repeatedly pointed out that your claim “natural selection selects for NP properties that improve an organisms chances of survival, but does not select for the content or propositions of those NP properties” is a biological claim. I guess it’s inconvenient for you to respond to this, so you focus your attention elsewhere and argue that, you don’t want to follow this line of reasoning anymore and some other question is the actual focus.

      No, I have responded to this repeatedly. What basis do you have for the assertion that it is a biological claim? You seem to take this as a given that does not need defense. But assuming that the content and propositions of a thought are biological is an enormous leap of faith that has no empirical support whatsoever. I asked you before and I will ask you again: If the content and propositions of a thought are biologically determined, how could you distinguish a true belief from a false belief? Would a true belief have a higher voltage during the firing of the synapses in the brain? Would the brain chemistry of a true belief be different from the brain chemistry of a false belief?

      Please, please explain how biology could be applied to distinguish between a true and false belief. You have not done this, but rather have continued to merely assert your view that the content and propositions of a belief are biologically determined. You must present an argument, not merely repeat an assertion.

      That’s nice, but no one cares what Darwin said.

      Now that is a first. I have never heard an atheist say such a thing.

  5. Barry Whyte says:

    This guy is such a joke -he is typical of a closed minded Creationist…he has been brainwashed. If he lived 10,000 years ago (before God created the world haha) he wouldn’t believe any of what he is saying.He hasn’t presented 1 argument that i have thought good point and i came here to find one.

    • I am going to say this as gently as possible, but I will have to be blunt when bluntness is necessary:

      Sir, you have to decide if you want to respond with rationally constructed arguments, or just with strident rhetoric (which is all that you have done so far…”joke,” “brainwashed”). Secondly, you need to decide if you want to respond to the ACTUAL ARGUMENTS, presented by Christians such as myself, or take the easy road out and just react to your crude caricatures of Christian arguments.

      For example, regarding your “10,000 years ago” comment, the bible and science are remarkably similar in their accounts of the age of the earth. This is demonstrated in my essay titled Doesn’t Evolution Prove the Biblical Account of Creation to Be False. In this essay, I cite (and link to a video by) Gerald Schroeder, who has the unique qualifications of being both a physicist (formerly on staff at MIT) and a biblical scholar.

      (Click here) to read an article by Schroeder about this remarkable agreement between the bible and science about the age of the earth.

      In my experience with this website, it has been my observation that doctrinaire atheists will not reply to the arguments presented with actual logically constructed rebuttals. Rather, they will just fire off salvo after salvo of forceful and repetitious rhetoric. If you would like to be an exception to this rule, I would be delighted.

      I must point out that rhetoric and caricatures are what one falls back on when one is not able to present a rationally constructed, fact based defense. Any reasonable third party viewer will be able to see that what you have presented so far amounts to nothing more than an evasion of the arguments, as opposed to a logical reply to the arguments.

      This, I will assert, is because you can’t respond to the arguments made in this essay, because the atheist position is indefensible. What are your specific, point-by-point, logically constructed replies to the points made in this essay?

      I eagerly await your reply to the points made in this essay, and if you are so inclined, to the points in the other essay that I have mentioned above. I will warn that the Gerald Schroeder video will present a fairly significant time commitment, but nothing would please me more than an atheist who is actually willing to logically engage with the material rather than merely evade it.

  6. Justin says:

    I must say this is bullshit. I’m an atheist, and I can confirm that in general, creationists and fundies (not all Christians, mostly fundies) will not respond to or engage in any real logic. If you carefully construct arguments, they will often either evade them, or counter them with bullshit. There are 7 billion people in this world, and there are no unique thoughts or conversations. All arguments are bound to be repeated, and atheists just can’t get through to creationists because they are unwilling to use logic or reason. For this reason, any time we see classic bullshit arguments, that’s what we call “brainwashed” because we know them to be true and know that refuting them logically will get us nowhere. I’m not here to actually respond to the content of the post though, so I hope you can be mature and understand that I’m not “evading” your points. It’s simply 3 am and I have no intention of debating right now about anything. I just found it necessary to shoot down your militant holier-than-thou attitude. You do seem to be much more logic oriented than most who I come accross, so I hope you do find stimulating discussion with rational atheists in the future, as it’s almost an atheist doctrine to discuss arguments in detail rather than rhetoric. I believe you can find my email address via this comment if you feel the need to discuss anything further, or if you are interested in debating over the actual content of your post. Cheers!

    • Justin:

      I would love to engage in a rational discussion with you about this topic. However, so far you haven’t done anything more than spew out forceful rhetoric (“bullshit”, “brainwashed”, “militant holier-than-thou” etc).

      I must remind you that all rational third party observers of any debate will recognize that the use of forceful or strident rhetoric amounts to a tacit acknowledgement that one’s stance cannot withstand logical scrutiny. In other words, logical arguments that can stand up on their own do not need the support of harsh words (“bullshit”, “brainwashed”, “militant holier-than-thou”, etc.). Such harsh words are what one falls back upon when one’s arguments have fallen apart under logical scrutiny. Any time that you engage in such rhetoric, you will be tipping off third party observers of our debate that you have realized that your stance has not withstood the test of reason.

      For example, just think about it…did Einstein need to use forceful rhetoric to support his theories, or did he support his theories with logic and facts? Did he say something to the effect of “time and space can be warped,” and then support this point with logic…..or did he say “time and space can be warped, and if you don’t agree with me you are a brainwashed idiot”?

      The same goes for mudslinging generalizations about Christians: (“Christians will evade arguments or counter them with bullshit.”) Such generalizations about Christians are another way to distract attention from the inadequacy of your logical arguments…and will tip off third party observers of our debate as such. You must reply to the arguments made by this Christian rather than by making vague references to arguments made by someone else.

      I eagerly await any logically constructed, fact-based, point-by-point rebuttals that you have to the argument presented in this essay (or any of the other essays).

  7. Rubina says:

    Justin: That was clearly abusive ad hominem………

  8. Australian Atheist says:

    Scott, I am an atheist,a not very good one at that, TBH I think I just like arguing with people. You have honestly got me thinking. I don’t understand much of what the comments were, however it is clear that you destroyed all your ‘opponents’ them with actual logic. I am amazed at the few that had a cheap shot at you and as you said they probably won’t reply because it is all just rhetoric mumbo jumbo.

    Just thought I would say, you have me thinking really hard. I have a Christian enemy (really I like the guy) and I think I might just explore it further.

    For the anti creation camp, what a dear shame you couldn’t have the courtesy to reply (bar bob, who I just wish would have answered some of Scott’s questions)

    Atheist (maybe new agnostic) but still searching.

    Troy (Australia)

  9. Beachbum says:

    This is silly sophism. Fact: we don’t take our reasoning at face value because it does fail us continually. Our brain is evolved to keep us replicating our genetic make up whether by hook or by crook. And our brain can’t be trusted to reason intuitively about many things because of this ability of our brain to manipulate us emotionally. A powerful example of which would be an emotional attachment to the supernatural facilitation of an afterlife. A deity, of all things. Why else would primitive intellects, which can barely think at all, can easily imagine gods exists?

    A deity would make our intuition a viable source for knowledge, but nothing could be further from the truth. You, Alvin Plantinga, et al. have just destroyed theism: by admitting that intuition is a weak basis for beliefs, the god concept and creationism are finished. You have eliminated thought as a means for knowing some god. To reiterate: Our inability to intuit reality eliminates both the religionists’ and creationists’ claim that our brain is the product of creation facilitating belief in some deity. End of argument. Full stop.

    Science knows this due to its roots in, and violent escape from, what was a platonic view of reality proffered by religion. It was called the Enlightenment when men like David Hume said that a wise man proportions his beliefs to the evidence for a reason. The platonic nature of thought prior to Hume’s era retarded science for a thousand years. That is: we tried it your way. It fails miserably.

    It is the methodology of science that separates knowledge from belief and opinion. It separates emotionally held beliefs from facts. For opinions, hypotheses, etc. to attain the level of information, of knowledge, they need to be substantiated by evidence, peer reviews of interpretations of that evidence (as rendered by objective observation, experimentation, but by no means contemplation), and this means those opinions need to be falsifiable in the first place. Which means your feelings, your intuitions, don’t count as knowledge. This is why science has been so successful.

    And one more point: this is actually an old argument once proffered by C.S. Lewis, and it didn’t stand up to scrutiny then. Wow! The desperation speaks volumes.

    Tell Plantinga, and Strobel, thanks for eliminating theism though. Not that it needed another nail in its coffin.

    • All that you have done here is make some assertions. You have not made a case for how humans could have access to truth if naturalism is true. Please state clearly how mindless random processes could have granted humans access to truth considering that natural selection selects for survivability, and not for anything else…such as truth. Further, the forcefulness with which you make some of your assertions (“silly sophism”, “the desperation speaks volumes”) should be considered a flag. A person with a rationally sound argument does not need to resort to such forceful rhetoric. Indeed, forceful rhetoric is a crutch used to support an argument that cannot stand on its own with logic. Each and every time you use forceful rhetoric in lieu of logical reasoning, you advertise the inadequacy of your argument to third party viewers of this discussion.

      Belief in God is not based upon mere intuition. Rather, it is based in solid logic. For just a taste of this logic, please read The Ultimate Cart Before the Horse (Why Atheism is Illogical) and Is There A God? What is the chance that our world is the result of chance? (and the closely related post titled OK…I want numbers. What is the probability that our universe is the result of chance?). Please also read Why Life Could Not Have Emerged Without God and the related post titled Why Trying to Explain Away God With Science is an Error.

      It is ironic that you have stated that “the methodology of science…separates emotionally held beliefs from facts.” Considering the emotional outpouring contained in your strident rhetoric, and your failure to demonstrate how humans could have access to truth assuming the veracity of naturalism, why should we consider your atheism to be anything other than an emotionally based belief?

      Upon what emotions do I suspect your atheism is based? Please read If the Evidence for God Is So Strong, Why Are So Many Smart People Unconvinced? In a nutshell, it is the repugnance that atheists feel towards the concept of being subject to a higher authority which serves as the emotional motivation for atheism.

      Next, I have to mention that whenever you imply that science and God are competing explanations, you are committing what is known in philosophy as a category error. I delve into this topic in Why Trying to Explain Away God With Science is an ERROR. In a nutshell, science does not explain anything. Rather, it only describes natural phenomena and provides no explanation.

      Lastly, you have not replied to the last part of THIS essay, in which I detail how Christianity was responsible for the rise of science. To this end, please review my citation of Stanley Jaki towards the end of the essay.

    • Graceus says:

      Beachbum,

      You write: “And our brain can’t be trusted to reason intuitively about many things because of this ability of our brain to manipulate us emotionally. A powerful example of which would be an emotional attachment to the supernatural facilitation of an afterlife. A deity, of all things. Why else would primitive intellects, which can barely think at all, can easily imagine gods exists?”

      What does the statement have to do with the truth of whether there is an afterlife or gods? You have shown us that natural selection would select for survival benefit-perhaps to alleviate emotional tension that would mentally harm the brain, but then could it not be said that atheists also decide not to believe in a God in order to relieve cognitive dissonance as well in order to aid in survival? So if both atheist and theist want to relieve cognitive dissonance by choosing their belief/unbelief, then which is true? They both cannot be right. So, natural selection does not select for _truth_ value – only survival value, and relieving cognitive dissonance would be a survival technique.

      I’m not sure if you read the essay when you write “You, Alvin Plantinga, et al. have just destroyed theism: by admitting that intuition is a weak basis for beliefs.” Actually, Plantinga is arguing that under naturalism, we have no reason to trust our reasoning. Scott has already given you a couple of essays that show that belief in God is not based upon mere intuition, and he has shown how the relationship between natural selection, intuition, and truth are problematic by Darwin, Churchland, and Nagel. If you and Bob think otherwise, the burden of proof is upon you to produce the justification for any counter claims that you have.

    • Jkwo says:

      I don’t understand, BeachBum. You state that the fact that the tendency for our fluctuating emotional states and other human attributes to compromise our ability to reason is highly damaging to the case for a divinely inspired intuition and therefore to the case for the Christian God. Basically, unless we are innately programmed to make maximally beneficial decisions for ourselves at every juncture (unless we are robots), then obviously the Christian God does not exist. Now I hate to use the Bible to argue with atheists, because I know how the response typically goes. However, I think it’s fair game here, since you’ve brought this debate into the territory of doctrine (by referencing the Christian account of the origins of human intuition). This is an incredibly basic topic, doctrinally speaking. If someone were to compile even a one-page summary of the Bible, you can guarantee that somewhere in there would be the notion that we were bestowed with free will by God, which includes the ability to make poor decisions for ourselves and others. No one has ever claimed that we were granted the omniscience of God or that human follies couldn’t corrupt our judgement. This argument is a non-starter.

  10. Graceus says:

    Don’t mention it ;)

  11. Troy says:

    Scott,
    Could you provide any of your scientific peer reviewed articles.

    • This is not a website for performing scientific inquiry. You would need to go to a biology, physics, or chemistry (etc.) website for that. Rather this is a website that discusses the ontological question of God’s existence.

  12. Mitch Buck says:

    Sorry if this doesn’t add much to elevate the discussion, but I’m going to say it anyway: Scott clearly dismantled Bob’s arguments–nicely done. (This is coming from a doubting atheist by the way.) I cannot stand it when atheists presume that they’re the rational ones when, in fact, it’s clear to onlookers that they’re clueless. The smug, self-congratulatory attitude New Atheists have about their own “reasoning skills” is excruciatingly ironic.

    Plantinga’s argument is obscure sounding initially, but the more thought I’ve given it, the more troubling it is… Piling determinism onto it is even more of a challenge for the naturalist. An intellectual pursuit like physics or pure mathematics, where getting at the truth of things is the highest priority, just doesn’t seem to fit well within that sort of worldview; scientific laws or mathematical theorems did not arise out of an agent’s insight or creativity–no, the thoughts of these brilliant minds were determined by chemical/electrical events in the brain and were inevitable.

    Naturalism has some major holes in it, that’s for sure. But I cannot be held accountable for this opinion since the opinion itself was determined by prior chemical/electrical causes in my brain…

  13. Gerry says:

    Hi Scott. I hope you don’t mind me jumping in on this conversation you have had with Bob. I am a theist, I have read ‘Naturalism Defeated, and I tend to accept Plantinga’s argument. But still if I look at it from the point of view of an atheist naturalist, I can see something that is troubling and I wonder if you could help me clear that up.

    Let’s say Paul may run away from the tiger prompted by 10 possible beliefs and only 2 of those are ‘correct’, that is, only 2 are logical and reasonable (‘I do not want to feel pain’ or ‘I do not want to get killed’). Now, according to your argument (and Plantinga’s) the only thing that really matters here and the only thing that can be selected for is whether or not Paul is the kind of person who actually runs away from the tiger. The reason that prompts him to run away is irrelevant and cannot be selected for. That makes perfect sense to me. But then what do we do with the following reasoning.

    Let’s say it were possible for us to quiz every adult on earth. The quiz would consis of 1 question:”If you were confronted with a tiger, why would you run?”. And there would be 10 ‘multiple choice’ answers. 8 would be nonsensical, and 2 would be logical/reasonable. I think we should agree that upwards of 98% of these adults would pick 1 of the 2 ‘correct’ answers. Since we, more often than not, have the ability to pick the ‘correct’ answers, why shouldn’t we then assume that our ancestors had the abililty to pick the correct answers when confronted with survival situations? And why wouldn’t his ability to see the ‘correct’ answer then account for the survival of our ancestors and the survival of ourselves? This all seems to imply that seeing the correct answers (that is, reasoning) is something that can be selected for. And this seems to directly contradict the argument against naturalism. It just seems to be a valid assumption that our ancestors would be amoung those who could better reason how to survive. So we inherited an enhanced ability to reason (i.e. discern what is true).

    Part of me says there must be a flaw in there somewhere, but I am unable to see what it is.

    Your thoughts?

    • Gerry:

      Thanks for your question.

      Glancing at your proposed argument, here is a flaw that I see:

      You write, “Since we, more often than not, have the ability to pick the ‘correct’ answers, why shouldn’t we assume that our ancestors had the ability to pick the correct answers when confronted with survival situations?”

      Neither I nor Plantinga is suggesting that our ancestors did not have the ability to pick the correct answers. The problem with the naturalist worldview (in which atheism is rooted) is that it does not explain WHY our ancestors had this ability. There is no survival value in truth. Rather, there is only survival value in neuro-physiological (NP) responses…such as running and hiding from a predator.

      As Plantinga points out, our ancestors would have to select their beliefs associated with NP responses from a “pot luck.” For example, one could believe that eating a poisonous plant is a bad idea because it will put poison in one’s body, or, conversely, one could believe that eating a poisonous plant is a bad idea because it will turn one into a werewolf. Both beliefs provide the same survival value, because they both produce the same NP response…namely, not eating the poisonous plant. So natural selection cannot be cited as being responsible for our ability to discern truth from falsehood.

      And I will take Plantinga’s argument a step further: The naturalist worldview does not even explain why there should be any beliefs whatsoever associated with NP responses (either true OR false). Survival only requires NP responses, not beliefs associated with those responses. Put another way, if the fundamental composition of reality (a.k.a “ultimate reality”) is one of mindless matter (as naturalism/materialism suggests), why would an organism not just behave like a mindless robot with no beliefs (or thoughts of any kind) whatsoever?

      The materialist/naturalist worldview cannot coherently explain the origin of consciousness from non-conscious matter. Please read my essay titled The Ultimate Cart Before the Horse (Why Atheism is Illogical) to explore this topic further.

      Scott

      • There is no survival value in truth.

        A belief being true makes it likely to be selected for by natural selection.

        • Scott Youngren says:

          Bob,

          How is it that survival value and truth have anything to do with one another? The following two beliefs provide the same survival value because they both would prevent a person from eating a poisonous plant:

          1) “I should not eat this plant because it is poisonous and could kill me.”

          2) “I should not eat this plant because it will turn me into a vampire.”

          One of the above beliefs would be true with regards to a poisonous plant, but the other is false. And yet, both beliefs provide the same survival value.

          Scott

          • The following two beliefs provide the same survival value

            Right. And are they equally likely to occur to one? “This is bad to eat because it’s poisonous (like so many other things I’ve seen)” seems like a reasonable thought, while “This is bad to eat because the Purple People of Pluto would be offended” (and a billion other ridiculous, unrealistic, not-grounded-in-reality ideas) isn’t.

            You seem to imagine that a naturalist is obliged to see all possible beliefs just floating around out there, sticking to us like thetans without regard for whether they’re true. But reality gives us feedback.

            • Scott Youngren says:

              Bob,

              Yes, “It’s bad to eat this plant because it is poisonous” seems like a reasonable thought. And your “Purple People of Pluto” example does not seen reasonable.

              But you are just assuming the reliability of human reason. If atheism is true, and evolution works to promote survival value, then we have no way to trust that our beliefs are true. This is because both of the beliefs you cited above provide the same survival value as long as they both provide the same result of preventing a person from eating a poisonous plant.

              PLEASE EXPLAIN WHY WE SHOULD TRUST ANY OF OUR BELIEFS ASSUMING THAT OUR BELIEFS EVOLVED TO PROMOTE SURVIVAL, RATHER THAN JUST ASSUMING THAT HUMAN REASONING CAN LEAD TO TRUTH.

              You say that “reality gives us feedback,” but if we are just “survival machines – robot vehicles blindly programmed to preserve the selfish molecules known as genes” (as the famous atheist biologist Richard Dawkins put it), then why should we assume that this feedback can lead us to beliefs that are true (such as a belief in the naturalist worldview in which atheism is grounded)??? Why does a “survival machine” or a “robot vehicle” need truth to survive?? A survival machine only needs to survive to pass on its genes, and this has nothing whatsoever to do with truth.

              This is yet another example of how the just-so storytelling that is so very characteristic of the atheist worldview. Atheism must assume that we can trust that our beliefs can lead us to truth because we just can.

              Theism provides an objective standard of truth to reason towards (God’s truth), but atheism can only reason toward subjective human truth.

              • But you are just assuming the reliability of human reason

                Nope. I’m assuming the effectiveness of tough-love feedback from reality. If I believe that banging my head against a tree will relieve thirst, I’m too dumb to live.

                • Scott Youngren says:

                  Bob,

                  You just aren’t getting this. I’m not sure how your “tough love from reality” comments relate to this discussion. Your banging-your-head-against-a-tree-to-relieve-thirst example is a belief that is both false and does not provide survival value. But there are plenty of false beliefs that DO provide survival value…such as the belief that one should not eat a poisonous plant because it will turn one into a vampire, etc.. In fact, there are many more false beliefs that can lead to survival than true beliefs. How can I be so sure? Because, for a given context, there is only one true belief, but many false beliefs. If you are twenty five years of age, then the only true belief regarding the question of how old you are is that you are twenty five years of age. But in this same context, there would be many false beliefs about your age…5, 75, 28, etc…

                  I am looking for an explanation as to how natural selection (which is based on survival value, not truth) can lead us to evolve beliefs which are true. It is understandable how natural selection can cause us to evolve beliefs which provide survival value, but it is not clear how natural selection can cause us to evolve beliefs which lead to truth.

                  Again, the belief that one should not eat a certain plant because it will turn one into a vampire provides the same survival value as the belief that one should not eat that plant because it is poisonous. False beliefs and true beliefs can both provide survival value. Survival value and truth have nothing to do with one another.

                  Please answer the question.

  14. G says:

    Response to: “Evolutionary naturalism implies that we should not take any of our convictions seriously, including the scientific world picture on which evolutionary naturalism itself depends… For example, the belief that eating a particular plant should be avoided because doing so would cause one to turn into a werewolf provides just as much survival value as the belief that eating that plant should be avoided because doing so puts poison into one’s body.”

    No, the idea that seeking truth through reason has the same amount of survival as belief in the supernatural is false. If one is able to use reason and figures out that certain plant (plant 1) has poison, one can figure out a way to test new plants (plant 2 etc.: those one has not eaten before) for poison. This is an obvious advantage of reason. One can apply it to many (especially new) things. However, if the same person were to only believe that plant 1 is bad because it can make someone a werewolf, why would someone test plants 2 etc. for poison? The person would not even know about the poison to test. Not testing plants for poison is obviously disadvantageous to survival, especially if one were to migrate. Reason is useful because it leads to predictive validity.

    If reason is such a poor way to understand the world, then why is the world dominated by human life?

    • G says:

      Correction: the same amount survival benefit or survivability

    • You clearly have not read the essay thoroughly. The point of the essay is not that “reason is…a poor way to understand the world,” as you put it. If you think that was the point of the essay, then you must have only given it a cursory skimming.

      The point of the essay is that, assuming the naturalist/materialist worldview is true, we have no reason for trusting the accuracy of our beliefs…and therefore no reason to think that we can reason accurately. Further, without any reason for trusting the accuracy of our beliefs, we cannot judge any of our beliefs to be true…such as a belief in naturalism/materialism.

      • Scott I wonder if you would accept ‘results’ as evidence of proper reasoning.

        In other words if I said

        Plants seem to grow from seeds.
        I believe that if we take these seeds from inside the apple and plant them then we will grow a tree that produces apples.”

        If we plant the seeds and grow a tree that produces apples would you say that I reasoned correctly?

        • Scott Youngren says:

          Dennis,

          That sounds like a reasonable observation to me. But you agreed with the atheist Jane’s statement that, “There’s no way to verify that what we experience is genuine, or relevant.” Therefore, assuming the truth of your atheist worldview, we have no way to verify that your “experience is genuine or relevant” (as Jane put it) when you experience the seeds from an apple produce a tree that produces apples.

          Under your atheist worldview, even your own reasoning cannot be determined to be “genuine or relevant,” because atheism provides no objective standard of truth to reason towards.

          Scott

  15. Zia says:

    What people don’t understand is that the EAAN is not a critique of evolution. It’s a critique of evolution in conjuction with metaphysical naturalism. In the wild, actions are what ultimately make the difference, not beliefs. Scott, have you read Plantinga’s recent paper on the issue or are you just going by what was in his book? Regardless, here’s a link for interested readers.

    http://www.andrewmbailey.com/ap/Content_Natural_Selection.pdf

  16. […] revelation.  (Those wishing to argue that moral beliefs evolved are directed to my essay titled Why Atheism Is Self-Defeating, where this argument is […]

  17. […] guides our beliefs, then beliefs evolve to provide survival value, NOT truth. (Please read Why Atheism is Self-Defeating to explore this subject in more […]

  18. Jane says:

    Instead of a straightforward rebuttal (discussing axioms, and how unlikely it is an internally-consistent system would result from randomness), I’d like to instead make a similar case against theism. That is, to show theism itself is similarly self-defeating.

    Suppose God is not truthful. Then God, as our creator, has no reason to make sure what we experience is in any way valid or truthful. Indeed, it’s easy for such a God to design our minds and the world such that, we’d think he would need to be good, or truthful, without him being so.
    If God is not truthful, all of religion falls flat. While you could go so far as to say God exists, you can’t trust anything Jesus said (if he is God’s son/emissary), or indeed what God has said. Neither can you trust what you experience of the world, as it too was created by a deceitful deity.
    The problem is, from a perspective within the universe, you have no way to tell whether God is truthful, or dishonest. Both resulting universes could well be identical: and if you think they’re not, then who’s to say that’s not the result of a lying God’s design of the mind?

    What you post is not a problem with atheism, it’s a problem with human thought. There’s no way to verify that what we experience is genuine, or relevant: it is, however, necessary to assume it. It might be true, it might not be true, but this is the time necessity wins out.
    Perhaps reason can’t lead to truth, and our justification for thinking so (repeated verification) is somehow flawed. Perhaps God cannot lead to truth as he refuses to share it. Either’s possible.

    Regardless, you can’t use your post as a case against atheism, when theism is just as much a victim. Not even faith gets you out of it: faith too could be the result of an untruthful God.

    • Scott Youngren says:

      Jane, your above comments highlight the self-defeating incoherence in which atheism is caught. Atheist reasoning is every bit as meaningless as atheists insist that the universe is. In his book Illogical Atheism, Bo Jinn incisively lays down the failure of atheism to provide an epistemological grounding for reason:

      “Scientific facts cannot justify reason. It is reason [that] justifies science. But, then, what justifies reason? The reliability of reason, just as the existence of morality and beauty is simply taken for granted by the atheist on purely pragmatic grounds. There is no sufficient ontic referent for their actual existence. Truth/value judgments can be supported by empirical facts, but at the very last instance they will always require a judgment from a personal agent. And unless that judgment is made on the basis of an objective standard of truth, then the judgment is therefore meaningless.”

      “…As we speak, there are atheists the world over insisting that atheism is a conclusion which intelligent people come to on the basis of reason. But, if atheism is true, then human reasoning has no validity at all, because valid reasoning implies a standard of truth that can be reasoned toward and a sufficient reason for believing that human reasoning works in the first place.”

      “…Theism reasons to and from an objective standard of ultimate truth grounded in an absolute mind (God) which gives validity to rational beliefs, and atheism reasons to and from a completely subjective standard that cannot give validity to any belief (ourselves). We cannot reason to the conclusion that our reasoning is valid, since it is as circular as the proposition B → B”

      Jane, theism provides a grounding for truth and reason: the mind of a TRUTHFUL God. Atheism provides no such grounding.…no such objective standard of truth. Therefore, your suggestion that theism is “just as much a victim” of this “problem with human thought” falls flat. What objective standard of truth does atheism provide? None.

      Just think about it…if this “problem with human thought” (which you allude to) is true, then the words which you have typed in your comment have no meaning. I hate to have to use your own words against you, but if your reasoning is correct then the words you have typed to me “might be true” or “might not be true.” If (to use your words), “reason can’t lead to truth, and our justification for thinking…is somehow flawed,” then the reasoning behind THE ARGUMENTS WHICH YOU HAVE TYPED IN YOUR COMMENTS also can’t lead to truth, and your justification for thinking them are also flawed. This is the self-defeating incoherence in which atheism is caught.

      • Jane says:

        The standard of truth of atheism is the universe. Clearly, any statement about it is true or false: the only assumption made is that we experience the universe, or that our minds are made to think truthfully about it. Your post says this.
        This is a perfectly objective grounding, so long as it’s relevant: that is, that we genuinely experience it. In precisely the same way, a truthful God is a fair enough objective grounding: so long as it, too, is relevant. Until you can demonstrate that God is truthful however, your justification means nothing.
        It’s exactly the same argument you make. the arguments just typed in your comment, by your logic, mean nothing until you can demonstrate a truthful God. And, of course, you can’t do that because without a truthful God you have no way to justify anything.

        This isn’t a problem with atheism: it’s a problem with anything. We assume our experiences mean something by necessity: and they’ve clearly served us well.

        Regardless, don’t take aim at me. Theism provides no grounding for truth and reason because you cannot arbitrarily state that there is a truthful God, and expect it to be true. if you can do that, why can an atheist not simply state the laws of logic etc are true?
        All thought is subject to the incoherence you refer to. It’s called axiom: every field of study requires them, things taken as given so that what we discuss has meaning.

        Wrapping up, all I’m saying is that your post is a problem with all human though. Sure, maybe this argument has no meaning: but then neither does yours. That’s the purpose of an axiom.
        If you have a problem with this, then I’m more than happy to discuss it, if and only if you tell me how you know God is truthful, without presupposing that God has designed things to give a truthful impression of him.
        Until you do this, your arguments are (by your own logic) meaningless.

      • From Mr. God_evidence: “Jane, theism provides a grounding for truth and reason: the mind of a TRUTHFUL God.”

        This is pure unsubstantiated opinion.

        And here Jane gets it right: “There’s no way to verify that what we experience is genuine, or relevant:”

        Probably why you have yet to show me that we aren’t simply brains in vats.

        Cheers!

        • Scott Youngren says:

          Dennis,

          You say that Jane is right when she claims, ““There’s no way to verify that what we experience is genuine, or relevant.” Dennis, can THAT be verified to be genuine or relevant?!

          This is the self-defeating incoherence in which atheism is caught. If “we cannot know whether or not our reasoning ability leads us to genuine or relevant truth,” then we cannot know if this very statement is genuine or relevant.

          You suggest that my point about God providing an objective standard of truth to reason towards is “pure unsubstantiated opinion.” But the problem is that the atheist worldview provides no objective standard of truth whatsoever to reason towards. Therefore, on the atheist view, ALL statements are “pure unsubstantiated opinion.” This must therefore also include your very statement that my views are “pure unsubstantiated opinion.”

          Put another way, without an objective standard of truth to reason towards, there is no objective truth…just subjective opinions. Therefore, your view that Jane’s statement is right is just a subjective opinion without any objective truth.

          Do I know if we aren’t just brains in vats? No, I don’t. But you are confusing two entirely different concepts:

          1) An ontological question about the nature of physical reality (if we are brains in vats, or if there exists an outside physical world).
          2) The ability of our reason to lead us to truth.

          Put another way, we could be brains in vats with reasoning ability that leads us to truth.

          Scott

          • on the atheist view, ALL statements are “pure unsubstantiated opinion.”

            Except those that have evidence pointing to their being right. They’re not unsubstantiated anymore.

            without an objective standard of truth to reason towards, there is no objective truth

            I’ve never seen anyone do more than handwave that there are objective truths for difficult things like morality. (Simple truths like “cyanide will kill you” or “1 + 1 = 2” I’ll grant as objective.)

            But perhaps this is a tangent.

            • Scott Youngren says:

              Bob,

              How about the following beliefs…are they objectively true, or just subjective human opinion?:

              1) It is morally wrong to kill kindergarten children with an assault rifle (as in the Sandy Hook massacre).

              2) It is morally wrong to try to exterminate entire ethnic groups (as the Nazi’s tried to do).

              What do you think, are these statements objectively true, or were the Sandy Hook killer and the Nazis merely violating subjective cultural norms?

              Please note that the nazis did not find anything objectively morally wrong with trying to exterminate entire ethnic groups.

              The Nazis killed anyone and everyone who they did not feel was worthy to pass on their genes. It seemed to them perfectly justifiable to kill any “survival machines” with what they perceived to be “undesirable” genes. As historian Richard Weikart points out in his book From Darwin to Hitler, the racist Nazi rationalization for killing comes straight from Darwin. In The Descent of Man, Charles Darwin writes:

              “With savages, the weak in body or mind are soon eliminated; and those that survive commonly exhibit a vigorous state of health. We civilized men, on the other hand, do our utmost to check the process of elimination; we build asylums for the imbecile, the maimed, the sick;….Thus the weak members of civilized societies propagate their kind. No one who has attended to the breeding of domestic animals will doubt that this must be highly injurious to the race of man.”

              At another point in The Descent of Man, Darwin writes:

              “The civilized races of man will almost certainly exterminate, and replace, the savage races throughout the world.”

              And exterminating everyone perceived to be “savage” or unworthy of passing on their genes is exactly what the Nazis tried to do. It is not, then, difficult to see why Weikart was justified in saying that:

              “Darwinism by itself did not produce the Holocaust, but without Darwinism…neither Hitler nor his Nazi followers would have had the necessary scientific underpinnings to convince themselves and their collaborators that one of the world’s greatest atrocities was really morally praiseworthy.”

              This can be seen in the statements made by Hitler which betrayed his Darwinist views. Hitler once said:

              “The law of selection justifies this incessant struggle, by allowing for the survival of the fittest. Christianity is a rebellion against natural law, a protest against nature. Taken to its logical extreme, Christianity would mean the systematic cultivation of the human failure.”

              • What do you think, are these statements objectively true, or were the Sandy Hook killer and the Nazis merely violating subjective cultural norms?

                Objectively true? Wow—that would be a remarkable claim. As I noted, I’ve never seen any evidence of objective moral truth. You got any?

                As historian Richard Weikart points out in his book From Darwin to Hitler, the racist Nazi rationalization for killing comes straight from Darwin.

                You have a short memory. German anti-semitism came from our ol’ buddy Martin Luther.

                At another point in The Descent of Man, Darwin writes…

                (1) No one cares what Charles Darwin wrote. That’s history. No one consults the Great Darwin to make sure his new hypothesis comports with what the great man thought.

                (2) Eugenics is policy, not science. You don’t like eugenics? Then complain to the politicians who implemented it.

                This can be seen in the statements made by Hitler which betrayed his Darwinist views.

                Huh? Books by Darwin were banned by the Nazis!

                • Scott Youngren says:

                  Bob,

                  You ask if I have any evidence of objective moral truth. On atheism, what we call “morality” is just sociobiologically ingrained behavior patterns. Therefore, when an atheist makes a moral judgement like, “Everybody has the right to believe what they want,” they are being inconsistent with their worldview.

                  Bob, do you believe that you have the right to believe what you want? Do you believe that you have the right to be an atheist and that I shouldn’t be able to force you to be a Christian? If so, then you believe in an objective moral value…the right to choose your beliefs. A lot of atheists are appealing to the objective moral value of freedom of religion when they fight against public expressions of Christianity.

                  If someone were to burglarize your house, you are very likely to say (or at least think) something to the effect of, “They should not have done that!!!” And when you said (or thought) that, you would be appealing to an objective moral standard. Namely, you would be appealing to the objective moral standard that burglary is wrong.

                  Regarding this subject, C.S. Lewis wrote:

                  “I know that some people say the idea of a Law of Nature or decent behaviour known to all men is unsound, because different civilisations and different ages have had quite different moralities.”

                  “But this is not true. There have been differences between their moralities, but these have never amounted to anything like a total difference. If anyone will take the trouble to compare the moral teaching of, say, the ancient Egyptians, Babylonians, Hindus, Chinese, Greeks and Romans, what will really strike him will be how very like they are to each other and to our own.”

                  Now some societies have deviated from these objective moral standards…societies such as Nazi Germany. This is why societies which supported the objective moral standard that genocide is wrong had to fight a war against the Nazis.

                  On the atheist view, the serial killer, for example, is just acting out of fashion, like a person who belches at the dinner table. Is serial killing merely unfashionable in virtually every society that has ever existed?! Or is the view that “serial killing is morally wrong” an objective moral value?

                  What Hitler did was just contrary to the patterns of sociobiologically behavior that has been ingrained into the human species not to kill each other off, according to atheism. Therefore, we have no right to accuse Hitler and the Nazis of any wrong doing if atheism is true.

                  Bob, I cited an historian (Richard Weikart from California State University) for my points about Darwinism leading to the holocaust. Please cite an historian from an accredited university who will back up your allegation that Martin Luther was the one who was responsible for German anti-semitism. I am suggesting that you will not be able to. We need to be very careful to distinguish between hearsay and gossip, on one hand, and legitimate history, on the other hand. In an age of internet search engines, it should be very easy for you to find such an historian…if one really exists.

                  Scott

                  • On atheism, what we call “morality” is just sociobiologically ingrained behavior patterns.

                    “On atheism”? I think you should be looking for the standard we both use, the dictionary. Morality is pretty clearly defined. There’s no objective anything in there.

                    when an atheist makes a moral judgement like, “Everybody has the right to believe what they want,” they are being inconsistent with their worldview.

                    I don’t follow. Where’s the inconstancy?

                    do you believe that you have the right to believe what you want? Do you believe that you have the right to be an atheist and that I shouldn’t be able to force you to be a Christian? If so, then you believe in an objective moral value…the right to choose your beliefs.

                    We must be not sharing the same definition of “objective.”

                    If you mean a reliably verifiable belief, then “Bob has a yellow car” would be objectively true. There are simple tests that you could use to find this to be a true statement. And with this definition, these shared moral beliefs you list would be “objective.”

                    But that’s not an interesting definition of the word. More relevant is Wm. Lane Craig’s definition, that objective moral truth is something that would be true whether there were people here to experience it or not. This is how Christians often use the word. And this is the one that must be defended. If you say that objective moral truth in this sense doesn’t exist, join the club.

                    If someone were to burglarize your house, you are very likely to say (or at least think) something to the effect of, “They should not have done that!!!” And when you said (or thought) that, you would be appealing to an objective moral standard. Namely, you would be appealing to the objective moral standard that burglary is wrong.

                    And this is objective moral value as something viscerally or universally held. Yes, that’s yet another definition but not the interesting one.

                    “If anyone will take the trouble to compare the moral teaching of, say, the ancient Egyptians, Babylonians, Hindus, Chinese, Greeks and Romans, what will really strike him will be how very like they are to each other and to our own.”

                    We’re all Homo sapiens. Yes, we have common moral programming. This is also not what I would call objective moral truth.

                    On the atheist view, the serial killer, for example, is just acting out of fashion, like a person who belches at the dinner table. Is serial killing merely unfashionable in virtually every society that has ever existed?! Or is the view that “serial killing is morally wrong” an objective moral value?

                    “objectively true” = viscerally felt.

                    we have no right to accuse Hitler and the Nazis of any wrong doing if atheism is true.

                    Nonsense. I will with pleasure point out all the ways your morality is flawed. Indeed, anywhere it diverges from mine, it’s flawed.

                    You’ll say that this is inconsistent. Show me.

                    cite an historian from an accredited university who will back up your allegation that Martin Luther was the one who was responsible for German anti-semitism. I am suggesting that you will not be able to.

                    The German man who wrote On the Jews and Their Lies and launched the Protestant Reformation didn’t have an impact on German thinking about Jews? That surprises me.

                    I have little interest in a quote war where I argue that my historian has more credibility than yours. But you’re right that I can’t just toss out hearsay without backup. It seems quite plausible to me, and Wikipedia agrees: “Martin Luther (1483–1546), a German Reformation leader, had a significant influence on German antisemitism by his virulent anti-Jewish statements and writings.”

                    • Scott Youngren says:

                      Bob,

                      You say that beliefs need to be “reliably verifiable.” You believe that beliefs must be able to “pass simple tests” to see if they are objectively true. But your view is self-refuting, since it cannot pass any such “simple tests.” This is the self-refuting incoherence in which atheism is caught. Please describe for me the “simple tests” which you would use to verify the truth of the following statements you have made (in bold):

                      1)If you mean a reliably verifiable belief, then “Bob has a yellow car” would be objectively true. There are simple tests that you could use to find this to be a true statement. And with this definition, these shared moral beliefs you list would be “objective.”

                      What “simple test” did you apply to verify that “there are simple tests” which one could use to find out if a statement is objectively true? Perhaps it was a chemistry experiment involving a bunsen burner and test tubes? Did you just assume that your methods for verifying objective truth are objectively true, or did you verify them with some sort of empirical observation (“simple test”)?

                      2)”…your morality is flawed. Indeed, anywhere it diverges from mine, it’s flawed.”

                      What simple test did you use to verify that my morality is flawed where it diverges from yours? A biology experiment involving observation under a microscope, perhaps?

                      3)“objectively true” = viscerally felt.

                      By your own standards, if people share viscerally felt reactions, and there is no objective morality, then you should be able to cite the tests which you used to verify the objectivity of this belief. How did you observe these reactions inside of people’s minds? What “simple test” did you use to verify that these “visceral reactions” are not just the mechanistic and mindless behavior of “survival machines – robot vehicles blindly programmed to preserve the selfish molecules known as genes”…as the famous atheist biologist Richard Dawkins describes human beings?

                      4)”…you should be looking for the standard we both use, the dictionary.”

                      Please describe the “simple test” (or empirical observation) which you used to verify the objective truth of this statement. What test did you use to verify the objective truth of your belief that I should be using the dictionary definition?

                      Regarding Martin Luther, I was not talking about anti-Semitism. Luther was a sinful human being, like all of us, and perhaps you are right that he held anti-Semitic beliefs. But merely having anti-Semitic beliefs is something entirely different from philosophically justifying the sending of millions of people to gas chambers. As Weikart demonstrates in his book From Darwin to Hitler, a severely degraded concept of human dignity is required to commit such atrocities. If people do not have any intrinsic or transcendent value (which is objective value), and are really just mechanistic and mindless “robot vehicles blindly programmed to preserve the selfish molecules known as genes,” then what moral objection could one possibly have to sending such “robots” to their deaths by the millions?

                      If people don’t objectively have any more intrinsic or transcendent value than cattle, then what objection could one possibly have to slaughtering people by the millions like cattle…as the Nazis, and the atheist communists did?

                      For people to have intrinsic and transcendent value—a value which is objectively real—there has to be an objective reference point for this value. Atheism provides no such objective reference point…merely the subjective opinions of various people.

                      Atheists can choose to judge human dignity and transcendent value to be their subjective truth, and many atheists are good and nice people who choose to do just that. However, since atheism holds that morality is purely subjective (as you do, Bob), atheists are also free to choose bloodthirsty mass murder as their subjective morality.

                      This is why it should be no surprise that the most prolific killers in all of history were sworn atheists…people such as Joseph Stalin, Pol Pot, Mao Tse Tung, and the leaders of the French Revolution, etc. It is telling that the number of people who have been killed by atheistic communism is estimated to run as high as 110 million (sources: Death by Government by political science professor R.J. Rummel and The Black Book of Communism).

                      When it comes to the connection between atheism and unrestrained killing, mathematician David Berlinski hits the nail on the head in his book The Devil’s Delusion: Atheism and It’s Scientific Pretensions:

                      Somewhere in Eastern Europe, a [Nazi] SS officer watched languidly, his machine gun cradled, as an elderly and bearded Hasidic Jew laboriously dug what he knew to be his grave. Standing up straight, he addressed his executioner. “God is watching what you are doing,” he said. And then he was shot dead. What Hitler did not believe, and what Stalin did not believe, and what Mao did not believe, and what the SS did not believe, and what the Gestapo did not believe, and what the NKVD did not believe, and what the commissars, functionaries, swaggering executioners, Nazi doctors, Communist Party theoreticians, intellectuals, Brown Shirts, Blackshirts, Gauleiters, and a thousand party hacks did not believe, was that God was watching what they were doing. And as far as we can tell, very few of those carrying out the horrors of the twentieth century worried overmuch that God was watching what they were doing either. That is, after all, the meaning of a secular society.

                    • ?? You can only reply so many times?

                      You say that beliefs need to be “reliably verifiable.” You believe that beliefs must be able to “pass simple tests” to see if they are objectively true.

                      Not at all where I was going. I’m amazed that you’re confused by my straightforward attempt at clarifying. I was simply pointing out the squishy nature of the word “objective.”

                      Unfortunately, until we are using the same definition, there’s nothing to talk about. My suggestion is to use Wm. Lane Craig’s definition. If that’s what you want, then justify why objective moral truths (with this definition) exist rather than some other kind of “objective” (widely shared, viscerally felt, etc.)

                    • Scott Youngren says:

                      Bob,

                      As I said in my other comment, the readers of these comments are intelligent enough to recognize that you are trying to use semantic maneuvering to obscure the weakness of your stance. The definition of specific words is an irrelevant semantic issue…a smokescreen.

                      The pertinent question regarding the existence of God is whether or not there exists morality above and beyond subjective human opinion. If there is such a morality, then there needs to be a source for it. This is true regardless of what definition of “objective” one uses…Wm Lane Craig’s definition, or any other definition.

                      You gave the example of the civil rights movement in your other comment. Prior to the civil rights movement, the subjective opinions of the white majority regarding racial discrimination were in worse compliance with the objective moral truth that racial discrimination is wrong. This is not to suggest that full compliance with this exists today.

                      The leaders of the civil rights movement appealed to a higher objective moral standard, which many whites were able to recognize as being objectively true and superior to their subjective biases about race. This helped bring the biased subjective morality of the white majority more into compliance with objective moral truth.

                      If the leaders of the civil rights movement did not succeed by appealing to a higher objective moral truth, then how did they succeed in changing the subjective opinions of so many people in our society? How did they get so many white people to change their subjective opinions about race considering that there was no personal benefit for them to do so? Please explain.

                      Scott

  19. Scott:

    If people don’t objectively have any more intrinsic or transcendent value than cattle, then what objection could one possibly have to slaughtering people by the millions like cattle…as the Nazis did?

    I’ve been over this more times than I can count with Christians who never want to see if they can answer their rhetorical questions themselves, so I may be a little more impatient than usual. Seems pretty straightforward to me.

    “Morality” is an English word. We get the definition from the dictionary. There is no objective anything in the definition, so don’t pretend that morality has no meaning without an objective something-or-other attached to it.

    I see no objective morality in the “true whether or not people are here to appreciate it sense.” You think this exists? Then show me that (1) objective morality defined this way exists and (2) we humans can reliably access it. When you think of how morality has changed through the centuries (slavery, child labor, alcohol, civil rights, etc.), I think you’ll agree that such a thing doesn’t exist.

    • Scott Youngren says:

      Bob,

      What definition of “morality” we use is a semantic issue that is of no relevance. By bringing this up, you are trying to distract attention from the weakness of your stance. Of what relevance would it be whether or not there is something objective in a dictionary definition? Words serve to represent aspects of reality.

      The pertinent question is not whether there is something objective in a dictionary definition. Rather, the pertinent question is whether or not there exists objective morality…in other words, morality which exists above and beyond subjective human opinion. If there is an objective morality that exists above and beyond subjective human opinion, then there must be a source for this morality.

      The readers of these comments are intelligent enough to recognize that you are trying to use semantic maneuvering in order to create a smokescreen which would serve to obscure the weakness of your stance. Debating definitions is nothing but semantic maneuvering.

      Further, it is unclear why your “true whether or not people are here to appreciate it sense” of objectivity applies. You need to explain why this standard applies to the question of the existence of God. If there exists a morality which is above and beyond mere subjective human opinion, then there needs to be a source for this morality. Whether or not people are here to appreciate this morality is an utterly irrelevant, and is another distraction which you have created to evade my arguments.

      Human recognition of, and compliance with, objective morality has changed through the centuries, not the morality itself. As an example, you mention civil rights. Are you suggesting that it was moral to discriminate against people based on the color of their skin prior to the civil rights movement? Yes or no?

      We are still not in compliance with this one as a species, because racial discrimination (and other forms of discrimination) still exists. Just because we as a society have had a less-than-stellar track record of complying with the objective moral truth that racial discrimination is wrong, does not mean that this has not always been an objective moral truth.

      Before the civil rights movement, the biased subjective opinions about race of the white majority were unjustly in violation of the objective moral truth that racial discrimination is wrong. People like Dr. Martin Luther King helped bring the biased subjective opinions of the white majority more into compliance with objective morality.

      Scott

      • What definition of “morality” we use is a semantic issue that is of no relevance.

        You’re the one who brought up objective morality: “If people don’t objectively have any more intrinsic or transcendent value than cattle, then what objection could one possibly have to slaughtering people by the millions like cattle…as the Nazis did?” If we’re going to refer to “morality” or “objective morality” we kinda need to agree on what they mean.

        My point is simple: “morality” is quite different from “objective morality.”

        Of what relevance would it be whether or not there is something objective in a dictionary definition?

        Because when you claim that morality has an objective component, but we don’t find that when we actually look it up in a dictionary, your argument fails.

        the pertinent question is whether or not there is objective morality…in other words, morality which exists above and beyond subjective human opinion.

        Oh—so definitions are important to you. OK, that works for me as well.

        I think of objective morality as moral truth that would be true whether or not there were people to appreciate it. It seems that this is the definition you use.

        If there is an objective morality that exists above and beyond subjective human opinion, then there must be a source for this morality.

        Let’s not worry about the consequences. First, let’s verify that (1) objective morality exists and (2) that we humans can reliably access it.

        The readers of these comments are intelligent enough to recognize that you are trying to use semantic maneuvering in order to create a smokescreen which would serve to obscure the weakness of your stance.

        I have an idea: instead of handwaving about how weak my position is, actually show us. The readers of these comments are intelligent enough to recognize when you’re casting vague aspersions instead of actually doing the hard work of pointing out errors in my position.

        Debating definitions is nothing but semantic maneuvering.

        Since you’ve conflated morality and objective morality, I think we do need to go back to the beginning. Don’t blame me for your error.

        Further, it is unclear why your “true whether or not people are here to appreciate it sense” of objectivity applies.

        This is the angle Wm. Lane Craig takes. I thought this was the same as your position, stated above. If there’s a subtle difference, then I guess it’s good we’re discussing it so we can agree on a single definition.

        You need to explain why this standard applies to the question of the existence of God.

        We’re talking about morality.

        If there exists a morality which is above and beyond mere subjective human opinion, then there needs to be a source for this morality.

        Whatever. I see no evidence for such a morality, so I don’t worry about the consequences.

        Whether or not people are here to appreciate this morality is an utterly irrelevant, and is another distraction which you have created to evade my arguments.

        Wrong. Suggestion: instead of calling me a cowardly liar, just respond to the arguments.

        It seemed to me that a definition offered by Wm. Lane Craig wouldn’t be amiss here. If instead you see him as the pompous clown that I do, then I apologize for using him as an authority. Either way, we need to find a common definition.

        It is human recognition of, and compliance with, objective morality which has changed through the centuries, not the morality itself.

        Show us that objective morality exists first.

        Are you suggesting that it was moral to discriminate against people based on the color of their skin prior to the civil rights movement?

        From my position, no.

        we as a society have had a less-than-stellar track record of complying with the objective moral truth the racial discrimination is wrong, does not mean that this has not always been an objective moral truth.

        I never said otherwise. And the very large challenge remains for you to show that objective morality exists. I see no evidence.

  20. I don’t think that there are any third party readers of these comments who are fooled by your evasion tactics.

    I’m done. I’ve tried to work with you. If much of your argument is simply insulting your commenters, I’ll go elsewhere. Looks to me like you’re interested in an opportunity to preach, not to explore new ideas. I’ll also note that your comment is full of questions without any support for your remarkable claim that objective morality exists, despite my repeated requests.

    • Scott Youngren says:

      Bob,

      I never insulted you or called you any bad names. You are clearly a very smart man. And you are a worthy debate opponent. Further, I am certainly in no position to criticize you for your beliefs, since I held beliefs very similar to your own for many years.

      It is just that getting to the truth requires some tough love sometimes. You came to this website to try to discredit my arguments. If you are going to do so, it can only be through rational discourse. However, rational discourse becomes impossible when a person refuses to logically engage with my arguments, and tries to create distractions (“red-herrings”) instead.

      I pointed out the self-refuting nature of your epistemology: You apparently believe that we should only accept beliefs as objectively true which have been empirically verified (“Bob has a yellow car”). But, as I pointed out, this very belief cannot be empirically verified, and is therefore self-refuting. Atheism hides behind the facade of being an empirically based belief system. However, no belief system can be empirically based because truth can only be arrived at by the application of reason…not from bare empirical observation. Extra-scientific premises are a necessary part of the process of reason. It is just a question of which extra-scientific premises are most reasonable.

      I delve into this topic in my essays titled The Mythology of Atheism and I Believe in Science! Why Do I Need Religion?! An excerpt from the second of these two essays:

      How could a statement such as, “The only kind of knowledge that we can have is scientific knowledge,” be verified scientifically? With a chemistry experiment utilizing a bunsen burner and test tubes? With a physics experiment utilizing a particle accelerator? Because the belief that, “The only kind of knowledge that we can have is scientific knowledge” CAN NEVER ITSELF BE SCIENTIFIC KNOWLEDGE, it is a self-refuting belief.

      Or take the premise, “No belief can be accepted as true and rational unless it can be known by science or quantified and tested empirically.” How can THAT belief be known by science or quantified and tested empirically? It can’t, and therefore such a premise is self-refuting.

      Einstein surely understood that scientific knowledge cannot be the only kind of knowledge, and that it must necessarily interact with religious/philosophical reasoning…which is why he said, “Science without religion is lame, religion without science is blind.”

      You say that I do not present evidence for objective moral truth. But I have presented evidence…you just refuse to respond to it. If someone burglarized your house, for example, I am quite certain that you would express a sense of moral outrage. You would probably say (or at least think) something to the effect of, “That burglar was wrong to have done that.” Regarding this point, C.S. Lewis writes:

      “If ‘being good’ meant simply joining the side you happened to fancy, for no real reason, then good would not deserve to be called good. So we must mean that one of the two powers is actually wrong and the other actually right.”

      “But the moment you say that, you are putting into the universe a third thing in addition to the two powers: some law or standard or rule of good which one of the powers conforms to and the other fails to conform to. But since the two powers are judged by this standard, then this standard, or the Being who made this standard, is farther back and higher up than either of them, and He will be the real God.”

      This law or standard of good which Lewis refers to is what allows us to judge the Nazis and burglars (etc.) as being morally wrong in an objective sense. If there were no standard of good which exists independent of human opinions, then there would be no way for us to make any moral judgements such as, “It is morally wrong to try to exterminate entire ethnic groups” or, “It is morally wrong to commit burglary.”

  21. Non Credenti says:

    I haven’t read all the comments, but I’m a bit surprised the discussion wasn’t finished after Scott’s comment of November 18, in which he conceded the argument by admitting that the premise for his entire essay was wrong. (The premise being that atheists don’t have an avenue to certain rationality and knowledge, but theists do.)

    Scott says, “Do I know if we aren’t just brains in vats? No, I don’t,” and there he obliterates his own position. His essay argues that atheists cannot be certain that they know something because they can only arrive at knowledge via reasoning, but that reasoning is fallible. But here he admits that the theist is in nearly the same epistemological boat. (Actually, it’s worse because–in addition to having the same imperfect sense perceptions, memory, and cognitive faculties as the atheist–the Christian claims as his source of knowledge and rationality an admitted deceiver who sends delusions and deceptive spirits, and impairs their cognitive faculties by ‘hardening their hearts.’)

    According to Scott, the Christian’s avenue to knowledge is the vague notion that, “Christian theism anchors man’s rationality in God’s rationality.” But by acknowledging that he cannot know he isn’t a brain in a vat, he admits to the exact same limitations as every–as far as our senses, memory, and cognitive faculties are concerned—as everyone else. Scott cannot know that his rationality is anchored in God’s rationality when that conclusion only comes to him via his own imperfect cognitive faculties. Scott cannot know that God is perfectly rational to begin with, when that conclusion only comes to him via his own imperfect cognitive faculties. In the same way he wants to say the atheist cannot trust his conclusions because they stem from imperfect cognitive faculties, he cannot trust his.

    Scott continues, “But you are confusing two entirely different concepts:
    1) An ontological question about the nature of physical reality (if we are brains in vats, or if there exists an outside physical world).
    2) The ability of our reason to lead us to truth.”

    Scott is the one who is confused. The Brain-In-Vat (BIV) thought experiment, itself an update of Descartes’s Evil Demon, is a key concept in the Philosophy of Mind. It forces us to consider that we don’t know everything we think we know—at least (and this is important) not with certainty. The point of BIV is not whether we actually are a BIV or not; it’s that we cannot KNOW if we are a BIV or not.

    Here is a link to an article about BIV by Jose Zalabardo, the head of the Philosophy department at University College London. http://www.ucl.ac.uk/~uctyjlz/BIVRevisedRIP.pdf All one need do is read the first sentence (or spend five seconds thinking about the thought experiment) to see that BIV is an epistemological issue (dealing with knowledge), not an ontological one.

    Scott concludes, “Put another way, we could be brains in vats with reasoning ability that leads us to truth.”

    This is also exactly wrong. The whole point of the BIV scenario is that—with a bare few exceptions–we cannot arrive at (absolute) truth because we cannot know (with absolute certainty) that our memories and senses are not false signals being injected into our brains by a demon, mad scientist’s supercomputer, or squid-like machine a la The Matrix (where we were bodies in vats).

    Notice that I’ve added a bunch of parenthetical clarifications to my use of words like “knowledge.” There are two ways we can talk about knowledge: First, we can talk about absolute knowledge; something we know with certainty, we could not be wrong. I “know” my mother-in-law’s name is Susan, but I haven’t seen her birth certificate. Even if I have, am I absolutely sure it isn’t a forgery, or that she hasn’t stolen her dead twin sister’s identity? It turns out I don’t have absolute knowledge her name is Susan. If I extend this idea to the extreme, I don’t have absolute knowledge that I’m sitting here at my computer typing this. Maybe I’m in a hospital, in a coma, hallucinating this activity. Maybe I’m a BIV and all of my sense perceptions and memories are false. As we can imagine, the list of things we KNOW is exceedingly short.

    The second way we talk about knowledge is in a provisional way. I’m reasonably sure I’m sitting at my computer typing this, but only 99.99999% sure—still that’s close enough that it would be perverse to withhold assent. Provided I’m not a BIV, then, yes, I’m sitting here typing this. This is what we mean when we say that we “know” something in our everyday speech, though we don’t realize it because we’re not being that technical. If I’m talking to my young daughter I might say I’m 100% certain the KC Royals won the baseball World Series this year. Even if I’m talking to a philosophy buff, I’d still say I’m 100% certain of this, even though we both know I’m only 99.999999% sure, and I’ll trust the philosopher to not bog us down in a senseless BIV scenario.

    This entire presuppositional apologetic hinges on the equivocation that since we cannot KNOW everything, we cannot know anything. There are many things we can know provisionally, even though we cannot KNOW them absolutely.

    The rationality argument is similar. Reason depends on memory and, usually, our senses. But since our memory is not perfect, our reasoning ability (rationality, cognitive faculties) cannot be perfect, either. The apologist concludes that if we cannot be assured of 100% perfect rationality, we cannot claim any rationality at all. The vacuousness of this argument should be apparent by now.

    • Scott Youngren says:

      Non Credenti,

      You are here committing an absolutely terrible straw man argument: You utterly misrepresent my argument and then precede to attack the mischaracterized straw-man version of my argument…rather than my actual argument.

      I have seen a lot of straw man arguments in my time, but this one truly takes the cake.

      I am not saying that we cannot trust our reasoning faculties. Rather, I am saying that we cannot trust our reasoning faculties if we accept atheism as true. The only way to trust our reasoning faculties is to reject atheism and the naturalist worldview that serves as a ground for atheism.

      Your below characterization of my argument isn’t even remotely close to my argument:

      “His [Scott’s] essay argues that atheists cannot be certain that they know something because they can only arrive at knowledge via reasoning, but that reasoning is fallible.”

      and:

      “The apologist concludes that if we cannot be assured of 100% perfect rationality, we cannot claim any rationality at all.”

      and:

      “This entire presuppositional apologetic hinges on the equivocation that since we cannot KNOW everything, we cannot know anything.”

      Please try to respond to my actual argument rather than your mischaracterized version of my argument (your straw man).

      I am not arguing that we cannot trust our reasoning. Rather, I am only arguing that we cannot trust our reasoning if we start from the atheistic assumption that our reasoning evolved from an evolutionary mechanism that is driven by survival value. An evolutionary mechanism that works to promote survival does just that….it promotes survival, not truth.

      We do not need truth to survive because false beliefs can produce the same survival value as true beliefs. You will recall the example that I gave. Both of the below beliefs would produce the same survival value:

      —“I should not eat this plant because it contains poison.”

      —“I should not eat this plant because doing so will turn me into a werewolf.”

      The atheist worldview destroys our ability to trust ANY of our beliefs…such as a belief in atheism.

      The brain-in-the-vat is an example of a question (which is actually both ontological and epistemological) that we cannot answer with 100% certainty because we do not know everything. But to suggest that we cannot come to any reasonable conclusions because our perceptions have limitations is not even remotely close to my argument.

      I have never argued that we can arrive at absolute truth with 100% certainty. Of course our senses, memory, and cognitive faculties have limitations. Who on earth would argue otherwise? But to argue that we have to have 100% access to absolute truth in order to determine if theism or atheism is a more reasonable conclusion is a complete non-sequitur (does not follow).

      You do not know with 100% certainty that your employer will continue to pay you if you continue to go to work. But since you probably believe that your employer will continue to provide you with pay, you continue to go to work.

      Charles Colson and Nancy Pearcey do an even better job of demonstrating the absurdity of the atheist naturalistic position in their book How Now Shall We Live?

      Christians ought to argue that scientific naturalism is incoherent and self-contradictory, for scientists must exempt themselves from the very framework they prescribe for everyone else. All human beings are reduced to mechanisms operating by natural causes-except scientists themselves. Why? Because to carry out their experiments, they must assume that they, at least, are capable of transcending the network of material causes, capable of rational thought, of free deliberation, of formulating theories, of recognizing objective truth. They themselves must form the single glaring exception to their own theory. This is the fatal self-contradiction of naturalism.

      Lewis pointed out another contradiction that is equally devastating. The naturalist assumes that everything that exists can be explained in terms of natural forces. But that assumption itself cannot be the result of natural forces or it would not qualify as a genuine truth claim. For if an idea is simply the product of particles bumping around in our brains, then it is neither true nor false but merely a natural phenomenon. If, for example, a man tells us his room is on fire but we know that he just swallowed a hallucinogenic drug, then we probably will not call the fire department. If we think an idea is the result of physical, chemical causes in the brain, then we discount it and don’t even credit it as a rational thought.

      Now, scientific naturalism necessitates the conclusion that all ideas are products of natural causes in the brain-including the idea of scientific naturalism itself. Thus, if it is true, then it is not a rational thought and ought to be discounted. “Every theory of the universe which makes the human mind a result of irrational causes is inadmissible,” Lewis wrote. For “in order to think, we must claim for our reasoning a validity which is not credible if our own thought is merely a function of our brain and our brain a byproduct of irrational physical process.”‘

      As philosopher Charles Taylor puts it, the atheist materialist functions as though he were an “angelic observer” somehow able to float above the determinist cage in which he locks everyone else.

      Non Credenti, once again: Theism grounds reason in the reason of God’s mind. Conversely, atheism grounds reason in irrational physical processes.

      The theistic model is strongly supported by what we know about modern physics, as I demonstrate in God Is Real…Why Modern Physics Has Discredited Atheism, and with what we know about modern biology, as I demonstrate in There’s Nothing Random About Evolution.

      • Non Credenti says:

        Well I certainly wouldn’t want to be guilty of a straw man argument, so let’s see if I can clarify.

        With the first quote, I was attempting to summarize a lengthy essay in one sentence, and in doing so, left out a detail. What if I amended my summary to, “Atheists cannot be certain that they know something because they can only arrive at knowledge via reasoning, but that reasoning is fallible, since it works from the assumption that said reasoning derives from an evolutionary mechanism which utilizes survival value, not truth value”? I don’t know… it seems like this is what I was saying; only that I left out the reason why our reasoning is fallible. I don’t think it’s necessary to detail your why if I’m not arguing with your conclusion. Regardless, if you’re satisfied with my re-working the summary, I’m happy to amend it, and I’m equally happy to apologize for any confusion. If you think it still has problems, let me know.

        Still, a strawman argument is attacking a mis-representation of another’s argument. I didn’t attack your claim regarding atheists, reason, evolution, and truth. (I think I could, and I think I will shortly, but I don’t think I did in my post.) I was willing to grant that point for the moment, because I was making a different point about your BIV admission. So I’m not sure why you’re so upset over a quick summary that we would seem to agree on, and that I didn’t attack. Maybe I’m wrong. I invite you to show me where I attacked a mis-representation of your account of atheist reason.

        I’ll take the blame for the next two statements you quote. The paragraphs regarding absolute and provisional knowledge, which include these quotes, were not really direct responses to your essay or your admission of defeat where BIV is concerned. I was attempting to give some clarifying information to other readers about the general presuppostional argument, which you seem to be trying to make, and I didn’t clarify the change in focus, so I apologize. That’s definitely my fault. A number of times in the comments you said things like, ”Further, without any reason for trusting the accuracy of our beliefs, we cannot judge any of our beliefs to be true…” (Emphasis mine) With statements like these, which traffic in extreme “any” statements, perhaps I can be forgiven for melding the standard presuppositionalist argument with your version of it. Still, I’ll repeat that I was very unclear, and I apologize. But again, even if it seems I unintentionally mis-represented your claims, I didn’t attack them, so not a strawman, just a bad summary.

        You want me to respond to your “ACTUAL ARGUMENT,” but I didn’t set out to do that, and I didn’t need to–you did it for me! The premise of your essay was two-fold—you attacked atheistic reason, and offered an explanation for theistic reason, but then decimated your theistic claims all by yourself.

        It’s ironic that you became so agitated over my perceived refusal to address your argument while completely ignoring mine. I offered up a quick summary of your argument only as a lead-in my actual point, which was your admission that your argument fails, but the part you threw a fit over was the quick lead-in summary? I said “he obliterates his own position,” and you had nothing to say? That’s telling.

        The brain-in-the-vat is an example of a question (which is actually both ontological and epistemological)….
        No, still wrong. BIV is an epistemological issue. It’s no more ontological than a sports discussion is ontological because baseballs exist.

        … that we cannot answer with 100% certainty because we do not know everything.
        Wrong again. It has nothing to do with whether or not we know everything. I’ve already explained it to you and provided a link. The point of BIV isn’t whether or not an external world exists (it does whether we have access to it or not); it’s that we cannot KNOW if our mind’s map of the external world is accurate or not.

        To recap what my first post was actually about:
        – If the Christian grounds his rationality on a God who admits to being a deceiver, he has a known unreliable mechanism. He cannot make any greater claims of access to truth than the atheist. (The atheist is bumbling around thinking about werewolves, while the theist is listening to a deceiver.)
        – Even if the Christian God did not admit to being a deceiver, the Christian cannot KNOW his God isn’t one, and has no independent way to verify any Divine pronouncements, so he still cannot make any greater claims of access to truth than the atheist.
        – If the Christian believes he is receiving some direct communication from God, he cannot know God is being truthful.
        – If the Christian believes he is receiving some direct communication from God, he cannot know that it is actually God sending the message (could be Satan).
        – If the Christian believes he is receiving some direct communication from God, he cannot know that his fallible senses are receiving the message correctly. (Too many Christians kill their own children because they perceive that God is directly ordering them to.)
        – If the Christian grounds his rationality on God’s rationality, you need to explain how this grounding takes place, or is accomplished.
        – If The Christian grounds his rationality on God’s rationality, you need to explain how you know God is rational to begin with. If God is irrational, He’s hardly likely to admit it to you (or realize it Himself.)
        – If you try to launch a rational argument for your own or God’s ultimate rationality, then you are using your (fallible) reason to reason about reason, which is circular and invalid. (Here the atheist is in the same boat.)

        • Scott Youngren says:

          Non Credenti,

          The below is merely an addendum to my other reply…in which I respond to your two most recent comments:

          University of Delaware physicist Stephen Barr echoes Nancy Pearcey’s and physicist Amit Goswami’s (cited in my other reply) points about the contradiction between the materialist/naturalist worldview (in which atheism is grounded) and the existence of ideas, or abstract concepts, in Modern Physics and Ancient Faith:

          “Cognitive scientists talk about neurons, for example. But ‘neuron’ itself is an abstract concept that arose from the researches of biologists. For the materialist, then, even this concept of ‘neuron’ is nothing but a neurological creation; it also is a pattern of neurons firing in someone’s brain. If this sounds like a vicious circle, it is. We explain certain biological phenomena using the abstract concept ‘neuron,’ and then we proceed to explain the abstract concept ‘neuron’ as a biological phenomenon—indeed, a biological phenomenon produced by the activity of neurons. What we are observing here is the snake eating its own tail, or rather its own head. The very theory which says that theories are neurons firing is itself naught but neurons firing.”

          “…Why should anyone believe the materialist, then? If ideas are just patterns of nerve impulses, then how can one say that any idea (including the idea of materialism itself) is superior to any other? One pattern of nerve impulses cannot be truer or less true than any other pattern, any more than a toothache can be truer or less true than another toothache.”

          Indeed, as Barr alludes to above, the existence of truth (and ideas) cannot be explained through the lens of the materialist/naturalist worldview. If true, the materialist belief that human consciousness is nothing but the firing of neurons in the brain cannot be anything other than…the firing of neurons in the brain. If consciousness is nothing but neuronal impulses, how could a true neuronal impulse be distinguished from a false one? By measuring the voltage of the impulse?

          Atheism is caught in a circular incoherence.

  22. Non Credenti says:

    To make his claim that atheists cannot trust any of our beliefs under an evolutionary paradigm, Scott makes the following argument:

    “We do not need truth to survive because false beliefs can produce the same survival value as true beliefs… Both of the below beliefs would produce the same survival value:
    —‘I should not eat this plant because it contains poison.’
    —‘I should not eat this plant because doing so will turn me into a werewolf.’

    The atheist worldview destroys our ability to trust ANY of our beliefs…such as a belief in atheism.” (emphasis of mine)

    Sure, we can imagine specific circumstances in which having a short neck is better for a giraffe, but we can’t ignore the more numerous, more likely, and more weighty circumstances where a long neck is advantageous. We can also imagine times that exposing ourselves to large doses of radiation can make us healthier (if we have cancer), but I wouldn’t recommend it as a general rule. Likewise, It’s trivially easy to see that false beliefs can sometimes produce the same survival value (as in the werewolf example) as true beliefs, but it’s equally trivial to see that, most often, true beliefs have more survival value, and thus, overall, true beliefs are better for survival.

    Let’s imagine Joe finding a poisonous plant. What odds of survival does he have under various circumstances? Let’s choose E for Eat, and A for Avoid. If he eats it he’s dead, and if he avoids it he lives (for now). The various analogies and quotes Scott marshals for his argument would have us believe this is all random—that thoughts are just flitting through Joe’s empty head like butterflies, and he chooses them at random. (Scott doesn’t explicate this randomness, but it seems the only option, since he’s claiming there’s no reason to choose one belief over another on the atheist view.) But Joe also randomly forms a justification for his decision. If he eats it, his only real justification is that he thinks it is edible, so we’ll call that E-E. If Joe avoids it, there are millions of possible justifications, but let’s just simplify and say one justification is he thinks it’s poisonous (A-P), and the other is that it will turn him into a werewolf (A-W). So Joe has a 67% chance of living if he chooses one of these beliefs randomly:
    E-E (death): 33%
    A-P (life): 33%
    A-W (life): 33%

    Scott says via quotation that natural selection (NS) acts on beliefs, the truths of which are irrelevant as long as the conclusions are correct. But this isn’t the case, and it seems to me that this mischaracterization causes the chief confusion. In this case, NS acts on genes, genes affect the brain’s physiology, and the brain’s physiology affects beliefs. If we want to identify a more proximate agent than genes for NS to act on, the closest we can get is the brain’s physiology (via genes), not the beliefs themselves.

    The physiology of the brain is not a random belief generator–among other traits, it is an impressive pattern-recognition machine. In fact, many neuroscientists say that recognizing patterns is what the brain does best and most often, and that the brain is the world’s best pattern-recognition machine. The powerful survival advantages this confers should be obvious.

    Imagine that Mary has evolved the brain physiology to find patterns in (what it perceives as) reality. She notices the pattern that when Fred and Ethel both ate certain plants, they died. She also notices that other plants are frequently eaten by she and her friends, who don’t die. Further, she has noticed that on a number of occasions, when she touched the plants that had poisoned others, they caused a rash on her skin. Putting all this together, when encountering a new plant, she first crushes the leaves and rubs them on her skin. If it doesn’t cause a rash, she concludes it’s probably safe to eat. If it causes a rash she concludes it’s probably poisonous.

    Joe and Mary both happen upon a strange plant of which they have no previous experience. Joe picks one of the pretty little thought butterflies roaming around his brain, while Mary engages her pattern-recognition faculties, crushes a leaf and rubs it on her arm. When her arm develops a rash, she avoids the plant because it appears to be poisonous.

    As we’ve seen, Joe’s chances of survival upon encountering this plant are 67%. Mary’s survival chances, however, are much higher. The chances are not 100% (maybe this poisonous plant doesn’t cause a rash) but her pattern-recognition machine doesn’t need to confer 100% survival odds, it only has to be better than Joe’s random selection method and nature will select for it.

    Importantly, notice that Joe and Mary don’t have to pass just this one test. So Mary reasons that this plant is likely poisonous, and Joe catches the A-W thought butterfly and they both avoid the plant and continue down the path. Soon they happen upon a frog on a log. Mary remembers that brightly colored frogs are often poisonous (using much the same reason as she did with the plant), and avoids it. Joe grabs a random thought and concludes the frog will be yummy, and he dies from the toxin on its skin. The point is that Scott’s argument relies on a one-off event where false beliefs happen to be no worse than true beliefs (this time), but he ignores the patently obvious concept that a systematic method of making good decisions is infinitely better than one that relies on a continuous series of lucky guesses.

    Scott’s argument isn’t against recognizing patterns, but likewise, recognizing patterns isn’t the only tool Mary has at her disposal. She combined that with her ability to imagine consequences and think abstractly. These abilities are also granted by her brain’s physiology, which was selected by nature because it aided her in navigating the complex social structures of her society, in addition to helping her not eat poisonous plants. Likewise, the survival advantages of being able to remember past incidents, think abstractly, recognize patterns, draw conclusions, and to make plans accordingly—to reason—are equally obvious or more so.

    To conclude, Scott says:
    “The point of the essay is that, assuming the naturalist/materialist worldview is true, we have no reason for trusting the accuracy of our beliefs…and therefore no reason to think that we can reason accurately. Further, without any reason for trusting the accuracy of our beliefs, we cannot judge any of our beliefs to be true…such as a belief in naturalism/materialism.”

    To the contrary, we’ve seen at length that we have plenty of reason to think true beliefs confer a significant survival advantage. Nature will select for the physiologies that produce this ability to reason accurately and arrive at true beliefs. Therefore, we have reason to trust in our ability to reason accurately, and thus trust the general reliability of our beliefs… such as a belief in naturalism/materialism.

    If anyone seriously thinks forming reliable (and reliably true) beliefs has no survival advantage, I’ve got a frog to sell them, wrapped in a nice decorative leaf.

    • Scott Youngren says:

      Non Credenti,

      I will reply to your last two comments below. If I forget anything, please let me know:

      You do not clarify how I “decimated [my] theistic claims all by [myself].” You baldly assert this, but do not provide any logical demonstration. You suggest that I admit that my argument fails. Where did I do this? This is very strange.

      Your argument is still a straw-man because you continue to misrepresent my argument and then precede to attack the misrepresented version of my argument. Specifically, you misrepresent my argument as consisting entirely of doubts about the reliability of reason under assuming the truth of atheism. It is true that I doubt the reliability of human reason if we assume that the naturalist worldview is true. And so did Charles Darwin, as I have demonstrated.

      You have expressed doubts about the reliability of reason assuming that God is the source of reason. But you seem to confuse expressing doubts about my grounding for reason with providing your own grounding for reason. Further, I am not sure how it is that you come to the conclusion that God is a deceiver. This is very strange.

      My argument also consists of the point that it is impossible, even in principle, to ground the naturalist worldview (in which atheism is rooted) in reason. You precede to attack my grounding for reason (God), but you simultaneously fail to provide your own grounding for reason. I am very extremely curious to hear how you provide a grounding for reason.

      Bo Jinn hammers down the inability of atheism to ground reason in Illogical Atheism:

      Scientific facts cannot justify reason. It is reason justifies science. But, then, what justifies reason? The reliability of reason, just as the existence of morality and beauty is simply taken for granted by the atheist on purely pragmatic grounds. There is no sufficient ontic referent for their actual existence. Truth/ value judgments can be supported by empirical facts, but at the very last instance they will always require a judgment from a personal agent. And unless that judgment is made on the basis of an objective standard of truth, then the judgment is therefore meaningless. By denying a sufficient reason for the existence of the universe, that objective standard of truth simply does not exist. It is lost in the nihilistic vacuum left in the wake of the death of God.

      If human minds are the only personal agents in existence, it follows that all truth judgments based on reason are completely relative…As we speak, there are atheists the world over insisting that atheism is a conclusion which intelligent people come to on the basis of reason. But, if atheism is true, then human reasoning has no validity at all, because valid reasoning implies a standard of truth that can be reasoned toward and a sufficient reason for believing that human reasoning works in the first place. That guarantee is non-existent in an atheist world. Since the universe is a self-evident brute fact which exists (like us) for no purpose in and of itself we, in effect, become our own standard of truth and reasoning is thus rendered completely invalid.

      So, Non Credenti, I will ask you directly: What ontic referent do you provide for reason? What objective standard of truth does atheism reason toward? Your standard? Richard Dawkins’ standard? Carl Sagan’s standard? Human standards of truth can only be subjective, not objective.

      Non Credenti, for the THIRD TIME: Theism grounds reason in the reason of God’s mind. Conversely, atheism can only ground reason in irrational physical processes. The naturalist assumes that all beliefs are the result of irrational physical processes in the brain. You gave us a perfect example of this with your long discourse about how Mary evolved the brain physiology to find patterns in (what it perceives as) reality. In your example, Mary is a natural mechanism with its reasoning grounded in irrational physical processes. Your explanation is self-defeating because it necessitates that your own theories about how Mary evolved the brain physiology for survival are nothing but the product of an irrational physical process that evolved to provide survival.

      If your worldview is true, then your ideas about how our reasoining evolved are natural phenomena in the brain. How can natural phenomena in the brain be either true or false? If we think that an idea is the result of physical, chemical causes in the brain, then we don’t even credit it as a rational thought…as with the example of the man who tells us his room is on fire because he just took a hallucinogenic drug.

      Again, the results of mindless and irrational natural forces cannot qualify as genuine truth claims. Brain physiology is a natural phenomenon and natural phenomena can neither be true nor false. Similarly, the rock I see laying on the ground across the street is neither true nor false. Rather, it is merely a natural phenomenon.

      Regarding this point, Nancy Pearcey writes in her book Finding Truth:

      Materialism reduces thinking to biochemical processes in the brain, akin to the chemical reactions in digestion. But digestion is not something that can be true or false. It is just a biological fact. If thinking is reduced to brain processes, then our ideas are not true or false either. But in that case, how can the materialist know that materialism is true? The philosophy is self-refuting.

      As philosopher Charles Taylor puts it, the atheist materialist functions as though he were an “angelic observer” somehow able to float above the determinist cage in which he locks everyone else.

      You write, “These abilities are also granted by her brain’s physiology, which was selected by nature because it aided her in navigating the complex social structures of her society, in addition to helping her not eat poisonous plants.” But survival and navigating complex social structures are PURPOSES. Physicist Amit Goswami explains why suggesting that inanimate material things can have purposes (such as survival…let alone navigating complex social structures) is absurd:

      The Darwinian theory of evolution is based on natural selection: Nature selects those organisms that are fittest to survive. In the materialist view, an organism is just a bundle of molecules that are completely specified by their physical and chemical properties. Nowhere among these properties will you find a property called survivability. No piece of inanimate matter has ever attempted to survive or in any way tried to maintain its integrity under any circumstances. But living bodies do exhibit a property called survivability. Now the paradox. A Darwinist would say that the survivability of the living form comes from evolutionary adaptation via natural selection. But natural selection itself depends on survival of the fittest.

      See the circularity of the argument? Survival depends on evolution, but evolution depends on survival! A paradox is a sure-fire sign that the basic assumptions of the paradigm are incomplete or inconsistent; they need a reexamination. Aren’t biologists missing something by objectifying subjective phenomena? The philosopher Robert Efron (1968) thinks so:

      The reductionist attacks the definition and usage of every word, which has historically referred to an action of a living entity: “memory,” “reflex,” “free will,” “cognition,” and so forth. He then redefines the same word so that it will be applicable to an action of an inanimate entity. By using this epistemological technique he deludes himself into thinking that inanimate entities have the same properties found in living organisms, that a common denominator has been found, and that the problem of reduction has been “solved.” The solution is primitive animism expressed in scientific jargon.

      Whether or not an external world exists is an ontological question. If you did not mean the BIV example as an ontological question, that is fine, and I will let it go. But the BIV example does touch upon an ontological question about the existence of an external physical world…that is all that I was trying to say.

      Lastly, Non Credenti, you have again selectively ignored the points which you cannot respond to from the framework of your worldview. A copy and paste from my previous comment:

      The theistic model is strongly supported by what we know about modern physics, as I demonstrate in God Is Real…Why Modern Physics Has Discredited Atheism, and with what we know about modern biology, as I demonstrate in There’s Nothing Random About Evolution.

      Again, I am very extremely curious to hear your rebuttal to my two above essays regarding how both physics and biology strongly support the theistic model.

      • Non Credenti says:

        I appreciate your efforts at brevity in responding to my two long posts at once, but I suspect my response is going to be very long, so I might have to break it up into separate posts so you don’t have to take a vacation day to read it all at once.

        A quick point for clarity… Your essay had two main points—(a) an attack on atheistic reason and (b) support for theistic reason. Until my most recent post (December 8), I was only addressing (b). So when I said you had decimated your own argument, I meant you had decimated (b); I didn’t care about (a) at the time. The clarification you’re asking about (how you attacked your own argument) was at the end of the Dec. 7 post, in a series of bulleted points at the end of my response.

        I’m not going to reproduce them, but let me try to summarize them this way: If you want to claim that God is somehow, in some vague way, the ground for your reason, how can you know this? You cannot use your reason to reason toward your desired conclusion (that of reason) because that’s circular and assumes what you’re trying to discover. Even if God has access to or is the source of reason, you cannot know that He is willing or able to help you attain reason or truth. If God has created your cognitive faculties, they are whatever He wants them to be, and you cannot know that He hasn’t made sure that they are faulty and incapable of rationality (maybe He has something to hide, so he keeps you blinded to reality). The problem is made starker by the fact that God admits to being a deceiver at times, although even without the admission, you still couldn’t know He wasn’t. He also seems to portray Himself (assuming he’s the author of the Bible) as irrational at times, but again, even without the admission you couldn’t know He was rational.

        Note I’m not saying that on the claim of Divine grounding it’s impossible for the Christian to access reason/rationality/truth, just that it’s impossible to be sure he has access to them. He may feel he has access to something, but he can’t ever know that God is not lying to him, or that Satan is not impersonating God, or that he isn’t hallucinating the entire experience. By admitting that you can’t know this, you’ve refuted the (b) part of your argument and admitted that you’re in the same boat as the atheist.

        Suppose I say I ground my rationality on the invisible Magic Ice-Cream Sandwich (MICS). No doubt you would ask some obvious questions like: How do you know MICS herself is rational? How do you know MICS isn’t lying to you (about her being rational, or about anything else)? How do you know MICS is able to impart rationality to you, and that you properly understand what MICS is revealing? And if I show you MICS’s autobiography, in which MICS admits that she sometimes lies to puny humans, you’re going to be doubly skeptical of my claims. I might run through some mental gymnastics and try to lay out a case for MICS as the source of my rationality, but then you’d counter that as soon as I use my rationality to justify my rationality, my argument has become circular, and thus invalid. Replace MICS with God and you see the problem for your position.

        The point can be summed up thusly: Without using your reason, demonstrate that God is rational… or that He has made rationality available to you… or that he isn’t deceiving you with false senses or memories. It cannot be done, and that is true of anyone who attempts to justify his reason.

        You continue, ”Your argument is still a straw-man because you continue to misrepresent my argument and then [proceed] to attack the misrepresented version of my argument. Specifically, you misrepresent my argument as consisting entirely of doubts about the reliability of reason under assuming the truth of atheism. It is true that I doubt the reliability of human reason if we assume that the naturalist worldview is true… [Different paragraph] My argument also consists of the point that it is impossible, even in principle, to ground the naturalist worldview… in reason.”

        I’m sorry, but I’m going to need you to quote me claiming that your argument consisted entirely of doubts about the reliability of reason. To the contrary, I specifically said that in the interest of brevity I was offering only a one-sentence summary. Do I need to quote the entire essay every time I reference it so you won’t claim I’m leaving something out? Besides, when I did finally respond to your attack on atheist reason, on Dec. 8, My rebuttal encompassed both of the claims you’re making here. I said your attack failed because it attacks an incorrect notion of evolution (ironically, a straw man), hinging its argument on the false idea that NS acts on ideas rather than genes.

        Since we can follow the development of distinct regions of the brain, from the lizard R-complex, to the mammalian limbic system (along with some lizards), to the neo-cortex (shared with other primates and cetaceans), and have identified their functions, we can identify which areas of the brain—and which connections between different areas of the brain—are responsible for various higher mental functions. This isn’t the result of some thought experiments; scientists are working with actual brains, while you’re relying on the erroneous idea that NS acts on ideas. Once one understands that NS acts on genes, rather than ideas, the werewolf dilemma evaporates.

        Next you say, ”You have expressed doubts about the reliability of reason assuming that God is the source of reason. But you seem to confuse expressing doubts about my grounding for reason with providing your own grounding for reason. Further, I am not sure how it is that you come to the conclusion that God is a deceiver. This is very strange.”

        Sorry, but again the confusion is yours. If I find problems with an argument, I’m free to attack it without being required to present a better one—or, indeed, any alternative at all. For over a thousand years people noticed the retrograde motion of Mars, and pointed out the problems it caused to the geocentric, spherical orbit theory of astronomy. Do you imagine they were required to provide theories of their own before they could criticize one with obvious flaws?

        As for God being a deceiver, He says He sends delusions, deceiving spirits meant to entice people (king Ahab) to their deaths, and hardens people’s hearts so their mental faculties don’t work well, leading to the deaths of thousands of innocent children (Pharoah of Egypt). Even if He didn’t admit to these ghastly things, you cannot know he isn’t deceiving you, no matter how much He pinky-swears He’s telling the truth. (“But… but… but… God cannot lie! It says so right there in the Bible!” Yeah, that might be a lie.)

        From here your response gets into atheist grounding, so I’ll tackle that in a separate response.

      • Non Credenti says:

        To answer your questions about grounding reason, I’m going to lay some groundwork with some information about epistemology, or Theory of Knowledge

        The Problem of Global Skepticism: How do I know I’m not a Brain in a Vat?

        Short answer: I don’t, and neither does anybody else.

        I don’t worry much about the specific BIV question, because if I was a BIV I couldn’t know it, and couldn’t escape it, so I’d have no choice but to continue on with my life the way I always have. However, the point that BIV is making is an important issue in epistemology, which studies how we distinguish justified belief from mere opinion. It forces us to acknowledge the assumptions on which all of our knowledge claims—and therefore, our claims of reason/rationality–are based.

        When we make a knowledge claim, we need to give a justification for it; how do we know this thing? Our answer will require us to justify that claim, and so on, and so on. In philosophical jargon, we would say “I know that P because Q. I know that Q because R. I know that R because S, etc.” This would seem like an infinite chain of justifications, but it quickly ends because we reach a point where we can no longer justify our knowledge claims. A real-world example might help:

        1. I know this chair will support my weight because it’s made of metal.
        2. I know this chair is made of metal because when I tap it with my knuckle I can feel it, and it also makes a sound characteristic of metal.
        3. I know what sound metal makes when I hit it with my knuckle because I remember the sound from earlier experiences of hitting metal with my knuckle.
        4a. I know I can feel my knuckle hitting the chair because… uh… I just do.
        4b. I know I remember the sound of a knuckle hitting metal because… uh… I just do.

        It doesn’t take long to get to a point where we “just know” something, and we can’t justify it beyond that. As Wittgenstein said, “If I have exhausted the justifications, I have reached bedrock and my spade is turned. I am then inclined to say: ‘This is simply what I do.’” When you think about Brain-In-Vat (BIV) scenarios, you realize that in a very basic way, you don’t KNOW that your memory is correct, or that your sense perceptions are correct. Your mind thinks it remembers something, and thinks your knuckle feels something, but you also know that your minds and perceptions are sometimes wrong. You can justify your beliefs up to the points of senses and memory, but your senses and memories can not themselves be justified, you must simply take them for granted. That’s not the ideal situation, but we have no other way to arrive at knowledge about the world (reality). If you want to claim that we do have another way, namely God, you need to be very specific in explaining how you obtain knowledge of the world from God, while not using your sense perceptions, memory, or cognitive faculties, since that would be circular reasoning.

        This highlights the difference between saying we know something (provisionally), and saying that we KNOW something (absolutely). In a very technical (and pedantic) sense, even something as simple as saying that I knocked on a metal chair with my knuckle requires correctly functioning hearing, touch, and memory. We experience the world through our brain (or maybe I should say as a brain), but our brain cannot directly experience the world; it requires signals from the sensory organs. These electrochemical impulses set off other impulses which are presented to, and interpreted by, the brain as what we call sights, sounds, etc. And all of that assumes our memory is working properly. So in the end there are a very, very few things we can claim to KNOW (“I exist” is one), and everything else we only know.

        While our sense perceptions are how we experience the world, reason and rationality (which I’ll use interchangeably) are how we make sense of the world. We put together senses and memories, and apply to them inferential practices like spotting patterns and noting consistencies–inducing, deducing, abducing, etc. Applying our inferential practices to our sense perceptions and memory, we form a model of the external world; we make a “map of reality,” which we update as we go along. We accept that “the map is not the terrain,” but we also realize that it seems to be useful in making predictions about the world. When our predictions start to fail, we adjust the map.

        In the example about the metal chair, we dug into my belief about the chair until we reached some things I just had to accept as assumptions—that my senses of touch and hearing were sending (at least somewhat) reliable signals to my brain, and that I could remember them somewhat reliably. But there were other assumptions I was making, as well:

        – I am a self, an agent (this entails that I’m not a BIV)
        – There is a mind-independent reality
        – Deduction, induction, and causation obtain
        – The reality of the past (not all philosophers agree on this one, reject it if you like)
        – The concepts of truth and falsity (to me this is entailed in reality, since truth is reality)
        If I was running through my justifications with another person, I was also assuming
        – The existence of other minds

        These are some of the basic assumptions we make, which we cannot justify, but instead we take for granted. We cannot justify them because we cannot reach any conclusions without using our sense perceptions, memory, and rationality, and these are the very things we’re trying to justify. While we cannot justifiably claim that our senses and cognitive faculties are True and Inerrant, there’s also no principled reason to doubt their general reliability; there’s no reason to think there’s a systematic failure of our cognitive and inferential systems. There’s also no principled reason to suspect I’m being deceived, a la BIV, which is why I don’t worry about the possibility overmuch.

        These basal assumptions are universally held, and are irrevisable (that doesn’t mean they’re inerrant), but we don’t have a basis upon which to revise them that doesn’t use our cognitive faculties, which would be circular. These assumptions are what constitute rationality, and operate in every form of rationality. To deny them—to deny the general reliability of our senses and memory, or causation, or our own existence—are manifestations of irrationality. A person who denied these things would be considered insane.

        Note that while some of these assumptions are the result of some higher thinking, animals other than ourselves also act according to some of them without thinking about it or being formally taught. Many animals have at least a limited understanding of cause and effect. Other primates understand truth (well enough to be deceptive toward others). Many animals are able to use tools based on inductive inferences, etc. Some other primates and some cephalopods exhibit self-awareness.

        That should pretty much cover it, but I’ll answer a few direct questions and claims, just to be sure.

        You ask, “What ontic referent do you provide for reason? What objective standard of truth does atheism reason toward? Your standard? Richard Dawkins’ standard? Carl Sagan’s standard? Human standards of truth can only be subjective, not objective.

        I’m not sure you and/or Jinn are using “ontic referent” in the ‘standard’ way, but it seems you mean it as “objective standard of truth.” If not, I’ll need you to clarify your usage. The standard of truth is reality. The JTB version of knowledge, used overwhelmingly by Christians, defines truth as “that which comports with reality,” and that works for me. Given that understanding, your claim, “human standards of truth can only be subjective” is patently wrong. There isn’t even such a thing as a human standard of truth. There is only truth; the way things actually are. Human understanding or grasp of truth/reality can never be complete (or completely reliable), but this is not the same as being subjective. Additionally, even if there is some being with access to complete truth, and that being decides to share this truth with humans, we’re still limited by our imperfect sensory perceptions, memory, and cognitive faculties, such that our grasp of truth could still never be complete.

        “Non Credenti, for the THIRD TIME: Theism grounds reason in the reason of God’s mind.” You say this as if you’re impressed with yourself, but I’ve already told you this is so vague it’s meaningless. In philosophy (this is a discussion of epistemology) there are at least three ways to use the term grounding. Which of those are you using, or are you using a fourth? Repeating an informationally-vacant phrase three times doesn’t imbue it with any more information. Tell me specifically what you mean by this. If you manage that, your next task will be to tell me how you can be any more sure of your conclusions than the atheist, who is forced to stumble around with nothing but his best perception of reality as a guide. If you manage that, go back and make sure you didn’t use any of your powers of reason to make your arguments, since that’s what you’re trying to justify.

        I think I hit your main points, but I’ll go over some more specific claims of yours in a follow-up.

        • Scott Youngren says:

          Non Credenti,

          I have bolded your comments and replied below:

          These assumptions are what constitute rationality, and operate in every form of rationality. To deny them—to deny the general reliability of our senses and memory, or causation, or our own existence—are manifestations of irrationality. A person who denied these things would be considered insane.

          Yes, these are some assumptions that help us make sense of reality. I agree with most of your epistemology, at least at first glance. But I am not sure what the relevance of this is. Are you trying to create a diversion?

          In order to be coherent, a worldview must explain the various facets of the world in which we live. For example, a worldview must explain why there exist such things as:

          1.The comprehensible rational order of the universe (as opposed to chaos)

          2.Moral truth

          3.Beauty

          4.Conscious agents such as ourselves.

          5.Abstract concepts

          6.Purposes

          There are assumptions which we need to make about the world which are not explainable if we assume that atheism is true. Atheism is grounded in materialism, which says that only matter and/or energy exist, and that there are no immaterial conscious agents. If we make this assumption, it is very difficult to explain, for example, why there is a rational order to the universe rather than just chaos. Why do material things so consistently follow laws (such as the laws of physics and thermodynamics)?

          Regarding this point, Albert Einstein wrote:

          “You find it strange that I consider the comprehensibility of the world (to the extent that we are authorized to speak of such a comprehensibility) as a miracle or as an eternal mystery. Well, a priori, one should expect a chaotic world, which cannot be grasped by the mind in any way… the kind of order created by Newton’s theory of gravitation, for example, is wholly different. Even if man proposes the axioms of the theory, the success of such a project presupposes a high degree of ordering of the objective world, and this could not be expecteda priori. That is the ‘miracle’ which is constantly reinforced as our knowledge expands.”

          The theistic explanation for the orderliness of the universe, for why material things so consistently follow laws (or regularities, if you prefer) is simple: The same mind which creates matter also governs it.

          As Robert Boyle, the founder of modern chemistry, put it: “The nature of this or that body is but the law of God prescribed to it [and] to speak properly, a law [is] but a notional rule of acting according to the declared will of a superior.” [italics added]

          Or, as James Joule, the propounder of the first law of thermodynamics, for whom the thermal unit of the “Joule” was named, put it: “It is evident that an acquaintance with natural laws means no less than an acquaintance with the mind of God therein expressed.”

          I am not trying to be rude (yelling at you) by using all caps in the following question. Rather, I am using all caps since I have noted how previous atheist commenters have tried to ignore the following question because it cannot be coherently explained from within the framework of the atheist worldview. (Please accept my future usage of all caps as a polite attempt to highlight an important point or question which cannot be ignored or dismissed if we are going to have a constructive discussion):

          WHY DO MATTER AND ENERGY SO CONSISTENTLY FOLLOW LAWS (or “regularities,” or whatever term you prefer)? Please do not try to dismiss or ignore this crucial question.

          The rise of science happened in Christian Europe despite the fact that it failed to take root in a multitude of other societies. It was the Christian objective grounding of reason in God’s mind that provided the groundwork for the rise of science. Nancy Pearcey comments on this point:

          As historian Edward Grant writes, “It is indisputable that modern science emerged in the seventeenth century in Western Europe and nowhere else.”

          This fact is certainly suggestive, and it has prompted scholars to ask why it is that modern science emerged only out of medieval Europe. Sociologist of religion Rodney Stark identified the 52 figures who made the most significant contributions to the scientific revolution, then researched biographical sources to discover their religious views. He found that among the top contributors to science, surprisingly only two were skeptics (Paracelsus and Edmund Halley).

          …This insight into polytheism goes back to Isaac Newton, who once argued that the basis for believing there can be universal laws of nature is monotheism, since it implies that all of nature reflects the creative activity of a single Mind. Newton was arguing against the Greek notion, still prevalent in his day, that the earth was a place of change and corruption, whereas the heavily bodies were perfect and incorruptible. Against that view, Newton believed that both were products of a single divine Mind and therefore both were subject to the same laws. This opened the way for his breakthrough concept of gravity–the then-revolutionary idea that the same force that explains why apples fall to the ground also explains the orbits of the planets.

          More recently a similar argument was made by the Nobel Prize-winning biochemist Melvin Calvin. Speaking about the conviction that the universe has a rational order, he says, “As I try to discern the origin of that conviction, I seem to find it in a basic notion . . . enunciated first in the Western world by the ancient Hebrews: namely, that the universe is governed by a single God,and is not the product of the whims of many gods, each governing his own province according to his own laws. This monotheistic view seems to be the historical foundation for modern science.”

          The insurmountable problem for atheism is that it provides no grounding for reason. Please recall that atheism is grounded in materialism, which says that all that exists is mindless matter and/or energy, and that there is no such thing as immaterial consciousness.

          The materialist/atheist worldview’s explanatory resources are limited to matter and/or energy, and mindless natural processes. Put another way, atheism must explain such things as reason, truth, consciousness, abstract concepts, etc. in terms of mindless matter and/or energy and mindless natural phenomena.

          You, for example, you have tried to explain human beliefs (truth claims) in terms of brain physiology. But brain physiology is a natural phenomena, and natural phenomena can neither be true nor false. If human thought is nothing but chemical and electrical signals in the brain, then human beliefs can neither be true nor false. One chemical/electrical signal in the neurons of the brain cannot be more true or false than another such signal….anymore than one toothache can be more or less true than any other toothache (to cite the example given by the physicists Stephen Barr in my previous citation). I will copy and paste some elements of my previous comment since YOU HAVE CONVENIENTLY FAILED TO RESPOND:

          University of Delaware physicist Stephen Barr echoes Nancy Pearcey’s and physicist Amit Goswami’s (cited in my other reply) points about the contradiction between the materialist/naturalist worldview (in which atheism is grounded) and the existence of ideas, or abstract concepts, in Modern Physics and Ancient Faith:

          “Cognitive scientists talk about neurons, for example. But ‘neuron’ itself is an abstract concept that arose from the researches of biologists. For the materialist, then, even this concept of ‘neuron’ is nothing but a neurological creation; it also is a pattern of neurons firing in someone’s brain. If this sounds like a vicious circle, it is. We explain certain biological phenomena using the abstract concept ‘neuron,’ and then we proceed to explain the abstract concept ‘neuron’ as a biological phenomenon—indeed, a biological phenomenon produced by the activity of neurons. What we are observing here is the snake eating its own tail, or rather its own head. The very theory which says that theories are neurons firing is itself naught but neurons firing.”

          “…Why should anyone believe the materialist, then? If ideas are just patterns of nerve impulses, then how can one say that any idea (including the idea of materialism itself) is superior to any other? One pattern of nerve impulses cannot be truer or less true than any other pattern, any more than a toothache can be truer or less true than another toothache.”

          Indeed, as Barr alludes to above, the existence of truth (and abstract concepts, etc.) cannot be explained through the lens of the materialist/naturalist worldview. If true, the materialist belief that human consciousness is nothing but the firing of neurons in the brain cannot be anything other than…..the firing of neurons in the brain. How could a true neuronal impulse be distinguished from a false one? By measuring the voltage of the impulse? Neuronal impulses and brain physiology are neither true nor false. Rather, they are merely natural phenomena just like rocks.

          Atheism is caught in a self-defeating incoherence.

          And, much as it is incoherent to try to explain consciousness in mindless material terms (as I have demonstrated above), it is impossible to explain the existence of purposes in material terms. You tried to explain the evolution of Mary’s beliefs in material terms (brain physiology), and I responded, but YOU CONVENIENTLY IGNORED MY RESPONSE. Evolution depends on survival of the fittest, but survival is a purpose. Inanimate material things do not have purposes. Please recall my citation of physicist Amit Goswami, which you conveniently ignored.

          Physicist Amit Goswami explains why suggesting that inanimate material things can have purposes (such as survival…let alone navigating complex social structures) is absurd:

          The Darwinian theory of evolution is based on natural selection: Nature selects those organisms that are fittest to survive. In the materialist view, an organism is just a bundle of molecules that are completely specified by their physical and chemical properties. Nowhere among these properties will you find a property called survivability. No piece of inanimate matter has ever attempted to survive or in any way tried to maintain its integrity under any circumstances. But living bodies do exhibit a property called survivability. Now the paradox. A Darwinist would say that the survivability of the living form comes from evolutionary adaptation via natural selection. But natural selection itself depends on survival of the fittest.

          See the circularity of the argument? Survival depends on evolution, but evolution depends on survival! A paradox is a sure-fire sign that the basic assumptions of the paradigm are incomplete or inconsistent; they need a reexamination. Aren’t biologists missing something by objectifying subjective phenomena? The philosopher Robert Efron (1968) thinks so:

          The reductionist attacks the definition and usage of every word, which has historically referred to an action of a living entity: “memory,” “reflex,” “free will,” “cognition,” and so forth. He then redefines the same word so that it will be applicable to an action of an inanimate entity. By using this epistemological technique he deludes himself into thinking that inanimate entities have the same properties found in living organisms, that a common denominator has been found, and that the problem of reduction has been “solved.” The solution is primitive animism expressed in scientific jargon.

          Atheism must also ignore the insights of modern physics, which clearly demonstrate that the material world is a construct of consciousness (as I illustrate in God Is Real…Why Modern Physics Has Discredited Atheism. And atheism must also ignore the insights of modern biology, which demonstrate that

          1.DNA is a language in very literal (not metaphorical) sense. Only minds create languages. Period. End of story.
          2.Evolution happens as a result of directed (as opposed to random) processes.

          ….as I illustrate in There’s Nothing Random About Evolution.

          There isn’t even such a thing as a human standard of truth. There is only truth; the way things actually are. Human understanding or grasp of truth/reality can never be complete (or completely reliable), but this is not the same as being subjective.

          Yes, as you put it, “There is only truth, the way things actually are.” And you are certainly correct that human understanding of truth/reality can never be completely reliable. But what we are discussing here is the ability of human reason to access this truth.

          How can we access this objective truth if our thoughts are nothing but chemical and electrical impulses in the brain? How can one chemical/electrical impulse be more true than another chemical/electrical impulse…anymore than one toothache can be more or less true than another toothache (as the physicist Stephen Barr put it in Modern Physics and Ancient Faith)?

          You are correct that there is no human standard of truth. And, as Bo Jinn put it, “Truth/ value judgments can be supported by empirical facts, but at the very last instance they will always require a judgment from a personal agent. And unless that judgment is made on the basis of an objective standard of truth, then the judgment is therefore meaningless. …If human minds are the only personal agents in existence, it follows that all truth judgments based on reason are completely relative.”

          Unless you cite some objective standard of truth which exists above and beyond subjective human standards of truth, all of your reasoning is completely subjective, and therefore meaningless. This of course includes your attacks on the objective standard of truth that I cite (God)…such as your arguments about how a Christian cannot trust God to be truthful.

          “Non Credenti, for the THIRD TIME: Theism grounds reason in the reason of God’s mind.” You say this as if you’re impressed with yourself, but I’ve already told you this is so vague it’s meaningless. In philosophy (this is a discussion of epistemology) there are at least three ways to use the term grounding. Which of those are you using, or are you using a fourth? Repeating an informationally-vacant phrase three times doesn’t imbue it with any more information. Tell me specifically what you mean by this. If you manage that, your next task will be to tell me how you can be any more sure of your conclusions than the atheist, who is forced to stumble around with nothing but his best perception of reality as a guide. If you manage that, go back and make sure you didn’t use any of your powers of reason to make your arguments, since that’s what you’re trying to justify.

          Specifically, I am asking why there is a rational order to the universe, and why this rational order is comprehensible to us. A copy and paste of my above citation of Albert Einstein:

          “You find it strange that I consider the comprehensibility of the world (to the extent that we are authorized to speak of such a comprehensibility) as a miracle or as an eternal mystery. Well, a priori, one should expect a chaotic world, which cannot be grasped by the mind in any way… the kind of order created by Newton’s theory of gravitation, for example, is wholly different. Even if man proposes the axioms of the theory, the success of such a project presupposes a high degree of ordering of the objective world, and this could not be expected a priori. That is the ‘miracle’ which is constantly reinforced as our knowledge expands.”


          I said your attack failed because it attacks an incorrect notion of evolution (ironically, a straw man), hinging its argument on the false idea that NS acts on ideas rather than genes.

          Since we can follow the development of distinct regions of the brain, from the lizard R-complex, to the mammalian limbic system (along with some lizards), to the neo-cortex (shared with other primates and cetaceans), and have identified their functions, we can identify which areas of the brain—and which connections between different areas of the brain—are responsible for various higher mental functions. This isn’t the result of some thought experiments; scientists are working with actual brains, while you’re relying on the erroneous idea that NS acts on ideas. Once one understands that NS acts on genes, rather than ideas, the werewolf dilemma evaporates.

          Non Credenti, you need to respond to my previous reply instead of conveniently ignoring it:

          Genes and brain physiology are natural phenomena, and natural phenomena can neither be true nor false. The rock I see laying on the ground across the street is neither true nor false. Rather, it is merely a natural phenomenon.

          Regarding this point, Nancy Pearcey writes in her book Finding Truth:

          Materialism reduces thinking to biochemical processes in the brain, akin to the chemical reactions in digestion. But digestion is not something that can be true or false. It is just a biological fact. If thinking is reduced to brain processes, then our ideas are not true or false either. But in that case, how can the materialist know that materialism is true? The philosophy is self-refuting.

          As philosopher Charles Taylor puts it, the atheist materialist functions as though he were an “angelic observer” somehow able to float above the determinist cage in which he locks everyone else.

          Please refer to my above citation of physicist Amit Goswami as to why the idea that inanimate material things can have purposes such as survival is absurd.

          Sorry, but again the confusion is yours. If I find problems with an argument, I’m free to attack it without being required to present a better one—or, indeed, any alternative at all. For over a thousand years people noticed the retrograde motion of Mars, and pointed out the problems it caused to the geocentric, spherical orbit theory of astronomy. Do you imagine they were required to provide theories of their own before they could criticize one with obvious flaws?

          Note I’m not saying that on the claim of Divine grounding it’s impossible for the Christian to access reason/rationality/truth, just that it’s impossible to be sure he has access to them. He may feel he has access tosomething, but he can’t ever know that God is not lying to him, or that Satan is not impersonating God, or that he isn’t hallucinating the entire experience. By admitting that you can’t know this, you’ve refuted the (b) part of your argument and admitted that you’re in the same boat as the atheist.

          This is a perfect example of how you attack my truth grounding, but fail to provide your own grounding for truth. Since you do not provide any objective standard of truth to reason towards, even your attack on my truth grounding is completely relative, and therefore meaningless.

          Is truth grounded in brain physiology? HOW CAN BRAIN PHYSIOLOGY AND CHEMICAL/ELECTRICAL SIGNALS IN THE BRIAN BE EITHER TRUE OR FALSE?

          The point can be summed up thusly: Without using your reason, demonstrate that God is rational… or that He has made rationality available to you… or that he isn’t deceiving you with false senses or memories. It cannot be done, and that is true of anyone who attempts to justify his reason.

          Again, you attack my grounding for truth. But since you do not provide your own grounding for truth (your own objective standard of truth to reason towards), your very attack on my objective grounding for truth is a completely subjective truth claim. It is therefore meaningless.

          Further, since you have reduced human beliefs and truth claims to brain physiology, you have completely decimated your ability to ground truth. Once again, material phenomena such as brain physiology can be neither true nor false.

          Again, atheism does a wonderful job of destroying our ability to trust our reasoning faculties, but it provides us no reason to trust our reason….such as the reasoning of its own truth claims.

          You again confuse attacking my grounding of truth with providing your own grounding of truth. Your attack on my truth claims has no grounding in objective truth….and is therefore an objectively meaningless subjective truth judgement.

          But the problem is that atheism can never, even in principle, provide an objective standard of truth to reason towards. Under atheism, one can never make an objective truth claim because atheism does not provide an objective standard of truth with which to evaluate various truth claims. This is not a problem that could be fixed by some future discovery, as with your example from astronomy. Will we (some bright and shining day in the future) discover which human being provides the objective standard of truth?

          And that is the crux of the problem for your atheist “reasoning.” Under atheism, all truth judgments are completely relative, as Bo Jinn points out. You doubtlessly spent a whole lot of time attacking my reasoning. But since you have not provided any objective standard of truth to reason towards, you have furnished a completely relative truth claim, and you have therefore wasted all of your time and effort. Again, without an objective standard to reason towards, all of your reasoning is useless because it does not reference an objective standard of truth.

          Atheism is very good at destroying reason, but fails miserably when it comes to providing any objective standard of truth with which to validate ATHEISTIC ARGUMENTS THEMSELVES, including your attacks on my reasoning and my ontic referent for objective reason (God). A copy and paste again of that commentary by Bo Jinn:

          Scientific facts cannot justify reason. It is reason justifies science. But, then, what justifies reason? The reliability of reason, just as the existence of morality and beauty is simply taken for granted by the atheist on purely pragmatic grounds. There is no sufficient ontic referent for their actual existence. Truth/ value judgments can be supported by empirical facts, but at the very last instance they will always require a judgment from a personal agent. And unless that judgment is made on the basis of an objective standard of truth, then the judgment is therefore meaningless. By denying a sufficient reason for the existence of the universe, that objective standard of truth simply does not exist. It is lost in the nihilistic vacuum left in the wake of the death of God.

          If human minds are the only personal agents in existence, it follows that all truth judgments based on reason are completely relative…As we speak, there are atheists the world over insisting that atheism is a conclusion which intelligent people come to on the basis of reason. But, if atheism is true, then human reasoning has no validity at all, because valid reasoning implies a standard of truth that can be reasoned toward and a sufficient reason for believing that human reasoning works in the first place. That guarantee is non-existent in an atheist world. Since the universe is a self-evident brute fact which exists (like us) for no purpose in and of itself we, in effect, become our own standard of truth and reasoning is thus rendered completely invalid.

          Lastly, YOU YET AGAIN FAILED TO RESPOND TO THE POINTS I made about how modern physics and modern biology both point to a consciousness which is the prime reality (or the something-from-which-everything-else-comes). The theistic model is strongly supported by what we know about modern physics, as I demonstrate in God Is Real…Why Modern Physics Has Discredited Atheism, and with what we know about modern biology, as I demonstrate in There’s Nothing Random About Evolution.

          As I point out in the first of the above essays, in order to prevent their worldview from collapsing, atheists must ignore or deny the fact that modern physics has conclusively shown us that material reality is a construct of consciousness. A citation from the first of the above two essays:

          Physicist Richard Conn Henry explains why people (such as atheist biologists) cling to materialism/naturalism despite the fact that it has been completely discredited by modern physics:

          “Why do people cling with such ferocity to belief in a mind-independent reality? It is surely because if there is no such reality, then ultimately (as far as we can know) mind alone exists. And if mind is not a product of real matter, but rather is the creator of the illusion of material reality (which has, in fact, despite the materialists, been known to be the case since the discovery of quantum mechanics in 1925), then a theistic view of our existence becomes the only rational alternative to solipsism.” [“Solipsism” is defined as “the view or theory that the self is all that can be known to exist.”]

          And, in There’s Nothing Random About Evolution, I point out how modern biology has demonstrated that evolution is the result of directed (as opposed to random) evolutionary mechanisms.

          Again, I am very extremely curious to hear your rebuttal to my two above essays regarding how both physics and biology strongly support the theistic model.

          • Non Credenti says:

            I apologize for the long delay. There were a number of posts from you that I wanted to respond to, not including the above, which I didn’t even see, and I thought I would be able to respond before Christmas….

            You’ve raised a great many questions, but I’m going to rein in the discussion back to the original point of mine, made in early December. I’ll do so by way of a response to a point you make in the above post and others:

            [strong]This is a perfect example of how you attack my truth grounding, but fail to provide your own grounding for truth. Since you do not provide any objective standard of truth to reason towards, even your attack on my truth grounding is completely relative, and therefore meaningless.[/strong]

            OK, let’s break this down.
            1. You claimed as a Christian you have access to reason and truth, and you offered as a supporting argument the claim that your reason is grounded on God.;
            2. I ‘attacked your truth grounding’ in multiple posts, and on multiple fronts, [strong]all of which you need to be able to address. [/strong];
            3. Your immediate response, on December 8, was, naturally, an attempt to avoid addressing my attack by saying I confused expressing doubts about your grounding for reason with providing my own grounding for reason. “Attacking my attack” is a valid response. If you can show why my attack doesn’t get off the ground, then you’re not obligated to respond to it.
            4. My response, on December 11, was to say ‘If I find a problem with your argument, I’m free to attack it without being required to present an alternate argument.” I supported this logically with the argument that the people who for over 1,000 years criticized the geocentric theory of astronomy, based on the retrograde motion of Mars, were not required to provide better explanations before they were allowed to criticize a theory with obvious flaws. The (implied) conclusion was that, likewise, your claim of rational grounding has obvious flaws, and if I point out these flaws, I don’t need to provide a better theory. It’s [em]your[/em] claim, and you need to address flaws as they’re pointed out to you, rather than demand a better explanation from me. That’s what logical arguments are. If you cannot address the objections, then your argument fails. Period. So your effort to avoid the need to defend your claim failed, and the ball is once again in your court—you must defend your position.
            5. At this point, you have two options: try again to defeat my attack, by launching a [em]new[/em] argument against it (not the ‘you have to provide a theory of your own’ argument, but a different one, since I’ve rebutted that argument), or by rebutting my mars/retrograde argument, so that your initial rebuttal to my attack would stand. [If you wish to argue that critics of the geocentric theory were required to explain the orbit of Mars before they could criticize geocentrism, then present that logical argument.] Your response, on December 13, was to stick your fingers in your ears and repeat that if I attack your grounding for truth, I need to provide my own grounding for truth.

            Sorry, Scott. I understand that you’ve never studied or read a book on logic (or evolution, or information theory, or quantum mechanics, or naturalism, or any of the things on which you comment….), you’ve only learned to regurgitate quotes from professional apologists, and this puts you at a tremendous disadvantage, but this is the nature of logical arguments… once I have dealt with an objection, it does no good to keep repeating it. Your attempt to undermine my objections has failed, so now you must either find another way to undermine it, or address the objections. If you cannot, you always have the option of conceding that your argument fails. (A failed argument doesn’t prove that the claim is false.)

            To recap, my objections are:
            1. You’ve already disproved your own claims of having access to truth when you admitted you can’t even know you have access to reality. You can’t know you’re not a brain in a vat.
            2. Your mysterious grounding of truth on God’s truth (or grounding reason on God’s reason) is so vague as to be meaningless. How, [strong]specifically[/strong], is anything “grounded” on God? (With this objection I don’t aim to refute your claim; I am to show it’s not even necessary to refute it, since your claim is so ill-defined that it’s useless.);
            3. You’ve grounded your reason/truth on an admitted deceiver, thus poisoning your own well;
            4. How do you know God is rational?
            5. Given the imperfections of your sense perceptions, memory, and cognitive faculties, how could you ever know you were receiving a signal accurately, even if it was broadcasting?
            6. When this mysterious grounding delivers some “truth” to you, how do you know it’s actually from your claimed source, God?:
            a) How do you know it isn’t from Satan, impersonating God?;
            b) How do you know it isn’t the result of a chemical imbalance in the brain (maybe some bad mushrooms in the spaghetti)?;
            c) How do you know it isn’t the result of brain trauma—that you’re not hallucinating this revelation of truth?

            I think I’ll grant that 6b) and 6c) are similar enough to the BIV objection, that dealing with 1) should also deal with 6b) and 6c).

            To defend your claim of grounding, you must answer each of these objections.

            For completeness, I’ll answer your other questions, charges, and objections in the course of time, but since they all attack my ground of reason–which I’ve conceded from the outset I don’t have–they’re not relevant to [em]your[/em] ground of reason. And, yes, my objections to your claims of grounding [em]are[/em] the topic at hand. It’s right there in my original response to this article. If you wish to deny that, you’ll need to go back and remove my first post, then edit all of our further correspondence to hide that fact.

            • Scott Youngren says:

              Non-Credenti,

              Let’s lay out the options so as to make this clear:

              Option 1) Atheism is true.

              With this option, Non-Credenti has no grounding for reason (as he admits). Therefore, we have no reason to trust the rational faculties which he uses to criticize Scott’s position. Without reliable rational faculties, he cannot produce a reliably rational argument. His attack on Scott’s position fails.

              As Charles Darwin himself admitted, “[W]ith me the horrid doubt always arises whether the convictions of man’s mind, which has been developed from the mind of the lower animals, are of any value or at all trustworthy. Would anyone trust in the convictions of a monkey’s mind, if there are any convictions in such a mind?”

              Option 2) Theism is true, but Non-Credenti’s criticisms of Scott’s grounding for reason are accepted as accurate (God cannot be trusted to provide us with reliable rational faculties, Scott could be a brain in a vat, God might not be rational, etc…).

              With this option, Non-Credenti still has no grounding for reason. Therefore, we still have no reason to trust the rational faculties which he uses to criticize Scott’s position. Without reliable rational faculties, he cannot produce a reliably rational argument. His attack on Scott’s position fails.

              Option 3) Theism is true, and Non-Credenti’s criticisms of Scott’s grounding for reason are rejected as false (God can be trusted to provide us with reliable rational faculties).

              With this option, Non-Credenti is wrong, and his attack on Scott’s position fails.

              Non-Credenti, this is why you cannot merely attack my position without providing your own grounding for reason. Atheism cannot under any circumstances explain why our reasoning should be deemed reliable or why there is a rational structure to the universe. Without a reason to trust your reasoning, there is no reason for us to trust the reasoning you use in your attacks on the theistic position.

              You say, ”I supported this logically with the argument that the people who for over 1,000 years criticized the geocentric theory of astronomy, based on the retrograde motion of Mars, were not required to provide better explanations before they were allowed to criticize a theory with obvious flaws.”

              But the problem with atheism is that it relies on irrational processes to explain things. Therefore, atheism cannot, even in principle, explain why there is a rational structure to the universe or why we should trust our reasoning. No advance in science is going to change this. As Cambridge University astrophysicist John Polkinghorne writes, “Science does not explain the mathematical intelligibility of the physical world, for it is part of science’s founding faith that this is so.”

              Keith Ward, a member of the Council of the Royal Institute of Philosophy, writes in his book Doubting Dawkins: Why There Almost Certainly is a God:

              “…there is force in the classical philosophical axiom that, for a truly explanatory cause to be intelligible, it must contain its effects potentially in itself. As the classical philosophers put it, the cause must contain more reality than its effects.”

              The implication of this philosophical axiom is that irrational causes do not contain the effect of reason potentially in themselves. Impersonal causes do not contain the effect of personhood potentially in themselves. Unintelligent causes do not contain the effect of intelligence potentially in themselves. This is why the only logical option is to cite a rational, personal, and intelligent cause (read: God) for rational, personal, and intelligent agents such as ourselves.

              Edwar Feser skillfully elaborates on this point:

              The basic idea is that a cause cannot give to its effect what it does not have to give, and it can be illustrated by a simple example.

              Suppose you come across a puddle of water near an outdoor spigot. You will naturally conclude that the puddle was caused by the spigot, either because someone turned it on or because it is leaking. The effect is a puddle of water and the cause is something fully capable of producing that effect, since it contains water in it already. But now suppose instead that you come across a puddle of thick, sticky, dark red liquid near the same spigot. In this case you will not conclude that the spigot was the cause, at least not by itself.

              The reason is that there is nothing in the spigot alone that could produce this specific effect, or at least not every feature of the effect. The spigot could produce a puddle of liquid alright, and maybe even a puddle of vaguely reddish liquid if there was rust in the line, but not a puddle of thick, sticky, dark red liquid specifically. You would be likely to conclude instead that someone had spilled a can of soda pop near the spigot, or perhaps that someone had been bleeding heavily nearby it.

              Even if these possibilities had been ruled out and you had evidence that the puddle came from the spigot after all, you’d conclude that somehow such a thick red liquid (blood, soda, or whatever) had somehow been put into the water line, or that if it had not, then there must have been something on the ground that when mixed with water from the spigot chemically produced this thick red liquid. What you would never seriously consider is the suggestion that normal water from the spigot all by itself produced the red puddle. For there is just nothing in water by itself that could produce the redness, thickness, or stickiness of the puddle; ergo there must have been something in addition to the water that produced the effect.

              As this example illustrates, the effect might be “contained in” the cause in various ways. It could be that the cause was itself red, as blood or cherry soda pop is red even before it causes a red puddle. But it could also be that the cause was not itself red but had the power to generate redness in the effect; for example, neither the water nor some chemical substance spread on the ground (a ground-up “Fizzy” drink tablet, say) might be red, yet will produce a thick red liquid when combined. Or, to take another example, the cause of a fire might itself be on fire, as when a torch is used to start a brushfire, or it may instead have the power to produce fire, as a cigarette lighter has even when it is not being used.

              The traditional way of making this distinction is to say that a cause has the feature that it generates in the effect “formally” in the first sort of case (e.g. when both the cause and the effect are red or on fire) and “eminently” in the second sort of case (e.g. when the cause is not itself red or on fire but has an inherent power to produce redness or fire). If a cause didn’t contain all the features of its effect either formally or eminently, there would be no way to account for how the effect came about in just the way it did. Again, a cause cannot give to its effect what it does not have to give.

              Keith Ward also notes, “[There is] the basic axiom that being is intelligible and has a fully rational structure. If this is so, then belief in God is more, not less, rational than atheism, which can give no convincing reason why the universe should be intelligible, or why reason should be able to understand it.”

              Oxford University mathematician John Lennox delves into this topic in his book God’s Undertaker. Has Science Buried God?:

              However much we may debate the essence of the scientific method, there is no question as to the foundation on which that method rests: the rational intelligibility of the universe. It was Albert Einstein’s astonishment at this that prompted him to make the famous comment, “The most incomprehensible thing about the universe is that it is comprehensible.” The very concept of the intelligibility of the universe presupposes the existence of a rationality capable of recognizing that intelligibility. Indeed, confidence that our human mental processes possess some degree of reliability and are capable of giving us some information about the world is fundamental to any kind of study, not only the study of science. This conviction is so central to all thinking that we cannot even question its validity without assuming it in the first place, since we have to rely on our minds in order to do the questioning. It is the bedrock belief upon which all intellectual inquiry is built. I shall argue that theism gives it a consistent and reasonable justification whereas naturalism seems powerless to do so.

              Rational intelligibility is one of the main considerations that have led thinkers of all generations to conclude that the universe must itself be a product of intelligence. Philosopher Keith Ward sums up: “To the majority of those who have reflected deeply and written about the origin and nature of the universe, it has seemed that it points beyond itself to a source which is non-physical and of great intelligence and power. Almost all of the great classical philosophers – certainly Plato, Aristotle, Descartes, Leibniz, Spinoza, Kant, Hegel, Locke, Berkeley – saw the origin of the universe as lying in a transcendent reality.”

              For Albert Einstein the comprehensibility of the universe was something to be wondered at: “You find it strange that I consider the comprehensibility of the world (to the extent that we are authorized to speak of such a comprehensibility) as a miracle or as an eternal mystery. Well, a priori, one should expect a chaotic world, which cannot be grasped by the mind in any way… the kind of order created by Newton’s theory of gravitation, for example, is wholly different. Even if man proposes the axioms of the theory, the success of such a project presupposes a high degree of ordering of the objective world, and this could not be expected a priori. That is the `miracle’ which is being constantly reinforced as our knowledge expands.”

              For, as the example of Newton’s theory shows, it is not only the fact that the universe is intelligible which is amazing; it is the mathematical nature of that intelligibility which is remarkable. We tend to take the usefulness of mathematics as obvious because we are so used to it. But why? [Physicist] Paul Davies is among those not satisfied with the glib response of people who say that the fundamental laws of nature are mathematical simply because we define as fundamental those laws that are mathematical. One of the main reasons for his dissatisfaction is that much of the mathematics found to be successfully applicable “was worked out as an abstract exercise by pure mathematicians, long before it was applied to the real world. The original investigations were entirely unconnected with their eventual application.” It is very striking that the most abstract mathematical concepts that seem to be pure inventions of the human mind can turn out to be of vital importance for branches of science, with a vast range of practical applications.

              Davies here echoes a famous essay by Eugene Wigner, a Nobel Laureate in Physics, in which he wrote:

              “The enormous usefulness of mathematics in the natural sciences is something bordering on the mysterious, and there is no rational explanation for it… it is an article of faith.” The relationship between mathematics and physics goes very deep and it is very hard to think of it as some random accident. Professor of Mathematics Sir Roger Penrose FRS, whose understanding of that relationship is unquestioned, has this to say about it: “It is hard for me to believe… that such SUPERB theories could have arisen merely by some random natural selection of ideas leaving only the good ones as survivors. The good ones are simply much too good to be the survivors of ideas that have arisen in a random way. There must be, instead, some deep underlying reason for the accord between mathematics and physics.” Certainly science itself cannot account for this phenomenon. Why? Because, in the words of John Polkinghorne: “Science does not explain the mathematical intelligibility of the physical world, for it is part of science’s founding faith that this is so.”

              Keith Ward strongly supports this view: “The continuing conformity of physical particles to precise mathematical relationships is something that is much more likely to exist if there is an ordering cosmic mathematician who sets up the correlation in the requisite way. The existence of laws of physics… strongly implies that there is a God who formulates such laws and ensures that the physical realm conforms to them.”

              Theism, therefore, upholds and makes sense of the rational intelligibility of the universe; whereas…the reductionist thesis undermines it and dissolves it into meaninglessness. Far from science abolishing God, it would seem that there is a substantial case for asserting that it is the existence of a Creator that gives to science its fundamental intellectual justification.

              Non-Credenti, I would strongly advise against using rhetorical language such as, “I understand that you’ve never studied or read a book on logic (or evolution, or information theory, or quantum mechanics, or naturalism, or any of the things on which you comment….), you’ve only learned to regurgitate quotes from professional apologists.”

              Such personal attacks as this only serve to highlight the emotional, psychological, and ideological (rather than rational) motivations for your atheism. Your stooping to personal attacks amounts to a “tell” that you are angry because you realize that your argument has been defeated….much as a nervous tick made by a poker player is a “tell” that he is holding a weak hand and is bluffing.

              • Non Credenti says:

                Wow, I’d love to play poker with you.

                Anyway, you seem to be making two main arguments: Your second point—that atheism cannot explain the order of the universe (while theism can)—doesn’t seem to have anything to do with the epistemological issue of how Christians claim to be able to ground reason, which is the claim I launched all of my attacks against, so I’m going to set it aside as irrelevant unless you can tell me how it pertains. (For example, what if I agree that the order of the universe is a mystery? I don’t see how that gains you any epistemological points. It seems one could grant that argument and you would still be left to demonstrate your claims of grounding, which is all I’m focusing on at the moment.)

                Your first point—that some of my attacks fail because I can’t launch a rational criticism, since irrational processes cannot produce rational processes—does pertain to the discussion. You present three options for how the larger conversation could conclude, and argue that I lose under any outcome, since under each option I have no grounding for reason. You claim I ‘have no reason to trust the rational faculties I use to criticize your position. Without reliable rational faculties, I cannot produce a reliably rational argument.’ This is because, according to you, irrational processes cannot produce rational processes.

                Firstly, even if you defeated my explanations of rationality, it wouldn’t mean your argument succeeds. At some point you’re going to need to present an argument or evidence in favor of your claim that you have some special access to reason, not just attack mine. This is especially so since I’ve never claimed to have the justification… I can only accept it as is. I would suggest starting with defining what you mean by “grounding” since your claim is meaningless without it.

                Secondly, you don’t claim my arguments are irrational–you simply don’t want to answer them so you want to stop me from launching the attack to begin with. That won’t work. You tried to give an argument for a claim you’ve made and that argument has been challenged–the source of the challenge doesn’t matter if the challenge itself is rational. The fact is that my arguments are rational, and they do damage your claim. So what if they’re the product of totally random thoughts flitting around my head like little butterflies? So what if I threw a handful of plastic letters in the air and by purest chance they spelled out various attacks on your claim? Accidental or not, they’re valid challenges, and your claim fails if you cannot address them.

                The rest of my argument isn’t even necessary, but for completeness, I’ll continue. To support your contention, you offer a number of quotes:
                -Darwin’s “horrid doubt,” but that’s useless as support, since quoting another person’s opinions is not a compelling argument.

                -Ward’s “axiom of potential cause” quote, which claims a cause “must contain its effects potentially in itself.” The words you left out of the quote are ”But I do think there is force… “ and I’m not sure there is any reason to think his opinion has force. It seems to me the classical philosophers were considering proximal, or immediate causes, not distal causes, and if so, it doesn’t seem to apply to this argument. The workings of the mind are still a mystery in many respects, but one thing there is agreement on is that the production of thoughts is the result of many different parts of the brain acting in together–and sometimes against each other. Nobody thinks, “This specific neuron fired, producing this specific thought.” Even assuming you were targeting a more directly proximal cause, it still isn’t the victory you think it is. You still have all of your work ahead of you… you need to demonstrate that non-rational processes do not even have the potential for rational thought. Sorry, but you don’t just get to proclaim that there is no potential for rationality in neurons. You need to demonstrate it.

                Your Feser “water spigot” quote does a good job of laying out the argument, but it, too, fails to demonstrate that neurons lack the potential to produce rational thoughts. It only argues for a certain conclusion if it can be shown that neurons don’t have this potential. But again, you need to demonstrate that.

                Your “epistemology 101” quote of Lennox is also useless as support. It lays out the epistemological position I explained earlier. You claimed not to understand the purpose of my explanation when I spelled it out for you, wondering aloud if it was only a diversion. Am I safe in assuming you don’t think Lennox was only creating a diversion? Regardless, he sets the table, claiming that he will argue in favor of theism—and against atheism—as a “consistent and reasonable justification,” and you present that table-setting as if it were an actual argument. You haven’t presented an argument, only an unsupported claim, and a quote from Lennox making the same unsupported claim.

                The rest of the quotes are addressing the “orderliness of the universe” argument which, as I’ve said, don’t seem to apply to the epistemological issue at hand, so there’s no need to address them unless you can show that they apply. Also, a number of the quotes are nothing but someone’s opinion, and would be useless even if you could satisfy the issue of relevance to epistemology.

                So ignoring the quotes that fail to provide any support for your argument, does the argument itself stand? You say “atheism cannot under any circumstances explain why our reasoning should be deemed reliable…” As I see it, there are two issues here. 1) How reliable does our reasoning need to be? Perfectly reasonable, or only provisionally reasonable? And; 2) Can we explain why our reasoning is reliable?

                We both agreed earlier that it is not reasonable to expect Absolutely Reliable reasoning. We’re not claiming perfection on either atheism or theism. Provisional reliability is the only reasonable expectation we can have.

                Regarding (1) I addressed this earlier and said, “These assumptions are what constitute rationality… To deny them [our base assumptions]… are manifestations of irrationality. A person who denied these things would be considered insane.” Your response, on December 13, was, “Yes, these are some assumptions that help us make sense of reality. I agree with most of your epistemology, at least at first glance…” So we both agree that the answer to (1) is ‘Yes, our reasoning is somewhat (provisionally) reliable, but not perfectly so, and we don’t require it to be.’

                At times you seem to be trying to advance the claim that, on atheism, even a provisional reliability of our reasoning abilities is not possible “under any circumstances.” If so, then that’s an argument you need to support. Recognizing that our reasoning is not perfectly Capital-R Reliable does not mean it’s not provisionaly reliable. Despite the lack of perfection in our senses, memory, or reasoning abilities, we seem to be able to make a somewhat accurate map of reality, one that enables us to make predictions about the world, and to get through life. We know it’s not perfectly reliable, but it doesn’t have to be. We don’t know how reliable it is, but we don’t have to know. “Under any circumstances” is a very low bar and is easily hurdled—all one need do is posit that a somewhat accurate map of reality has more survival benefit than a map that is not at all accurate. The next step is to posit that cognitive mechanisms which enable us to build a somewhat accurate map would have more survival benefit than ones in which the brain follows utterly random thoughts that are only accidentally—and rarely—correct. Done.

                2) Can we explain why our reasoning is (provisionally) reliable? Nope, not in the strictest sense, on pain of circularity. I can’t, and you can’t. We can only accept it as the apparent state of affairs. We can’t because it isn’t possible to do so without reasoning to the conclusion, which assumes valid reasoning—the very thing we’re trying to establish. It’s a circular argument and thus invalid. You might want to claim reason originates with God somehow, but that’s not an explanation, it’s just a vacuous claim–it holds no more force than if I claim an invisible Magic Sandwich oozes rationality into my brain. You’re welcome to try to justify it, but good luck doing so without using the very reason you’re trying to justify.

                Therefore, your attack on my ability to attack your claim has failed. You have tried to categorize the firings of individual neurons as “irrational,” but even accepting the poor choice of words, you have not shown that these individual neurons “cannot” produce rational thoughts. You haven’t shown that individual neural processes lack the “potential” for producing rational thought. (On the other hand, fMRI and other brain scanning technologies, as well as observations made after some brain traumas or surgeries, indicate that rational thoughts are the product of multiple regions of the brain acting in concert.) A single person would not seem to have the potential to lift 5,000 lbs with only a rope, but a group of people acting together can.

                More importantly, it doesn’t matter if, on atheism, I can’t claim access to reason. The fact is that I’ve launched rational arguments against your claim and—however I arrived them—your claim fails if you cannot address them.

                Let me recap by listing my attacks on your claim of a Divine Ground of Reason, and how your defenses and counter-attacks have fared:

                1: “Undefined Claim”: What is “grounding”? How is reason “grounded on” God’s reason?
                A claim using meaningless terms is meaningless.
                2: “BIV Admission”: You’ve already conceded your epistemological limitations when you admitted you couldn’t know you aren’t a brain in a vat. How can someone who can’t even say for sure they’re not a BIV claim to have access to a source of truth/reason?
                3: “Deceptive Source”: You poisoned your own well by claiming an admitted deceiver as Source.
                4: “Rationality of Source”: You must justify your claim that God is Rational.
                5: “Faulty Receiver”: Regardless of the source, your receiver (senses, memory, cognitive abilities) is known to be unreliable at times.
                6: “Source Verification-1”: “Divine Revelations” could be Satan impersonating God.
                7: “Source Verification-2”: “Diving Revelations” could be brain trauma and/or brain chemistry abnormalities.

                You need to address all of these challenges. So far, you’ve addressed none of them successfully.

                • Darwin’s “horrid doubt,” but that’s useless as support, since quoting another person’s opinions is not a compelling argument.

                  I’ve always wondered at Christians’ eagerness to quote Darwin (either in or out of context). It’s like they imagine that secularists are bound by what the great man said.

                  Everyone knows that it’s actually Richard Dawkins who is the Pope of atheists.

                  you need to demonstrate that non-rational processes do not even have the potential for rational thought. Sorry, but you don’t just get to proclaim that there is no potential for rationality in neurons. You need to demonstrate it.

                  Hitting a baseball seems a good example. You practice and gradually get better. Eventually, you think, “I hit that ball into left field,” and, sure enough, you are objectively correct. And with an imperfect human brain, too—whaddya know?

                  • Scott Youngren says:

                    Non-Credenti,

                    I have encased your comments with dashes, and then responded below:

                    ——At some point you’re going to need to present an argument or evidence in favor of your claim that you have some special access to reason, not just attack mine. This is especially so since I’ve never claimed to have the justification… I can only accept it as is. I would suggest starting with defining what you mean by “grounding” since your claim is meaningless without it.———-

                    ——–Secondly, you don’t claim my arguments are irrational–you simply don’t want to answer them so you want to stop me from launching the attack to begin with.——

                    I do not make any claim that I have special access to reason. This is a straw-man argument. Rather, I argue that there is no way to ground matter in reason. Non-Credenti, the difficulty with grounding matter in reason is very easy to see. Matter does not belong to a category of things which can be said to have truth value.
                    Material/electrical things like neurons and nerve impulses can be neither true nor false, just as the chair I am sitting in cannot be true or false. Therefore, grounding reason in a matter (and material things like neurons) is a category error. University of Delaware physicist Stephen Barr elaborates in Modern Physics and Ancient Faith:

                    “Cognitive scientists talk about neurons, for example. But ‘neuron’ itself is an abstract concept that arose from the researches of biologists. For the materialist, then, even this concept of ‘neuron’ is nothing but a neurological creation; it also is a pattern of neurons firing in someone’s brain. If this sounds like a vicious circle, it is. We explain certain biological phenomena using the abstract concept ‘neuron,’ and then we proceed to explain the abstract concept ‘neuron’ as a biological phenomenon—indeed, a biological phenomenon produced by the activity of neurons. What we are observing here is the snake eating its own tail, or rather its own head. The very theory which says that theories are neurons firing is itself naught but neurons firing.”

                    “…Why should anyone believe the materialist, then? If ideas are just patterns of nerve impulses, then how can one say that any idea (including the idea of materialism itself) is superior to any other? One pattern of nerve impulses cannot be truer or less true than any other pattern, any more than a toothache can be truer or less true than another toothache.”

                    Again, grounding reason in matter is an open-and-shut category error. Please answer the question that Barr alludes to above: How can one pattern of neuronal impulses be more true or less true than another pattern of neuronal impulses?

                    A conscious and intelligent mind (such as God’s mind), however, does belong to a category of things which can be said to posses the attribute of reason. I am using the following Dictionary.com definition of reason: “The mental powers concerned with forming conclusions, judgments, or inferences.” God’s mind CAN be the grounding for reason because a conscious and intelligent mind belongs to a category of things which can be said to have mental powers of forming conclusions, judgements, or inferences.

                    ——The fact is that my arguments are rational, and they do damage your claim. So what if they’re the product of totally random thoughts flitting around my head like little butterflies? So what if I threw a handful of plastic letters in the air and by purest chance they spelled out various attacks on your claim? Accidental or not, they’re valid challenges, and your claim fails if you cannot address them.———-

                    Your arguments damage my claim? But your arguments are the chemical and electrical impulses of neurons, aren’t they? How can a pattern of nerve impulses be true, and therefore damage my claims? This goes back to the citation I provided from Nancy Pearcey and Charles Colson, earlier in our debate:

                    “Materialism reduces thinking to biochemical processes in the brain, akin to the chemical reactions in digestion. But digestion is not something that can be true or false. It is just a biological fact. If thinking is reduced to brain processes, then our ideas are not true or false either.”

                    If your arguments are correct, then your “arguments” are a bunch of neuronal impulses in your brain. But how can neuronal impulses said to be either true or false? Is the chair you are sitting in true or false? Again, you are committing an open-and-shut category error.

                    –———You need to demonstrate that non-rational processes do not even have the potential for rational thought. Sorry, but you don’t just get to proclaim that there is no potential for rationality in neurons. You need to demonstrate it.———–

                    No, you need to demonstrate that non-rational processes (such as patterns of nerve impulses) have the potential for rational thought. Trying to shift the burden of proof is an attempt at sleight-of-hand that just falls flat, and you are committing the logical fallacy of assuming what you intend to prove. Albrecht Moritz comments:

                    Certainly the naturalist might still claim that evolution has endowed the human brain with basic and universal logical circuitry, shaped by its survival value, that reliably can decide “if we just give the issues some thought”. However, even if evolution could accomplish the creation of reliable logical circuitry (which is debatable), an informed decision for or against naturalism [or atheism] is not solely a matter of simple and straightforward logic based on premises that should be self-evident to everyone, independent of the angle from which they are looked at. Rather, when the issues are thoroughly studied and well thought through, it is a matter of careful weighing of (giving weight to) and interpreting abstract and rather complex evidence and arguments pro and con, and this goes far beyond basic circuitry that might have been induced by evolution for its survival value. So there cannot be an evolutionary ‘ought’ on this issue after all.

                    Thus, given all the above, it cannot rationally be claimed that evolution has shaped our brain circuits in such a way that they are bound to reliably settle the particular question at hand, “when engaged properly”. This, however, would be the only way to guarantee the right outcome under naturalistic determinism. (For theists there is no problem here; they usually view the brain as an integral part of the mind, on which the mind fully depends for its functioning, but they do not view it as identical to the mind – evolution then does not fulfill as ultimate a role in shaping the functioning of the mind as it does for the naturalist.)

                    How then can the naturalist nonetheless assert that naturalism is true and its acceptance rational? S/he considered the evidence, s/he will reply. Yet under naturalism the brain determines how to interpret the evidence – you have no say in that. So the naturalist’s brain determined that naturalism is true, and mine determined, considering the evidence as well, that naturalism is not true. Now, which brain is right? If the naturalist’s acceptance of naturalism is solely dependent on the firing of his/her neurons over which s/he has no control (under determinism), then it is not possible for him/her to know that his/her brain is right and that naturalism is true. Thus under naturalism the claim that naturalism is true becomes incoherent and self-contradictory. Naturalism defeats itself.

                    This might be countered with the following: our brain knows from experience that this or that kind of rational judgment will yield good practical results, and this experience is a sufficient judge for our brain to know that a related thought process is rational, even if we ourselves do not perform the judgment freely, but our brain does so in a deterministic way. Yet this argument is inadequate: also in areas where we do not have any prior experience we can know that, given the premises we work with, certain rational judgments are true even before we see verifiable results from them – since this also holds for complex and abstract thought processes, an explanation by basic circuitry induced by evolution for its survival value falls flat. While we are aware of this phenomenon from our own thought processes, it is exemplified in a particularly impressive manner in Einstein’s famous theory of general relativity. When Einstein published it, he knew that it had to be right if his premises were right, even though it was counterintuitive, unrelated to prior human experience, and would radically change our views of the physical world. Obviously, we also know that observation confirmed the theory.

                    ——–Your Feser “water spigot” quote does a good job of laying out the argument, but it, too, fails to demonstrate that neurons lack the potential to produce rational thoughts. It only argues for a certain conclusion if it can be shown that neurons don’t have this potential. But again, you need to demonstrate that. —————

                    So, Non-Credenti, if truth is nothing but the firing of neurons in your brain, then your belief that truth is nothing but the firing of neurons in the brain…..is nothing but the firing of neurons in your brain. How can the firing of neurons in your brain be true or false? Can other material bodily processes such as digestion be true or false?

                    Again, Non-Credenti, Feser elaborates:

                    …the cause of a fire might itself be on fire, as when a torch is used to start a brushfire, or it may instead have the power to produce fire, as a cigarette lighter has even when it is not being used.

                    The traditional way of making this distinction is to say that a cause has the feature that it generates in the effect “formally” in the first sort of case (e.g. when both the cause and the effect are on fire) and “eminently” in the second sort of case (e.g. when the cause is not itself on fire, but has an inherent power to produce fire). If a cause didn’t contain all the features of its effect either formally or eminently, there would be no way to account for how the effect came about in just the way it did. Again, a cause cannot give to its effect what it does not have to give.

                    A neuron is a purely non-rational physical thing which does not contain the effect of reason potentially in itself, either formally or eminently.

                    This goes back to what Feser pointed out: “What you would never seriously consider is the suggestion that normal water from the spigot all by itself produced the red puddle. For there is just nothing in water by itself that could produce the redness, thickness, or stickiness of the puddle; ergo there must have been something in addition to the water that produced the effect.”

                    Imagine if someone said, “You fail to demonstrate that normal water from a spigot could not produce a red puddle all by itself. Therefore, normal water from a spigot can produce a red puddle all by itself.”

                    Such an argument is patently fallacious, and is philosophically equivalent to your argument that neurons have the potential to produce rational thought because I have allegedly not demonstrated that neurons do not have the potential to produce rational thought.

                    Again, a neuron is a material object and you need to demonstrate how a material object contains the effect of rationality potentially in itself, either formally or eminently.

                    ———At times you seem to be trying to advance the claim that, on atheism, even a provisional reliability of our reasoning abilities is not possible “under any circumstances.” If so, then that’s an argument you need to support.—————-

                    No, this is not about the degree of reliability of our reasoning abilities. There is about how it is impossible to ground matter in reason.

                    ———-The next step is to posit that cognitive mechanisms which enable us to build a somewhat accurate map would have more survival benefit than ones in which the brain follows utterly random thoughts that are only accidentally—and rarely—correct. Done.—————–

                    But, according to atheism, theism is an inaccurate map. Therefore, we already have one indisputable example of where a belief allegedly produced by evolutionary processes is false. Was theistic belief not produced by the same alleged evolutionary processes as atheism?

                    ———-Therefore, your attack on my ability to attack your claim has failed. You have tried to categorize the firings of individual neurons as “irrational,” but even accepting the poor choice of words, you have not shown that these individual neurons “cannot” produce rational thoughts. You haven’t shown that individual neural processes lack the “potential” for producing rational thought.——————

                    Once again, you need to demonstrate that inanimate matter contains the effect of reason potentially within itself, either formally or eminently. Otherwise, you are just assuming what you intend to prove, which is an open-and-shut logical fallacy. Dictionary.com defines irrational as, “Without the faculty of reason; deprived of reason.” It is perfectly appropriate to characterize the firing of neurons as “without the faculty of reason,” just as it is perfectly appropriate to characterize the chair I am sitting in as “without the faculty of reason.”

                    You also need to demonstrate that inanimate matter contains potentially in itself the effects of consciousness, intelligence, personhood, etc. (either formally or eminently).

                    Nancy Pearcey and Charles Colson write in their book Finding Truth:

                    C. S. Lewis makes a similar argument in several of his writings. Here is an example: “If minds are wholly dependent on brains, and brains on biochemistry, and biochemistry (in the long run) on the meaningless flux of the atoms, I cannot understand how the thought of those minds should have any more significance than the sound of the wind in the trees.” Lewis then shows how this view defeats itself: “But if I can’t trust my own thinking, of course I can’t trust the arguments leading to Atheism, and therefore have no reason to be an Atheist, or anything else.” How do atheists or materialists avoid that self-refuting conclusion? They make what Lewis calls a “tacit exception” for their own theory— at least, at the moment they are stating their claims. In building their case, they must implicitly trust their own thinking.

                    They must exempt themselves from their own reductive categories of analysis. As one philosopher says, the materialist functions as though he were an “angelic observer” somehow able to float above the determinist cage in which he locks everyone else. In essence, materialists must tacitly assume a Christian epistemology, at least when they are arguing for their claims. Indeed, the sheer act of asserting materialism contradicts itself. If I say, “Everything that exists is material,” is that statement itself material? Is it merely a series of sound waves? If I write out the statement, is it nothing but marks on a piece of paper? Of course not. The statement has a linguistic meaning. It has logical properties. It has a social function (communicating to others)— all of which transcend the material dimension. Ironically, materialism cannot even be stated without refuting itself.

                    So, Non-Credenti, I have to ask you: Is the statement, “Everything that exists is material” a material object? If so, would you please send it to me via Fed Ex or UPS? I promise to reimburse you for the shipping and handling. Keep in mind, I am talking about the statement itself, not a piece of paper which has the statement written on it.

                    Bo Jinn continues:

                    ….for the sake of clarity, we shall spell out the problem of determinism, starting with why determinism and atheism are, by and large, inseparable. We have said that determinism is the denial of the human capacity for free will. Free will is incompatible with atheism for the reason that nondeterministic action would imply that there is something primary over matter and energy. Any naturalistic worldview would necessitate that human beings are an inextricable part of the same natural laws which created them. If atheism is true, then everything what we are and, furthermore, everything that we believe has to be the end result of consciousness responding to material stimuli without a say in the matter. The same way fire is doused by water and smoke rises from a burning log, everything about us has to be determined by the same physical rules of cause and effect that govern our material selves. If that were not the case, then we would be led to the conclusion that there is something at work controlling our material bodies- something that is separate and distinct from the body itself. Ergo; free will would necessitates that there is something primary over the matter which constitutes us, and that is a doorway to a very big problem for atheism.

                    Simply stated; free will automatically implies the existence of an immaterial, enduring self, since that enduring substance is guiding the body, and is therefore primary over it. If the self transcends the material human corpus that would mean that conscious experience is itself fundamental, and not simply a waste product of determined materialistic processes. And if consciousness is fundamental, it would mean that it had to have come from somewhere, something or (perish the thought) someone that is beyond mere matter and energy. Obviously the transcendental implications of this line of reasoning are anything but atheistic. That is why the vast majority of more refined atheists, including Dennett, Harris and Dawkins, are all determinists.

                    But what does this necessary implication of the atheist worldview say about our beliefs? Are not beliefs themselves the same result of this determined process? The possibility of having beliefs that are true or false necessitates that they were freely chosen. However, on atheism, that possibility does not exist. If all human action is determined, that would include the act of forming beliefs itself. If beliefs are determined, then they have no truth value. Atheism is one such belief, so atheism also loses its value and atheism becomes self-defeating once again.

                    Premise 1: Naturalism (atheism) implies that there is no free will.

                    Premise 2: Reasoning must be free in order to be valid.

                    Premise 3: Therefore, if free will does not exist, then reasoning is invalid.

                    Premise 4: If reasoning is invalid, then all our beliefs are invalid.

                    Conclusion: If all beliefs are invalid, naturalism (atheism) is invalid. If one commits oneself to the belief in determinism, that all beliefs can be reduced to uncontrolled material processes, then the possibility of maintaining a valid belief is negated, since the belief is an inevitable consequent of an irrational process with no interest in producing truth. [“Irrational”, is defined by Dictionary.com as, “Without the faculty of reason; deprived of reason.”]

                    That also goes for statements of science. On the other hand, if one accepts the existence of free will then that opens up a doorway to something superseding the natural universe and atheism is thrown into equal doubt.

                    Jinn above argues that consciousness is fundamental. And, as I have already pointed out to you, this is exactly what modern physics shows. In order to maintain your atheism, you have to ignore the insights of modern physics, which show that consciousness is fundamental, and that matter is derivative from consciousness.

                    Quantum physics shows that an immaterial conscious observer is required to collapse a possibility wave into material actuality. Please watch this video about the famous observer effect to see what I mean.

                    Physicist Amit Goswami writes,

                    “Now do you see why consciousness, to effect collapse, must be nonmaterial? A material consciousness arising in the brain is only a possibility wave. A possibility wave acting on a possibility wave just makes a bigger possibility wave. No actuality ever comes out of such an interaction (von Neumann 1955).”

                    “You may not have noticed, but we can see paradox in the observer effect in another way. The observer chooses, out of the quantum possibilities presented by the object, the actual event of experience. But before the collapse of the possibilities, the observer himself (or herself) consists of possibilities and is not manifest. So we can posit the paradox as a circularity: An observer is needed for collapsing the quantum possibility wave of an object; but collapse is needed for manifesting the observer. More succinctly, no collapse without an observer; but no observer without a collapse. If we stay in the material level, the paradox is unsolvable. The consciousness solution works only because we posit that consciousness collapses the possibility waves of both the observer (that is, his or her brain) and the object simultaneously from the transcendent reality of the ground of being that consciousness represents.”

                    The need for a conscious observer to collapse a probability wave into material reality means that matter is a construct of consciousness. Put another way, consciousness comes first and matter is produced by consciousness. This is why, for example, Max Planck, the founder of quantum physics, said:

                    “I regard consciousness as fundamental. I regard matter as derivative from consciousness. We cannot get behind consciousness. Everything that we talk about, everything that we regard as existing, postulates consciousness.”

                    It is also why Erwin Schroedinger, who made crucial contributions to modern physics, said:

                    “Consciousness cannot be accounted for in physical terms. For consciousness is absolutely fundamental. It cannot be accounted for in terms of anything else.”

                    It is also why Nobel Prize winning physicist Paul A. M. Dirac, who made crucial early contributions to both quantum mechanics and quantum electrodynamics, said:

                    “God is a mathematician of a very high order and He used advanced mathematics in constructing the universe.”

                    It is also why quantum physicist, John von Neumann, said,

                    “All real things are contents of consciousness.”

                    I have given my scientific reasons for believing in theism. Now please provide your scientific reasons for believing in materialism.

                    ———–Your second point—that atheism cannot explain the order of the universe (while theism can)—doesn’t seem to have anything to do with the epistemological issue of how Christians claim to be able to ground reason, which is the claim I launched all of my attacks against, so I’m going to set it aside as irrelevant unless you can tell me how it pertains.——————

                    The argument in my previous comment was about the accord between mathematics and physics. Physicist Paul Davies echoes a famous essay by Eugene Wigner, a Nobel Laureate in Physics, in which he wrote:

                    “The enormous usefulness of mathematics in the natural sciences is something bordering on the mysterious, and there is no rational explanation for it… it is an article of faith.” The relationship between mathematics and physics goes very deep and it is very hard to think of it as some random accident. Professor of Mathematics Sir Roger Penrose FRS, whose understanding of that relationship is unquestioned, has this to say about it: “It is hard for me to believe… that such SUPERB theories could have arisen merely by some random natural selection of ideas leaving only the good ones as survivors. The good ones are simply much too good to be the survivors of ideas that have arisen in a random way. There must be, instead, some deep underlying reason for the accord between mathematics and physics.” Certainly science itself cannot account for this phenomenon. Why? Because, in the words of John Polkinghorne: “Science does not explain the mathematical intelligibility of the physical world, for it is part of science’s founding faith that this is so.”

                    Non-Credenti, I am looking for your atheistic explanation for the deep accord between mathematics and physics….the mathematical intelligibility of the world. I can confidently predict that you will present an “it just is” response (if you don’t ignore this argument all together) because atheism is stuck with such just-so storytelling to explain such things.

                    Lastly, regarding your “BIV admission,” “Deceptive source,” “Rationality of source,” “Faulty receiver,” and “Source verification” arguments: We have been here before. If we cannot trust our reasoning because any or all of these objections are accurate, then we cannot trust the reasoning that you used to produce these objections. Your objections are self-defeating in the purest sense.

                    Your objections are “the convictions of a monkeys mind,” as Charles Darwin put it. And as Darwin argued, why should we trust the convictions of a monkeys mind?

                    You say that my claim fails if I cannot defeat your objections. But, if I cannot defeat your objections, it is not only my claim that fails. Your claims also fail, and all the claims made by all humans, ever, also fail because human reason cannot be said to be reliable. This is the self-defeating incoherence in which atheism is caught.

                    Your objections fail because we have solid reasons for trusting our reason. The intelligibility of the universe that Albert Einstein marveled at, and the “enormous usefulness of mathematics” which the Nobel Laureate Eugene Wigner marveled are two examples.

                    • Non Credenti says:

                      Sorry, this ended up being very long, but I think I’m getting to the point where future responses will be able to be much shorter.

                      Here you’ve re-presented your QM argument, about which we’ve had a little back-and-forth in that thread. As I said earlier, I want to stick to the topic at hand, which is your claim that Christians can in some way ground reason on God. QM is irrelevant to that epistemic claim. You seem to agree that this is the case, since you conclude your QM argument with the challenge: “I have given my scientific reasons for believing in theism. Now please provide your scientific reasons for believing in materialism.” But we agree that our conversation is much more specific than that. I’ve asked you a number of times to provide your reasons for believing the specific claim that you can ground reason on God. Please answer that question before branching out to such varied topics as QM. If you’d like, we can make a note to discuss QM separately. It’s a fascinating subject.

                      Theism explains Ordered Universe
                      Earlier you argued that Theism explains the Orderliness of the Universe. I questioned the relevance to the epistemic discussion, and dismissed your quotes from Ward and Lennox as useless opinion. You didn’t defend the quotes, so we’ll let them die as unsupported opinion. I concluded that even if it was relevant to your grounding claim, you hadn’t demonstrated that theism explains an ordered universe.

                      Your answers to both of my points were very similar. You repeated your Davies quote, and more specifically said, “… we have solid reasons for trusting our reason. The intelligibility of the universe that Albert Einstein marveled at…” You also mention the “usefulness of mathematics,” but these seem like nearly identical arguments, and it seems they’ll stand or fall by the same arguments.

                      This “Orderliness” argument hinges on two points, which you need to demonstrate: 1) Theists ground reason by means of the intelligibility of the universe, and; 2) God has ordered the universe.
                      Your claim, (2), seems to be both the most crucial and most difficult to demonstrate. I accept that the universe is ordered, but that doesn’t mean God ordered it. If you cannot demonstrate these two points, then my earlier objections–that this claim has not been demonstrated, and is irrelevant–stand.

                      Atheism cannot explain Ordered Universe
                      Next we move on to this claim’s counterpart—atheism cannot explain the Ordered Universe. You attempted to support it with quotes by Polkinghorne, Lennox, Einstein, Davies, Wigner, and a repeat of the same Polkinghorne quote. I challenged the Lennox quote as a claim that he (and by extension, you) had not demonstrated, and dismissed each of the others as useless opinion. You didn’t try to rescue the use of those opinion quotes. I also challenged this claim as irrelevant to the epistemic topic.

                      This claim is even less relevant than your previous claim concerning theism. If naturalists don’t have an explanation for the Ordered Universe, it doesn’t mean “Goddidit,” and it doesn’t undermine naturalistic reason. Naturalists don’t need to explain why the universe is ordered to make the simple observation that it is ordered. However, I’ll concede that if you can demonstrate that God has Ordered the Universe, then that would also be a demonstration of this contrary claim. You can kill two birds with one stone—all you need to do is show that God ordered the Universe.

                      You conclude your treatment of this topic by asking for my “atheistic explanation for the deep accord between mathematics and physics…” Your inability to demonstrate that God ordered the universe is not affected by my ability or inability to explain it, but since you asked…. The intelligibility of the universe is caused by the way N-Dimensional space folds. These dimensions “force” constants to be what they are—they cannot have other values because of the geometries of N-Dimensional space-time.

                      Your next group of claims surround reasoning under atheism. You claim that (1) the non-rational cannot cause the rational, therefore (2) atheists cannot produce a “reliably rational criticism,” and by contrast, (3) theists can produce rational arguments because of God’s conscious, intelligent mind. I’ll consider these in reverse order.

                      Theists can produce rationality because of God’s mind
                      (3) You say theists can make rational arguments because God’s “conscious, intelligent mind belongs to the category of things that can be said to have reason.” There are three problems with this claim:

                      First, it’s too ill-defined. Earlier I referenced a Magic Ice-Cream Sandwich (MICS) to illustrate the lack vacuity of your answer. If I claimed atheists can produce rationality because of the mind of MICS, you would object that it had no explanatory power. You would demand to know how MICS’s rationality meant anything about my rationality.

                      As I’ve been asking, what does grounding mean? How can the fact (if we accept it as a fact) that God has a rational mind guarantee your rationality? What’s the mechanism for transferring God’s rationality to you? How do you overcome the fact that God is deceptive? Or that the faculties you use to consider rational concepts are flawed? Or that you can’t know that any ‘reason’ that reaches you is actually from God? That it isn’t a powerful being impersonating God? That it isn’t a hallucination brought about by brain trauma or chemical changes in your brain? You can’t be sure you’re not in a coma hallucinating our entire exchange. Most fundamentally, how did you conclude any of this without using your reasoning abilities?—the very thing you’re trying to establish and justify. This can’t be done.

                      Second, your “consciousness is required for collapse” argument seems to undermine your claim that God is conscious. We’ve fired trillions of particles in double-slit experiments, but God has never collapsed a single one. If consciousness is required to collapse a wave function, yet none ever seem to be collapsed by God, then by your argument God is not conscious. It seems you need to abandon either the idea that consciousness causes collapse, or that God is conscious.

                      Third, it isn’t enough to contemplate that God could ground your reason, you need to demonstrate that God does ground your reason. A Magic Ice-Cream Sandwich could ground our reason… somehow… but could is not an impressive claim. Could would only suffice if you had demonstrated that no other explanation was possible, then the only possible explanation would carry, but you haven’t done that. You haven’t even demonstrated that naturalist reasoning is so unreliable that we have no choice but to reject it, let alone any of the nearly infinite other explanations we could come up with.

                      Atheists cannot produce reliably rational criticism
                      Moving on to your claim that, on atheism, atheists could not produce reliable thoughts, and therefore I cannot criticize your claims of theistic grounding, I countered with three arguments (and one clarification), one of which you did not challenge: (1) An accidentally rational challenge must be met. (2) You haven’t demonstrated your claim. (3) Our map and predictions indicate reliability. Additionally, I’ll introduce two more arguments in this post to defeat your claim: (4) My challenges are presented, “on theism.” (5) You sometimes accept the reasoning of atheists.

                      (1) As I’ve said, an accidentally rational challenge must still be met. Even if you could demonstrate that, on atheism, I can’t have rational thoughts, that doesn’t change the fact that, regardless of their source, I’ve somehow, by some fluke, managed to come up with seven rational arguments against your claim, and you can’t use the unknown nature of the source as an excuse to not address them. If you find a hungry lion in your living room, saying, “That’s not supposed to be here!” isn’t going to save you. Your response was to simply repeat the objection (nerve impulses cannot be true). That’s not an answer. A claim fails if it can’t meet rational challenges. My challenges are rational.

                      Interestingly, you quote Colson and Pearcey to support your argument, but they do more harm than good. You, through them, admit that thinking is a brain process. Well, there’s your answer. Some nerve impulses are thoughts, some thoughts express propositions, and some propositions are true. You’ve answered your own question.

                      (2) It seems your “nerve impulses can’t be true” argument also addresses my challenge to demonstrate your claim. We agree they’re not absolutely Reliable, but we also agree they’re not absolutely Unreliable, so you need to demonstrate they’re so amazingly faulty that I cannot challenge your claims. Your rebuttal doesn’t do this, and again, your quote of Colson and Pearcey sinks your argument.

                      (3) My third argument was that our ability to make a somewhat consistent map of reality–and to make consistent predictions using that map–is an indication that our reasoning is at least somewhat reliable. This is a direct rebuttal to your claim that, on atheism, we cannot ‘even in principle’ produce reliably rational criticism. You don’t directly respond, but in answer to a different point you say, (paraphrasing) ‘on atheism, theism is an inaccurate map, so don’t we have an example of evolution producing a false belief?’

                      I can immediately come up with four arguments against this, but for brevity’s sake, I’ll stick to the most important one: your argument doesn’t damage my claim, it only repeats it. I’ve said that our map is somewhat reliable. The main way we tell if the map is reliable is by whether predictions we make using the map are reliable or not. How could an ill-defined God who is free to break the rules whenever He pleases (and we humans are the ones who decide, post hoc, when God pleased to break the rules) ever be called either reliable or unreliable? Release an apple and it falls… God made it fall. Release an apple and it rises… God made it rise. Theism has no (or almost no) predictive qualities, so in that sense, maybe we should say it’s not even part of our map, or if so, it’s a really, really fuzzy part that we don’t rely on for navigation.

                      New arguments against your claim that atheists cannot produce reliably rational criticism:

                      (4) My challenges are Reductios showing that if I assume your claim it leads to logical absurdities or is refuted by itself or other claims of yours. My questions are… “On theism, what is grounding? On theism, how can we ground reason on a god who lies? On theism, how do we know Satan isn’t impersonating God and sending false information? Etc…” Since my challenges are presented “on theism,” you can’t avoid them by saying, “you can’t raise those questions, on atheism.”

                      (5) You present quotes from atheists all the time. It’s clear you have no problem accepting their reasoning when you agree with it. I claim your extensive quotations from atheists demonstrate that even you don’t buy your argument. You realize that atheists, “on atheism,” can produce rational arguments. (If you want to challenge this argument, I can provide many examples of you quoting atheists approvingly.)

                      In your concluding thoughts you directly address my seven objections and say, “If we cannot trust our reasoning because any or all of these objections are accurate,” then my challenges are self-defeating. This fails for two reasons: (1) I didn’t say if my challenges are correct, then ”we” can’t trust our “reasoning.” I said ”your” “argument” fails. It fails because you’re claiming some sort of grounding (a term you can’t define) on God’s perfect Reason (a thing you can’t establish), from a deceptive source (which has been established) to a faulty receiving apparatus (which has been established). My claims are not touched by my objections because I accept my epistemological limitations and don’t claim some external, supernatural source for anything. (2) Your objection requires that we interpret “cannot trust” as “cannot Trust” or “cannot trust even a little bit.” My argument is that we can (provisionally) trust our reasoning well enough to produce rational thoughts, as evidenced by our success navigating the map of reality we create. Unless you can demonstrate that, on atheism, my reasoning is guaranteed to always be faulty, such that everything I utter is automatically irrational, you can only test each claim of mine, individually, for rationality. You have not pointed out any irrationalities in the structure of any of my seven objections.

                      The non-rational can’t cause the rational
                      This brings us to what is probably your main argument, and the one upon which your “atheist criticism is irrational” argument is built. You claim that irrational things cannot cause rationality, specifically that non-rational neurons do not have the potential within themselves to cause rational thoughts.

                      In support of this you offered three quotes, which I rejected as either useless opinion or quotes that didn’t demonstrate your claim. In the case of your Ward quote, you didn’t object, but you challenged my dismissal of your Feser quote by way of returning to your “neurons firing cannot be true or false” argument. This won’t do you any good, because it doesn’t address my objection. Feser argues that if X lacks the potential to cause effect Y, then X cannot cause effect Y, which is pretty useless anyway because it’s a mere tautology. But I don’t object to his argument to this effect. My objection is that he (and you) has not demonstrated that X = collections of neurons, and Y = rational thought. You can’t just argue that things lacking feature X cannot do Y, you have to demonstrate that Feser’s argument applies to neurons and thoughts. You might be tempted to say, “Of course it applies! It’s obvious!” but it isn’t so obvious, as we’ll see shortly.

                      Next you launched a number of arguments against my objection that you hadn’t demonstrated your claim.

                      First you tried to shift the burden of proof (you try this twice in the same argument). “No, you need to demonstrate that non-rational processes have the potential for rational thought.” This is obviously false. If you make a claim, you need to support and defend that claim. If, in the course of objecting to your claim, I make claims of my own, that only means that now we both have assumed a burden of proof for different claims. So this defense is easily swatted aside by providing proof that you’ve actually made the claim I say you have:

                      From a post of yours on January 27: “The implication of this philosophical axiom is that irrational causes do not contain the effect of reason potentially in themselves. Impersonal causes do not contain the effect of personhood potentially in themselves. Unintelligent causes do not contain the effect of intelligence potentially in themselves.” This aptly demonstrates your burden.

                      Next you quote Moritz, who wants to create a difference between “basic” logical circuitry and the super-duper advanced circuitry needed for abstract arguments, etc. He concludes (without demonstrating it, naturally) that this couldn’t happen, and “given all the above,” says this would be the only way to “guarantee the right outcome.” Unfortunately, I don’t “give” all the above. Naturalism doesn’t claim to be able to “guarantee” any mental states. Evolution doesn’t need to “guarantee” any mental states. Also, if Moritz was correct in expecting a “guarantee,” then he is wrong to say there is no problem for theists, who would fail just as miserably if expected to “guarantee” any knowledge.

                      You then refer to Feser, and my objection that you need to demonstrate that his water spigot argument applies to brain matter. You say “if truth is nothing but the firing of neurons in your brain, then your belief that truth is nothing but the firing of neurons in the brain…..is nothing but the firing of neurons in your brain.” You, Feser, and Colson & Pearcy agree that, on atheism, “thinking is reduced to brain processes,” Therefore, on atheism, truth is a type of proposition, which is a type of thought, which is a brain state, which is a pattern of neuronal impulses. To say it’s “nothing but” the firing of neurons in the brain is woefully inadequate, and betrays an appalling ignorance of biology. “Other material bodily processes” cannot be true or false because they are not brain states corresponding to thoughts, which correspond to propositions, which may be true or false.

                      You continue with more from Feser, about formal vs eminent potential, but my earlier objection still stands: Demonstrate that “a neuron is a purely non-rational physical thing which does not contain the effect of reason potentially in itself, either formally or eminently.” Feel free to focus on “eminently,” since that’s what I would object to.

                      Finally, you object that, “if someone said, ‘You fail to demonstrate that normal water from a spigot could not produce a red puddle all by itself. Therefore, normal water from a spigot can produce a red puddle all by itself’… Such an argument is patently fallacious and is philosophically equivalent to your argument that neurons have the potential to produce rational thought because I have allegedly not demonstrated that neurons have the potential to produce rational thought.”

                      Here you misrepresent me. I didn’t say, “You failed to demonstrate your claim, therefore my claim stands.” I said “you failed to demonstrate your claim, therefore your argument fails.” Full stop–nothing about my claim. But I agree…. IF I had said that, it would have been fallacious.

                      Next, you quote Colson and Pearcey again, themselves quoting C. S. Lewis. Lewis’s ignorance doesn’t serve as an argument. If Lewis “cannot understand” how the mind should have any more significance than wind in trees, that speaks to the profoundness of his ignorance, not the power of his argument. While I disagree about what conclusion we should draw from it, I agree wholeheartedly with your observation that the universe is amazingly ordered… down to the atomic and subatomic levels. Perhaps if Lewis had considered that minds ‘depend on’ brains, and brains depend on biochemistry, and biochemistry depends on the amazing order at the atomic level, he might not have been so flummoxed. Regardless, as I said… Lewis’s bewilderment is not an argument.

                      You conclude their argument by asking, “Is the statement, ‘everything is material’ itself a material object'” Of course not, and no materialists say it is. It is a statement—an abstract–and like other abstracts, like numbers, emotions, etc., materialists don’t say it doesn’t exist, only that it’s a product of physical reality. Saying “everything ultimately stems from the material” is not at all the same as saying “everything is material.” Colson and Pearcey’s simplistic misrepresentation of materialism is not an argument.

                      You present a few more points meant to bolster your argument, some of which are variations of previous arguments. For example, you say that matter is not in the “category” of things that can have truth value. You enlist a quote from Barr to support this statement. The quote starts off with a faulty premise in that he says neurons are abstract concepts. There’s no need for me to refute that statement, as you’ve already done it for me many times. For example, in the very same post, you expand on a Feser quote and say, “a neuron is a purely non-rational physical thing.” [bold mine] I’ll let your refutation of Barr stand.

                      You conclude your quote of Barr with the $64,000 question, “How can one pattern of neuronal impulses be more true or less true than another pattern of neuronal impulses?” I’ve already answered it, and in fact you’ve answered it yourself, with your Colson-Pearcey quote, but it won’t hurt to repeat it to be sure the point is clear. Some “neuronal impulses,” firing within and among other groups of neurons, produce what we call brain states. Some brain states produce what we call thoughts or beliefs, or propositions. Some thoughts we call true if they map closely enough to reality. The brain state we call a thought is not the same as the sensory impulses we call a toothache, or the autonomic impulses (and chemical reactions) we call digestion. (Additionally, the thought “I have a toothache” is not the same as the sensory impulses causing us to perceive pain in that tooth.)

                      You also quote Jinn as he makes an argument against determinism and the ability of naturalists to have valid thoughts. Unfortunately, his argument depends on some unproven assumptions and faulty understanding. For example:
                      – He incorrectly states that free will is incompatible with atheism. He trusts his readers have never heard of compatibilism.
                      -He assumes free will.
                      -He claims that free will is incompatible with atheism
                      -He doesn’t disclose that Dennett is a famous compatibilist (this isn’t illogical as much as it is dishonest).
                      -Similarly, he draws the unfounded conclusion that if our reasoning is invalid (itself incorrect, see below) that all our beliefs are invalid. This is wrong because invalid reasoning can lead to true beliefs.
                      -He claims “having beliefs that are true or false necessitates that they were freely chosen,” and, “if beliefs are determined, then they have no truth value.” This is easily dismissed. Many philosophers would say that we can’t choose any of our beliefs. Most often our beliefs are compelled by mathematical or logical principles, or by our senses, which are “determined.” I cannot “freely choose” to believe that 2 + 2 = 4 or that 2 + 2 = 5. Once I understand how addition works, I’m compelled to believe that 2 + 2 = 4. True beliefs can also be compelled, for example, via hypnosis.

                      I have objections to all of his premises, though he could rescue (1) and (4), depending on how he defines things. But (2) and (3) are easily rejected, thus his argument fails.

                      To conclude, you directly address my seven initial challenges to your claim. You say, “If we cannot trust our reasoning because any or all of these objections are accurate, then we cannot trust the reasoning that you used to produce these objections.”

                      To this I have two objections: 1) I’ll refer to my earlier rebuttal, that how I reason things out is irrelevant. Worry about the validity and the rationality of the arguments presented. If you find them invalid or irrational, point that out, otherwise they’re valid challenges to your claim. (2) My reasoning would only be self-defeating if I couldn’t Absolutely Trust it. But that’s not my claim, and it’s not required, as you’ve agreed. All I need to do is trust my reasoning, not Trust my Reasoning. I agree that my reasoning can sometimes be faulty, and that’s why you are encouraged to examine each of my challenges individually, to be sure I haven’t succumbed to faulty reasoning. But, sorry, you don’t get to examine my challenges, find them valid and rational, and then try to dismiss them anyway as fruits of an absolute inability to reason.

                      Finally, you claim that if you cannot defeat my seven challenges, then not only your claim, but all claims would fail, since all human reason would have been proven unreliable. A scorched earth defense won’t save your argument. You can’t say, “Let my argument succeed because otherwise we wouldn’t like the outcome.” If your argument here is valid, then we’ll just have to live with the consequences if both of our claims of rational grounding die in the same fire. Fortunately, your argument is not valid. I haven’t claimed to ground reason on some outside source. All along, my claim has been that we cannot do this without engaging in invalid circular reasoning. So defeating your claimed ground for reason doesn’t touch my (non)claim, because I only attacked yours.

                      Having failed to justify why you’re not required to defend your claim, the challenges against it stand:
                      – Your claim is undefined. What is grounding?
                      – How can a person who cannot even be sure he’s not a BIV claim his reason is grounded on anything?
                      – How can you ground reason on a god who admits he lies to humans?
                      – How do you know God is rational? Did He tell you?
                      – If God is sending you some “Truth Signal” how do you know your faulty senses, memory, and logical faculties are processing the signal correctly?
                      – How do you know Satan is not impersonating God?
                      – How do you know you’re not hallucinating because of brain trauma or a chemical imbalance?

                    • Scott Youngren says:

                      Non-Credenti,

                      Apparently, my reply to your comment exceeded the maximum that the WordPress software will allow, so below is a continuation of my previous comment. Again, I have italicized your comments, and then responded below with asterisks surrounding my comments:

                      -He claims “having beliefs that are true or false necessitates that they were freely chosen,” and, “if beliefs are determined, then they have no truth value.” This is easily dismissed. Many philosophers would say that we can’t choose any of our beliefs. Most often our beliefs are compelled by mathematical or logical principles, or by our senses, which are “determined.” I cannot “freely choose” to believe that 2 + 2 = 4 or that 2 + 2 = 5. Once I understand how addition works, I’m compelled to believe that 2 + 2 = 4. True beliefs can also be compelled, for example, via hypnosis.

                      *************If we cannot choose any of our beliefs, then why are you at this website trying to convince people to choose to believe in atheism? This is what is known as a performative contradiction.

                      And if we cannot choose any of our beliefs, then we cannot choose to accept the belief that “we cannot choose any of our beliefs.” Therefore, even this belief is governed by mindless and non-rational natural forces outside of our control. Further, under atheism, theism is a false belief. Therefore we have one indisputable example of how a false belief has been “compelled by mathematical or logical principles, or by our senses, which are determined.”

                      How could one determine which beliefs are true and which are false if our beliefs are governed by natural forces outside of our control? Natural forces compelled you to be an atheist, and natural forces compelled me to be a theist. You HAD to be an atheist and I HAD to be a theist. On atheism, neither of us chose our beliefs. Rather, our beliefs were compelled upon us by natural forces outside our control. Therefore, it cannot be said that you used a process of reason to weigh the evidence and rationally choose atheism over theism. Atheism allows for no process of reason, and therefore cannot be said to have any grounding in reason.

                      On atheism, how could you determine that your beliefs are true? Since atheism only allows for beliefs which are “compelled” (to use YOUR OWN WORD), you were not able to apply your reasoning faculties to make a rational choice to embrace atheism.

                      And, aren’t our minds both governed by the same natural forces? Don’t you allege that we evolved from the same common ancestors?

                      Why were many other philosophers (and people such as myself) not compelled to believe that our beliefs are compelled? How can we determine who is right and why do the same natural forces compel conflicting beliefs?

                      You say that once you understand how addition works, you are compelled to believe that 2 + 2 = 4. But how do you know THIS if your beliefs were compelled by natural forces outside of your control? How did you weigh the evidence and make a volitional choice to believe that our beliefs are compelled by forces beyond your control….if your beliefs are compelled by forces beyond your control? Again, a rational conclusion requires a process of reason.***********

                      I have objections to all of his premises, though he could rescue (1) and (4), depending on how he defines things. But (2) and (3) are easily rejected, thus his argument fails.

                      To conclude, you directly address my seven initial challenges to your claim. You say, “If we cannot trust our reasoning because any or all of these objections are accurate, then we cannot trust the reasoning that you used to produce these objections.”

                      To this I have two objections: 1) I’ll refer to my earlier rebuttal, that how I reason things out is irrelevant. Worry about the validityand the rationality of the arguments presented. If you find them invalid or irrational, point that out, otherwise they’re valid challenges to your claim. (2) My reasoning would only be self-defeating if I couldn’t Absolutely Trust it. But that’s not my claim, and it’s not required, as you’ve agreed. All I need to do is trust my reasoning, not Trust my Reasoning. I agree that my reasoning can sometimes be faulty, and that’s why you are encouraged to examine each of my challenges individually, to be sure I haven’t succumbed to faulty reasoning. But, sorry, you don’t get to examine my challenges, find them valid and rational, and then try to dismiss them anyway as fruits of an absolute inability to reason.

                      Finally, you claim that if you cannot defeat my seven challenges, then not only your claim, but all claims would fail, since all human reason would have been proven unreliable. A scorched earth defense won’t save your argument. You can’t say, “Let my argument succeed because otherwise we wouldn’t like the outcome.” If your argument here is valid, then we’ll just have to live with the consequences if both of our claims of rational grounding die in the same fire. Fortunately, your argument is not valid. I haven’t claimed to ground reason on some outside source. All along, my claim has been that we cannot do this without engaging in invalid circular reasoning. So defeating your claimed ground for reason doesn’t touch my (non)claim, because I only attacked yours.

                      Having failed to justify why you’re not required to defend your claim, the challenges against it stand:
                      – Your claim is undefined. What is grounding?
                      – How can a person who cannot even be sure he’s not a BIV claim his reason is grounded on anything?
                      – How can you ground reason on a god who admits he lies to humans?
                      – How do you know God is rational? Did He tell you?
                      – If God is sending you some “Truth Signal” how do you know your faulty senses, memory, and logical faculties are processing the signal correctly?
                      – How do you know Satan is not impersonating God?
                      – How do you know you’re not hallucinating because of brain trauma or a chemical imbalance?

                      ************No, you don’t get it. In order for any of your objections to be accepted as rational, there must be a process of reason behind them. This is what grounding in reason means. But, under atheism, our thoughts are governed by the same mindless and non-rational natural processes that allegedly created us from non-living matter.

                      Thus, on atheism, there is no reason to accept any of your objections as rational. Rather, your objections are nothing but thoughts which are are determined by mindless natural forces outside of your control. Rational evaluation cannot occur if our thoughts are determined by mindless natural forces, because rational evaluation requires control of one’s reasoning faculties. Put another way, for a stance to be accepted as rational, there must be a process of reason behind it. On atheism, it cannot be said that you used a process of reason to weigh the evidence and rationally choose atheism over theism. Rather, your belief in atheism was “compelled” (to use your own word).

                      And if theism is true, but your objections are accepted as accurate, then we still have no reason to trust the reasoning faculties which you used to construct your objections. Therefore, we cannot trust your objections as rational. What you keep missing here is that there needs to be a grounding for reason in order for your arguments to be counted as rational.

                      You persistently confuse attacking my grounding for reason with constructing your own grounding for reason. In order to recognize an explanation as best, one need not have an explanation for the explanation. Requiring one to explain one’s explanation would lead to an infinite regress of explanations, and nothing could ever be explained. William Lane Craig comments:

                      “In order to recognize an explanation as the best, one needn’t have an explanation of the explanation. This is an elementary point concerning inference to the best explanation as practiced in the philosophy of science. If archaeologists digging in the earth were to discover things looking like arrowheads and hatchet heads and pottery shards, they would be justified in inferring that these artifacts are not the chance result of sedimentation and metamorphosis, but products of some unknown group of people, even though they had no explanation of who these people were or where they came from. Similarly, if astronauts were to come upon a pile of machinery on the back side of the moon, they would be justified in inferring that it was the product of intelligent, extra-terrestrial agents, even if they had no idea whatsoever who these extra-terrestrial agents were or how they got there. In order to recognize an explanation as the best, one needn’t be able to explain the explanation. In fact, so requiring would lead to an infinite regress of explanations, so that nothing could ever be explained and science would be destroyed. So in the case at hand, in order to recognize that intelligent design is the best explanation of the appearance of design in the universe, one needn’t be able to explain the designer.”

                      I have produced evidence and reasoning to explain why God is the best explanation…including evidence from modern physics and biology. Trying to get me to explain my explanation by presenting a bunch of objections is a red herring fallacy (diversionary tactic). This is especially true since you cannot even produce atheistic explanations for why we should trust our beliefs, or for the cause of order in the universe, etc….let alone an explanations for your explanations. Again, in order to attack theism, you must hold theism to a standard of scrutiny which atheism itself cannot withstand.

                      Let’s apply your standard of demanding an explanation for an explanation (which leads to an infinite regress of explanations) to the following hypothetical scientific discussion:

                      Person 1: “Observation and experiment shows that there is a law of gravity which draws objects towards one another.”

                      Person 2: “How do you know that the law of gravity is real? Maybe you are a brain in a vat who has been deceived by a mad scientist into thinking that this law exists. Or maybe when you (and everyone else) experience objects falling, you are hallucinating.

                      Please note that, no matter how person 1 answers the objections of person 2, person 2 can continue to present new objections ad infinitum, which will lead to an infinite regress of providing explanations for explanations.

                      Please also note that person 2 must provide a superior counter explanation for why we repeatedly experience objects being drawn to one another in order to invalidate the hypothesis of a law of gravity postulated by person 1. By merely providing objections (demands for explanations of explanations), person 2 cannot discredit person 1’s explanation.

                      Again, trying to get me to provide explanations for my explanations is a red herring fallacy (diversionary tactic). You are transparently trying to divert attention away from atheism’s inability to explain such things as 1) the cause of order in the universe and 2) the reason that we should trust our reasoning faculties.

                      Regarding the cause of order in the universe, you have admitted that atheism cannot produce an explanation. This must be counted as a fatal explanatory failure of atheism. With no counter-explanation whatsoever, you cannot invalidate my explanation with requests to provide explanations for explanations.

                      And regarding the reason we should trust our reasoning faculties, you have tried to use a tautological explanation: You write: “My argument is that we can (provisionally) trust our reasoning well enough to produce rational thoughts, as evidenced by our success navigating the map of reality we create.” This is tautological because we have to trust the reasoning faculties which we used to conclude that we have successfully navigated our map of reality, in order to trust that our assessment that we have successfully navigated our map of reality.

                      Here, you also confuse an observation, on one hand, with an explanation, on the other hand. A worldview must provide explanations in order to be coherent, not mere observations.

                      God admits he lies to humans? Where did you get that from? This is truly bizarre.**********

                    • Scott Youngren says:

                      Non Credenti,

                      I have italicized your comments, and then encased my replies with asterisks:

                      Sorry, this ended up being very long, but I think I’m getting to the point where future responses will be able to be much shorter.

                      Here you’ve re-presented your QM argument, about which we’ve had a little back-and-forth in that thread. As I said earlier, I want to stick to the topic at hand, which is your claim that Christians can in some way ground reason on God. QM is irrelevant to that epistemic claim. You seem to agree that this is the case, since you conclude your QM argument with the challenge: “I have given my scientific reasons for believing in theism. Now please provide your scientific reasons for believing in materialism.” But we agree that our conversation is much more specific than that. I’ve asked you a number of times to provide your reasons for believing the specific claim that you can ground reason on God.

                      ***********And I have answered that question. A cause cannot give to its effect what it does not have to give. Mindless matter does not contain potentially in itself the effects of consciousness, personhood, intelligence, morality, etc. Further, in order for mindless matter to produce these effects through some process of evolution, mindless matter must have organizing and creative properties. But what reason do we have to conclude that mindless matter has the ability to create or organize? To believe as such would be the worst form of superstition.***************

                      Please answer that question before branching out to such varied topics as QM. If you’d like, we can make a note to discuss QM separately. It’s a fascinating subject.

                      *************Since I’ve answered your question about my specific reasons for grounding reason in the mind of God, I will continue with my discussion of quantum physics. And since you have tried to avoid my points about quantum physics, I will just copy and paste them again:

                      Physicist Amit Goswami writes:

                      “First let’s discuss how the idea that consciousness is the ground of being is forced upon us by quantum physics. Take the idea that conscious choice affects the quantum possibility wave of an object by collapsing it into an actual event of our experience, into a “particle,” so to speak. This idea seems dualistic at first. Why? Because consciousness has to be nonmaterial to effect collapse. To see this, suppose, as materialist biologists believe, that consciousness is a brain epiphenomenon. But undoubtedly the brain is a conglomerate of elementary particles, quantum possibilities, so it must itself also consist of quantum possibilities. Ditto for any epiphenomenon associated with it.”

                      “Now do you see why consciousness, to effect collapse, must be nonmaterial? A material consciousness arising in the brain is only a possibility wave. A possibility wave acting on a possibility wave just makes a bigger possibility wave. No actuality ever comes out of such an interaction (von Neumann 1955).”

                      “You may not have noticed, but we can see paradox in the observer effect in another way. The observer chooses, out of the quantum possibilities presented by the object, the actual event of experience. But before the collapse of the possibilities, the observer himself (or herself) consists of possibilities and is not manifest. So we can posit the paradox as a circularity: An observer is needed for collapsing the quantum possibility wave of an object; but collapse is needed for manifesting the observer. More succinctly, no collapse without an observer; but no observer without a collapse. If we stay in the material level, the paradox is unsolvable. The consciousness solution works only because we posit that consciousness collapses the possibility waves of both the observer (that is, his or her brain) and the object simultaneously from the transcendent reality of the ground of being that consciousness represents.”

                      The need for a conscious observer to collapse a probability wave into material form means that matter is a construct of consciousness. Put another way, consciousness comes first and matter is produced by consciousness. This is why, for example, Max Planck, the founder of quantum physics, said:

                      “I regard consciousness as fundamental. I regard matter as derivative from consciousness. We cannot get behind consciousness. Everything that we talk about, everything that we regard as existing, postulates consciousness.”

                      It is also why Erwin Schroedinger, who made crucial contributions to modern physics, said:

                      “Consciousness cannot be accounted for in physical terms. For consciousness is absolutely fundamental. It cannot be accounted for in terms of anything else.”

                      It is also why Nobel Prize winning physicist Paul A. M. Dirac, who made crucial early contributions to both quantum mechanics and quantum electrodynamics, said:

                      “God is a mathematician of a very high order and He used advanced mathematics in constructing the universe.”

                      It is also why quantum physicist, John von Neumann, said,

                      “All real things are contents of consciousness.”************

                      Theism explains Ordered Universe

                      Earlier you argued that Theism explains the Orderliness of the Universe. I questioned the relevance to the epistemic discussion, and dismissed your quotes from Ward and Lennox as useless opinion. You didn’t defend the quotes, so we’ll let them die as unsupported opinion. I concluded that even if it was relevant to your grounding claim, you hadn’t demonstrated that theism explains an ordered universe.

                      Your answers to both of my points were very similar. You repeated your Davies quote, and more specifically said, “… we have solid reasons for trusting our reason. The intelligibility of the universe that Albert Einstein marveled at…” You also mention the “usefulness of mathematics,” but these seem like nearly identical arguments, and it seems they’ll stand or fall by the same arguments.

                      This “Orderliness” argument hinges on two points, which you need to demonstrate: 1) Theists ground reason by means of the intelligibility of the universe, and; 2) God has ordered the universe.
                      Your claim, (2), seems to be both the most crucial and most difficult to demonstrate. I accept that the universe is ordered, but that doesn’t mean God ordered it. If you cannot demonstrate these two points, then my earlier objections–that this claim has not been demonstrated, and is irrelevant–stand.

                      *******Conscious and intelligent agents belong to a category of things known to be able to produce order from disorder. Mindless matter (atoms and molecules) do not belong to a category of things known to be able to produce order from disorder. You can say that natural laws acting upon mindless material things can produce order, but this just shifts the question to what governs natural laws. And, as you have admitted, naturalism cannot explain order, and can therefore never explain who or what governs natural laws. Rather, naturalism is stuck with an “it just does” explanation for why matter follows natural laws. This is the just-so storytelling that is so characteristic of the atheist worldview.

                      Again, ascribing organizing and/or creative properties to mindless matter is the worst form of superstition imaginable.

                      Further, atheism must ignore the insights of modern physics, which demonstrate conclusively that an immaterial conscious observer is necessary to collapse a probability wave. And ignoring the insights of modern physics which I called attention to in a previous comment is exactly what you have chosen to do. “***********

                      Atheism cannot explain Ordered Universe

                      Next we move on to this claim’s counterpart—atheism cannot explain the Ordered Universe. You attempted to support it with quotes by Polkinghorne, Lennox, Einstein, Davies, Wigner, and a repeat of the same Polkinghorne quote. I challenged the Lennox quote as a claim that he (and by extension, you) had not demonstrated, and dismissed each of the others as useless opinion. You didn’t try to rescue the use of those opinion quotes. I also challenged this claim as irrelevant to the epistemic topic.

                      This claim is even less relevant than your previous claim concerning theism. If naturalists don’t have an explanation for the Ordered Universe, it doesn’t mean “Goddidit,” and it doesn’t undermine naturalistic reason. Naturalists don’t need to explain why the universe is ordered to make the simple observation that it isordered. However, I’ll concede that if you can demonstrate that God has Ordered the Universe, then that would also be a demonstration of this contrary claim. You can kill two birds with one stone—all you need to do is show that God ordered the Universe.

                      **************Here, you characterize my proposal that God ordered the universe as an arbitrary speculation for which I do not produce an argument (“Goddidit”). But this is another one of your transparent straw-man arguments. I have already given you my reasons for postulating that God caused the universe. The first of these arguments is that a cause cannot give to its effect what it does not have to give. A conscious, personal, and infinitely intelligent agent has the effects of consciousness, intelligence and personhood contained potentially in itself.

                      Mindless matter (atoms and molecules) does not have creative or organizing properties, nor does it contain the effects of consciousness, intelligence, and personhood potentially in itself. You can counter this with an assertion that they DO contain such properties, but your atheist worldview can offer nothing more substantive to back up this claim than a bald assertion. We observe conscious and intelligent agents creating and organizing every single day, but where do we experience mindless matter creating or organizing anything?

                      Further, the universe (which includes space, time, matter, and energy) originated at the Big Bang. Since something cannot cause itself, and since everything with a beginning (non-eternal) requires a cause, the cause of the universe must be spaceless, timeless, immaterial, and energy-less. Physicist George Stanciu and philosopher Robert Augros elaborate in their book The New Story of Science:

                      “In the New Story of science the whole universe–including matter, energy, space, and time–is a one-time event and had a definite beginning. But something must have always existed; for if ever absolutely nothing existed, then nothing would exist now, since nothing comes from nothing. The material universe cannot be the thing that always existed because matter had a beginning. It is 12 to 20 billion years old. This means that whatever has always existed is non-material. The only non-material reality seems to be mind. If mind is what has always existed, then matter must have been brought into existence by a mind that always was. This points to an intelligent, eternal being who created all things. Such a being is what we mean by the term God.”

                      The above reasoning is the same reason why, for example, the astronomer, physicist and founder of NASA’s Goddard Institute of Space Studies Robert Jastrow writes in his book God and the Astronomers:

                      “Astronomers now find they have painted themselves into a corner because they have proven, by their own methods, that the world began abruptly in an act of creation to which you can trace the seeds of every star, every planet, every living thing in this cosmos and on the earth. And they have found that all this happened as a product of forces they cannot hope to discover…. That there are what I or anyone would call supernatural forces at work is now, I think, a scientifically proven fact.”

                      The above reasoning is also what led Allan Sandage, who was widely regarded as the world’s greatest cosmologist until his death in 2010 (and one of the founders of modern astronomy), to accept the existence of God, as he announced at a conference on the origin of the universe in 1985. Sandage also became a Christian, and he wrote:

                      “I find it quite improbable that such order came out of chaos. There has to be some organizing principle. God to me is a mystery but is the explanation for the miracle of existence, why there is something instead of nothing.”

                      Atheism, does not provide any such first organizing principle…unless one chooses to very superstitiously ascribe organizing and creative properties to mindless matter. And failure to provide a first organizing principle is a fatal explanatory failure of atheism.**************

                      You conclude your treatment of this topic by asking for my “atheistic explanation for the deep accord between mathematics and physics…” Your inability to demonstrate that God ordered the universe is not affected by my ability or inability to explain it, but since you asked…. The intelligibility of the universe is caused by the way N-Dimensional space folds. These dimensions “force” constants to be what they are—they cannot have other values because of the geometries of N-Dimensional space-time.

                      ********This is a very bizarre answer to the question of the deep accord between mathematics and physics. Can you please elaborate upon how a “N-Dimensional space fold” can cause an accord between mathematics and physics? How can an “N-Dimensional space fold” cause an accord between mathematics and physics? How does this refute the Cambridge University physicist John Polkinghorne’s comment that “Science does not explain the mathematical intelligibility of the physical world, for it is part of science’s founding faith that this is so” ? Why is the universe structured so that such “geometries of N-Dimensional space-time” exist?************

                      Your next group of claims surround reasoning under atheism. You claim that (1) the non-rational cannot cause the rational, therefore (2) atheists cannot produce a “reliably rational criticism,” and by contrast, (3) theists can produce rational arguments because of God’s conscious, intelligent mind. I’ll consider these in reverse order.

                      Theists can produce rationality because of God’s mind
                      (3) You say theists can make rational arguments because God’s “conscious, intelligent mind belongs to the category of things that can be said to have reason.” There are three problems with this claim:

                      First, it’s too ill-defined. Earlier I referenced a Magic Ice-Cream Sandwich (MICS) to illustrate the lack vacuity of your answer. If I claimed atheists can produce rationality because of the mind of MICS, you would object that it had no explanatory power. You would demand to know how MICS’s rationality meant anything about my rationality.

                      ***********An ice cream sandwich is an inanimate material thing. A observation from an immaterial conscious mind is necessary to “collapse the wave function” before a material thing can exist. Mind precedes matter, as modern physics has demonstrated.

                      Further, to ascribe the property of mind to an inanimate material thing such as an ice cream sandwich would be another superstitious act. Ascribing the property of mind to a conscious and intelligent agent such as God, however, would not be a superstitious act.************

                      As I’ve been asking, what does grounding mean? How can the fact (if we accept it as a fact) that God has a rational mind guarantee your rationality? What’s the mechanism for transferring God’s rationality to you? How do you overcome the fact that God is deceptive? Or that the faculties you use to consider rational concepts are flawed? Or that you can’t know that any ‘reason’ that reaches you is actually from God? That it isn’t a powerful being impersonating God? That it isn’t a hallucination brought about by brain trauma or chemical changes in your brain? You can’t be sure you’re not in a coma hallucinating our entire exchange. Most fundamentally, how did you conclude any of this without using your reasoning abilities?—the very thing you’re trying to establish and justify. This can’t be done.

                      *************Grounding of reason is a very easy concept to understand. In order for a belief to be grounded in reason, there must be a process of reasoning behind it. But, on atheism, our thoughts are governed by the same mindless and non-rational processes which allegedly created us. There is no reason to believe that thoughts produced by such forces could be deemed rational. Put another way, thoughts produced by non-rational deterministic forces are non-rational determined thoughts, not rationally produced thoughts. Only a process of reason (a free-will weighing of competing views) can produce rational thoughts.

                      Again, if theism is true, but your objections are also true, then the reasoning faculties you used to construct your objections cannot be trusted. **************

                      Second, your “consciousness is required for collapse” argument seems to undermine your claim that God is conscious. We’ve fired trillions of particles in double-slit experiments, but God has never collapsed a single one. If consciousness is required to collapse a wave function, yet none ever seem to be collapsed by God, then by your argument God is not conscious. It seems you need to abandon either the idea that consciousness causes collapse, or that God is conscious.

                      **********An observation by a conscious observer is necessary for material things to take form from a possibility wave. A conscious observer was necessary to produce the physical reality which preceded human agents. This points to a conscious agent which produced the material world. ***************

                      Third, it isn’t enough to contemplate that God could ground your reason, you need to demonstrate that God does ground your reason. A Magic Ice-Cream Sandwich couldground our reason… somehow… but couldis not an impressive claim. Could would only suffice if you had demonstrated that no other explanation was possible, then the only possible explanation would carry, but you haven’t done that. You haven’t even demonstrated that naturalist reasoning is so unreliable that we have no choice but to reject it, let alone any of the nearly infinite other explanations we could come up with.

                      **********Here, you provide an excellent example of how atheists must apply degree of scrutiny to theism much higher than that which atheism can withstand. Theists must demonstrate that a conscious and intelligent agent which contains the attribute of reason grounds reason? In what way have you demonstrated that mindless matter DOES ground reason? You must hold theism to a degree of scrutiny which atheism cannot itself withstand in order to argue for atheism.

                      Further, it is superstitious to ascribe the attribute of reason to an inanimate object such as an ice cream sandwich. It is not superstitious, however, to ascribe that attribute of reason to a conscious and intelligent agent. This is especially the case in light of the insights of modern physics which demonstrate that consciousness precedes matter.**************

                      Atheists cannot produce reliably rational criticism

                      Moving on to your claim that, on atheism, atheists could not produce reliable thoughts, and therefore I cannot criticize your claims of theistic grounding, I countered with three arguments (and one clarification), one of which you did not challenge: (1) An accidentally rational challenge must be met. (2) You haven’t demonstrated your claim. (3) Our map and predictions indicate reliability. Additionally, I’ll introduce two more arguments in this post to defeat your claim: (4) My challenges are presented, “on theism.” (5) You sometimes accept the reasoning of atheists.

                      (1) As I’ve said, an accidentally rationalchallenge must still be met. Even if you could demonstrate that, on atheism, I can’t have rational thoughts, that doesn’t change the fact that, regardless of their source, I’vesomehow, by some fluke, managed to come up with seven rational arguments against your claim, and you can’t use the unknown nature of the source as an excuse to not address them. If you find a hungry lion in your living room, saying, “That’s not supposed to be here!” isn’t going to save you. Your response was to simply repeat the objection (nerve impulses cannot be true). That’s not an answer. A claim fails if it can’t meet rational challenges. My challenges are rational.

                      *************An accidentally rational argument can only occur if a person first has reasoning capability. But how can one have reasoning capability if we assume that naturalism is true? A copy and paste from Moritz:

                      “How then can the naturalist nonetheless assert that naturalism is true and its acceptance rational? S/he considered the evidence, s/he will reply. Yet under naturalism the brain determines how to interpret the evidence – you have no say in that. So the naturalist’s brain determined that naturalism is true, and mine determined, considering the evidence as well, that naturalism is not true. Now, which brain is right? If the naturalist’s acceptance of naturalism is solely dependent on the firing of his/her neurons over which s/he has no control (under determinism), then it is not possible for him/her to know that his/her brain is right and that naturalism is true. Thus under naturalism the claim that naturalism is true becomes incoherent and self-contradictory. Naturalism defeats itself.”

                      You have not responded to Moritz’s argument. Please also respond to Jinn’s below argument, which reflects Moritz:

                      Premise 1: Naturalism (atheism) implies that there is no free will.

                      Premise 2: Reasoning must be free in order to be valid.

                      Premise 3: Therefore, if free will does not exist, then reasoning is invalid.

                      Premise 4: If reasoning is invalid, then all our beliefs are invalid.

                      Conclusion: If all beliefs are invalid, naturalism (atheism) is invalid. If one commits oneself to the belief in determinism, that all beliefs can be reduced to uncontrolled material processes, then the possibility of maintaining a valid belief is negated, since the belief is an inevitable consequent of an irrational process with no interest in producing truth. [“Irrational”, is defined by Dictionary.com as, “Without the faculty of reason; deprived of reason.”]******************

                      Interestingly, you quote Colson and Pearcey to support your argument, but they do more harm than good. You, through them, admit that thinking is a brain process. Well, there’s your answer. Some nerve impulses are thoughts, some thoughts express propositions, and some propositions are true. You’ve answered your own question.

                      (2) It seems your “nerve impulses can’t be true” argument also addresses my challenge to demonstrate your claim. We agree they’re not absolutely Reliable, but we also agree they’re not absolutely Unreliable, so you need to demonstrate they’re so amazingly faulty that I cannot challenge your claims. Your rebuttal doesn’t do this, and again, your quote of Colson and Pearcey sinks your argument.

                      *****************Assuming that atheism is true, nerve impulses are governed by the same mindless and non-rational natural forces which allegedly created human beings. How can you weigh the evidence and make a logical choice between atheism and theism if your thoughts are determined by nerve impulses which are guided by natural forces outside of your control?

                      How did you make the assessment that atheism is true? By examining the evidence and making a logical choice? How can you make a logical choice if your thoughts are determined by natural forces outside of your control? Once again, atheism refutes itself and therefore makes the theist’s job very easy.

                      Recall that citation from Bo Jinn again about how determinism and atheism are inseparable:

                      “….for the sake of clarity, we shall spell out the problem of determinism, starting with why determinism and atheism are, by and large, inseparable. We have said that determinism is the denial of the human capacity for free will. Free will is incompatible with atheism for the reason that nondeterministic action would imply that there is something primary over matter and energy. Any naturalistic worldview would necessitate that human beings are an inextricable part of the same natural laws which created them. If atheism is true, then everything what we are and, furthermore, everything that we believe has to be the end result of consciousness responding to material stimuli without a say in the matter. The same way fire is doused by water and smoke rises from a burning log, everything about us has to be determined by the same physical rules of cause and effect that govern our material selves. If that were not the case, then we would be led to the conclusion that there is something at work controlling our material bodies- something that is separate and distinct from the body itself. Ergo; free will would necessitates that there is something primary over the matter which constitutes us, and that is a doorway to a very big problem for atheism.”

                      Non Credenti, if consciousness is the product of nerve impulses governed by natural forces, then natural forces govern your thoughts, and you do not have the ability to weigh the evidence and make a volitional choice between atheism and theism. Natural forces caused you to be an atheist, and natural forces caused me to be a theist. But aren’t you and I governed by the same natural forces? And if natural forces lead to contradictory stances such as theism and atheism, then how can natural forces lead us to truth?

                      And if nerve impulses are governed by something other than natural forces, then this leads inextricably to the conclusion that there is something primary over the matter which constitutes us (read: consciousness), just as modern physics has conclusively demonstrated, but which you choose to reject for apparently ideological reasons.

                      Further, don’t you allege that you and I both evolved from the same common ancestors as a result of the same mindless and non-rational natural forces? If so, then why did the natural forces which govern nerve impulses in the brain cause your nerve impulses to produce a belief in atheism, whereas my nerve impulses produce a belief in theism?

                      Which set of nerve impulses is right? How could we possibly make such a determination between atheism and theism if the nerve impulses in our brains are determined by natural forces outside of our control? Your stance is thoroughly incoherent.******************

                      (3) My third argument was that our ability to make a somewhat consistent map of reality–and to make consistent predictions using that map–is an indication that our reasoning is at least somewhat reliable. This is a direct rebuttal to your claim that, on atheism, we cannot ‘even in principle’ produce reliably rational criticism. You don’t directly respond, but in answer to a different point you say, (paraphrasing) ‘on atheism, theism is an inaccurate map, so don’t we have an example of evolution producing a false belief?’

                      ************But in order to reason, we must have the ability to weigh the evidence and make volitional choices between stances such as theism and atheism. But how can we weigh the evidence and make a volitional choice between conflicting views if our thoughts are the product of nerve impulses which are governed by natural forces outside of our control? And if our nerve impulses are governed by something other than natural forces which allegedly produced our bodies, this leads to the conclusion that there is something above and beyond the physical body itself governing our thoughts, which opens the door to consciousness independent of the material body. Your stance is incoherent.*****************

                      I can immediately come up with four arguments against this, but for brevity’s sake, I’ll stick to the most important one: your argument doesn’t damage my claim, it only repeats it. I’ve said that our map is somewhatreliable. The main way we tell if the map is reliable is by whether predictions we make using the map are reliable or not. How could an ill-defined God who is free to break the rules whenever He pleases (and we humans are the ones who decide, post hoc, when God pleased to break the rules) ever be called either reliable or unreliable? Release an apple and it falls… God made it fall. Release an apple and it rises… God made it rise. Theism has no (or almost no) predictive qualities, so in that sense, maybe we should say it’s not even part of our map, or if so, it’s a really, really fuzzy part that we don’t rely on for navigation.

                      New arguments against your claim that atheists cannot produce reliably rational criticism:

                      (4) My challenges are Reductios showing that if I assume your claim it leads to logical absurdities or is refuted by itself or other claims of yours. My questions are… “On theism, what is grounding? On theism, how can we ground reason on a god who lies?On theism, how do we know Satan isn’t impersonating God and sending false information? Etc…” Since my challenges are presented “on theism,” you can’t avoid them by saying, “you can’t raise those questions,on atheism.”

                      *************The concept of grounding is very easy to grasp. In order for reason to exist, we must have reasoning faculties which are in our control…not in the control of blind natural forces. Blind natural forces cannot produce reason, only beliefs which are determined. Beliefs which are determined by natural forces cannot be determined to be true because making such a determination requires the ability to examine evidence and make a volitional choice between competing views. If our thoughts are nothing but nerve impulses are governed by natural forces, then reasoning ability cannot exist. Again, reasoning ability requires control of our thoughts, but there is no way for us to have control if our thoughts are governed by non-rational natural forces.

                      As Pearcey, Colson, and Lewis point out, natural forces produce mechanical, determined actions, and nothing more. This is why, under atheism, it is accurate to characterize thought as an physically determined and involuntary bodily processes, just as digestion is a physically determined and involuntary bodily process. Yes, thought may be a more complex determined bodily process than digestion, but determined nonetheless. In order for reason to exist, we must have reasoning faculties which are in our control.

                      Your “what if God lies” and “Satan might be impersonating God” arguments fail because if there is not a reliable grounding for reason, then the reasoning faculties you used to construct these objections cannot be trusted. In order for these objections to be accepted as true, you must be able to have reliable reasoning faculties. But how can you have reliable reasoning faculties if your reasoning is under the control of natural forces beyond your control?**************

                      (5) You present quotes from atheists all the time. It’s clear you have no problem accepting their reasoning when you agree with it. I claim your extensive quotations from atheists demonstrate that even you don’t buy your argument. You realize that atheists, “on atheism,” can produce rational arguments. (If you want to challenge this argument, I can provide many examples of you quoting atheists approvingly.)

                      ***********Yes, I absolutely love to quote atheists, because in doing so, I can illustrate how even atheists know that theism is true. Their objections to theism can be illustrated to be ideological and psychological in nature, as opposed to logical.

                      For example, DNA (the language of life) is a language in the most literal sense. This is no metaphor. Specifically, DNA uses abstract symbolic representation to codify and store information. And information science tells us that codified information is always the product of a conscious and intelligent mind.

                      In Why Life Could Not Have Emerged Without God, I mention that the physicist Paul Davies points out that the phenomenon of the genetic code mediating information between the two languages of life (proteins and nucleic acids) provides a mystery: How can mindless processes set up codes and languages?

                      In his book Evolution 2.0, Perry Marshall explains the scientific reasons why DNA is a language in the most literal sense. This is not some “loosey-goosey analogy,” as he puts it:

                      Rutgers University professor Sungchul Ji’s excellent paper “The Linguistics of DNA: Words, Sentences, Grammar, Phonetics, and Semantics”starts off, “Biologic systems and processes cannot be fully accounted for in terms of the principles and laws of physics and chemistry alone, but they require in addition the principles of semiotics— the science of symbols and signs, including linguistics.”

                      Ji identifies 13 characteristics of human language. DNA shares 10 of them. Cells edit DNA. They also communicate with each other and literally speak a language he called “cellese,” described as “a self-organizing system of molecules, some of which encode, act as signs for, or trigger, gene-directed cell processes.”

                      This comparison between cell language and human language is not a loosey-goosey analogy; it’s formal and literal. Human language and cell language both employ multilayered symbols. Dr. Ji explains this similarity in his paper: “Bacterial chemical conversations also include assignment of contextual meaning to words and sentences (semantic) and conduction of dialogue (pragmatic)— the fundamental aspects of linguistic communication.” This is true of genetic material. Signals between cells do this as well.

                      Ultra-elite atheist biologists clearly know that DNA must necessarily be the product of a conscious and intelligent mind. This is why so many of them have adopted the aliens-brought-life-to-earth-in-thier-spaceship explanation for the origin of life from non-living matter (known as “directed panspermia”).

                      This would include Francis Crick, the atheist biologist who is famous as the co-discoverer of the DNA double helix. Click here to read an article which discusses his endorsement of this hypothesis in his book Life Itself.

                      Click here to watch Richard Dawkins (the world’s most famous atheist biologist) endorsing this hypothesis in an interview.

                      Other very prominent atheist scientists who endorse this hypothesis include the physicist and mathematician Fred Hoyle, the biologist Chandra Wickramsinghe, and the chemist Leslie Orgel. Fred Hoyle, an extremely prominent physicist and mathematician from Cambridge University, admits the following despite being an atheist:

                      “A common sense interpretation of the facts suggests that a superintellect has monkeyed with physics, as well as with chemistry and biology, and that there are no blind forces worth speaking about in nature. The numbers one calculates from the facts seem to me so overwhelming as to put this conclusion almost beyond question.”

                      When asked what this “superintellect” is, Hoyle responds that it is an “alien mind.” Citing space aliens as the source of mind which created the universe and life from non-life is a clear example of how atheists relieve the cognitive dissonance produced by their scientific knowledge of God.

                      Nobel Prize-winning Harvard University biologist George Wald, although certainly not an ideological ally of theism, is commendable for admitting the following in his address to the Quantum Biology Symposium titled Life and Mind in the Universe:

                      “It has occurred to me lately—–I must confess with some shock at first to my scientific sensibilities—–that both questions [the origin of mind and the origin of life from nonliving matter] might be brought into some degree of congruence. This is with the assumption that mind, rather than emerging as a late outgrowth in the evolution of life, has existed always as the matrix, the source and condition of physical reality—the stuff of which physical reality is composed is mind-stuff. It is mind that has composed a physical universe that breeds life and so eventually evolves creatures that know and create: science-, art-, and technology-making animals.”

                      ***********

                      In your concluding thoughts you directly address my seven objections and say, “If we cannot trust our reasoning because any or all of these objections are accurate,” then my challenges are self-defeating. This fails for two reasons: (1) I didn’t say if my challenges are correct, then ”we” can’t trust our “reasoning.” I said ”your” “argument” fails. It fails because you’re claiming some sort of grounding (a term you can’t define) on God’s perfect Reason (a thing you can’t establish), from a deceptive source (which has been established) to a faulty receiving apparatus (which has been established). My claims are not touched by my objections because I accept my epistemological limitations and don’t claim some external, supernatural source for anything. (2) Your objection requires that we interpret “cannot trust” as “cannot Trust” or “cannot trust even a little bit.” My argument is that we can (provisionally) trust our reasoning well enough to produce rational thoughts, as evidenced by our success navigating the map of reality we create. Unless you can demonstrate that, on atheism, my reasoning is guaranteed to always be faulty, such that everything I utter is automatically irrational, you can only test each claim of mine, individually, for rationality. You have not pointed out any irrationalities in the structure of any of my seven objections.

                      *************It has been established that God is deceptive? This is very bizarre. How have you established this? Via an assertion?

                      Secondly, in order for your objections to be accepted as accurate, we must have some reason to believe that the reasoning faculties which you used to create your objections are reliable. But how can we assume that you have reliable reasoning faculties if atheism is true? Again, if atheism is true, your objections to my stance are the product of mindless and non-rational natural forces. You must steal from Christian epistemology in order for your objections to stand. This means that your stance is self-defeating.

                      You have to produce SOME reason to accept human reasoning faculties as accurate before any of your objections can be accepted as accurate. But how can we even consider your objections as accurate if our thoughts are the product of natural forces which lie outside of our control? Logical consideration of alternative stances requires volitional control of reasoning faculties, but we cannot have such control if we assume that atheism is true. This is because atheism inextricably leads to the conclusion that our thoughts are the product of natural forces outside of our control.

                      You say, “My argument is that we can (provisionally) trust our reasoning well enough to produce rational thoughts, as evidenced by our success navigating the map of reality we create.” But in order to trust the conclusion that we have successfully navigated the map or reality we create, we must be able to trust the reasoning faculties which we used to determine that we have successfully navigated this map. Your argument is thoroughly tautological.*************

                      The non-rational can’t cause the rational

                      This brings us to what is probably your main argument, and the one upon which your “atheist criticism is irrational” argument is built. You claim that irrational things cannot cause rationality, specifically that non-rational neurons do not have the potential within themselves to cause rational thoughts.

                      In support of this you offered three quotes, which I rejected as either useless opinion or quotes that didn’t demonstrate your claim. In the case of your Ward quote, you didn’t object, but you challenged my dismissal of your Feser quote by way of returning to your “neurons firing cannot be true or false” argument. This won’t do you any good, because it doesn’t address my objection. Feser argues that if X lacks the potential to cause effect Y, then X cannot cause effect Y, which is pretty useless anyway because it’s a mere tautology. But I don’t object to his argument to this effect. My objection is that he (and you) has not demonstrated that X = collections of neurons, and Y = rational thought. You can’t just argue that things lacking feature X cannot do Y, you have to demonstrate that Feser’s argument applies to neurons and thoughts. You might be tempted to say, “Of course it applies! It’s obvious!” but it isn’t so obvious, as we’ll see shortly.

                      Next you launched a number of arguments against my objection that you hadn’t demonstrated your claim.

                      First you tried to shift the burden of proof (you try this twice in the same argument). “No, you need to demonstrate that non-rational processes have the potential for rational thought.” This is obviously false. If you make a claim, you need to support and defend that claim. If, in the course of objecting to your claim, I make claims of my own, that only means that now we both have assumed a burden of proof for different claims. So this defense is easily swatted aside by providing proof that you’ve actually made the claim I say you have:

                      From a post of yours on January 27: “The implication of this philosophical axiom is that irrational causes do not contain the effect of reason potentially in themselves. Impersonal causes do not contain the effect of personhood potentially in themselves. Unintelligent causes do not contain the effect of intelligence potentially in themselves.” This aptly demonstrates your burden.

                      Next you quote Moritz, who wants to create a difference between “basic” logical circuitry and the super-duper advanced circuitry needed for abstract arguments, etc. He concludes (without demonstrating it, naturally) that this couldn’t happen, and “given all the above,” says this would be the only way to “guarantee the right outcome.” Unfortunately, I don’t “give” all the above. Naturalism doesn’t claim to be able to “guarantee” any mental states. Evolution doesn’t need to “guarantee” any mental states. Also, if Moritz was correct in expecting a “guarantee,” then he is wrong to say there is no problem for theists, who would fail just as miserably if expected to “guarantee” any knowledge.

                      *********But, under atheism, brain circuitry cannot produce an abstract argument because brain circuitry is guided by natural forces outside of your control. The natural forces which govern your nerve impulses led you to be an atheist, and the natural forces which govern my nerve impulses led me to be a theist. But why would the natural forces which govern your nerve impulses be different than the natural forces which govern my nerve impulses? Are our nerve impulses not governed by the same natural forces?

                      Further, how could we consider the evidence and make a volitional choice between theism and atheism if our thoughts are governed by natural forces which lie outside of our control? Reasoning ability necessitates the free-will ability to weigh alternative explanations (using a process of reasoning) in order to make a volitional choice between these alternatives. But atheism provides us with no capacity for free-will, and therefore no capacity for a process of reasoning. Atheism and determinism (lack of free-will) are inextricable, as is further illustrated by the fact that top-rung atheists are almost exclusively determinist (Dawkins, Dennett, Harris, Crick, Sagan, Churchland, etc., etc.).

                      “Super duper advanced circuitry” is just as much under the control of non-rational natural forces as bodily actions such as digestion, if atheism is true. The complexity of circuitry is completely irrelevant. Again, reasoning faculties require the free-will ability to examine evidence and make volitional choices between competing views. But we can make no such choice if atheism is true because there is no way to have a free-will ability to use a process of reason to choose between various logical alternatives. Under atheism, our thoughts are just as much the product of mindless and non-rational natural forces as the wind in the trees. On atheism (and the determinism which necessarily stems from it), you HAD to be an atheist and I HAD to be a theist. But how can we possibly determine which of us is right if we do not even have the ability to control our thoughts and thus our reasoning faculties? The atheist stance is thoroughly incoherent.*************

                      You then refer to Feser, and my objection that you need to demonstrate that his water spigot argument applies to brain matter. You say “if truth is nothing but the firing of neurons in your brain, then your belief that truth is nothing but the firing of neurons in the brain…..is nothing but the firing of neurons in your brain.” You, Feser, and Colson & Pearcy agree that, on atheism, “thinking is reduced to brain processes,” Therefore, on atheism, truth is a type of proposition, which is a type of thought, which is a brain state, which is a pattern of neuronal impulses. To say it’s “nothing but” the firing of neurons in the brain is woefully inadequate, and betrays an appalling ignorance of biology. “Othermaterial bodily processes” cannot be true or false because they are not brain states corresponding to thoughts, which correspond to propositions, which may be true or false.

                      You continue with more from Feser, about formal vs eminent potential, but my earlier objection still stands: Demonstrate that “a neuron is a purely non-rational physical thing which does not contain the effect of reason potentially in itself, either formally or eminently.” Feel free to focus on “eminently,” since that’s what I would object to.

                      Finally, you object that, “if someone said, ‘You fail to demonstrate that normal water from a spigot could not produce a red puddle all by itself. Therefore, normal water from a spigot can produce a red puddle all by itself’… Such an argument is patently fallacious and is philosophically equivalent to your argument that neurons have the potential to produce rational thought because I have allegedly not demonstrated that neurons have the potential to produce rational thought.”

                      Here you misrepresent me. I didn’t say, “You failed to demonstrate your claim, therefore my claim stands.” I said “you failed to demonstrate your claim, thereforeyour argument fails.” Full stop–nothing about my claim. But I agree…. IF I had said that, it would have been fallacious.

                      **********Please see my above points about how brain matter is governed by natural forces outside of our control if atheism is true. Again, reasoning faculties require the free-will ability to examine evidence and make volitional choices between competing views. If thoughts are the product of brain matter and neuronal impulses which are governed by mindless natural forces beyond our control, then reasoning faculties cannot exist. Brain matter would be just as much under the control of mindless and non-rational natural forces as the wind in the trees, if atheism is true. Natural phenomena governed by mindless forces cannot be true or false. This point stands whether or not the natural phenomena is simple (such as wind) or more complex (such as neuronal impulses within brain matter).

                      Further, as the research with OCD patients cited (further down in this reply) by UCLA Professor of Research Psychiatry Jeffrey M. Schwartz demonstrates, brain states do not even occur until a volitional choice is made and volitional choices can cause structural changes in the physical brain. This dovetails very neatly with the conclusions of modern physics which demonstrate that consciousness produces matter.

                      George Gilder further elucidates upon this point:

                      “The usual materialist assumption is that the brain – the hardware – comes first and the mind somehow emerges from it. But the computer offers a contrary example. The computer design is itself a software design and determines the structure of the electronic circuitry that constitutes the computer…It is the human mind that brings meaning to the syntax of the machine, whether hardware, software or wetware. The higher-level languages of software lend significance to the dumb electrons circulating through the system.”***********

                      Next, you quote Colson and Pearcey again, themselves quoting C. S. Lewis. Lewis’s ignorance doesn’t serve as an argument. If Lewis “cannot understand” how the mind should have any more significance than wind in trees, that speaks to the profoundness of his ignorance, not the power of his argument. While I disagree about what conclusion we should draw from it, I agree wholeheartedly with your observation that the universe is amazingly ordered… down to the atomic and subatomic levels. Perhaps if Lewis had considered that minds ‘depend on’ brains, and brains depend on biochemistry, and biochemistry depends on the amazing order at the atomic level, he might not have been so flummoxed. Regardless, as I said… Lewis’s bewilderment is not an argument.

                      **********No, Lewis does not think that the mind has no more significance than the wind in the trees. Rather, he thinks that there can be no such thing as a true or false statement if our thoughts are the product of non-rational natural forces.

                      You admit that the universe is amazingly ordered, but atheism lacks a fundamental organizing principle. This must be counted as a fatal explanatory failure of atheism. What reason do we have to believe that mindless matter has organizing or creative properties? We see conscious and intelligent minds organizing and creating every day, but where do we see mindless matter organizing or creating? You can kick the ball down the road and cite natural laws as producing some order from disorder, but this just leaves us with the question of what governs natural laws. Under no circumstances can atheism ever answer this question. Ascribing such faculties to mindless matter would be the worst form of superstition.

                      WHAT PRODUCES ORDER FROM DISORDER? WHAT GOVERNS NATURAL LAWS SUCH AS THE LAWS OF PHYSICS? I have put this question in all caps to highlight its importance. You can substitute the word “regularity” for “law,” or whatever other term you prefer. Under NO CIRCUMSTANCES can atheism ever answer these crucial questions. In the theistic model, it is immediately obvious why matter follows natural laws: The same mind that creates matter (God’s mind) also directs it. As Robert Boyle, the founder of modern chemistry, put it:

                      “The nature of this or that body is but the law of God prescribed to it [and] to speak properly, a law [is] but a notional rule of acting according to the declared will of a superior.” [italics added]

                      Or, as James Joule, the propounder of the first law of thermodynamics, for whom the thermal unit of the “Joule” was named, put it:

                      “It is evident that an acquaintance with natural laws means no less than an acquaintance with the mind of God therein expressed.”

                      Or, as the knighted mathematician, physicist and astronomer Sir James Jeans put it in his book The Mysterious Universe:

                      “There is a wide measure of agreement which, on the physical side of science approaches almost unanimity, that the stream of knowledge is heading towards a non-mechanical reality; the universe begins to look more like a great thought than a great machine. Mind no longer appears as an accidental intruder into the realm of matter. We are beginning to suspect that we ought rather to hail mind as the creator and governor of the realm of matter.” (italics added)

                      What answer does the atheistic model provide to the question of how an inanimate thing can be compelled to follow a law? Only various versions of “matter follows laws because it just does.”*******************

                      You conclude their argument by asking, “Is the statement, ‘everything is material’ itself a material object’” Of course not, and no materialists say it is. It is a statement—an abstract–and like other abstracts, like numbers, emotions, etc., materialists don’t say it doesn’t exist, only that it’s a product of physical reality. Saying “everythingultimately stems from the material” is not at all the same as saying “everything ismaterial.” Colson and Pearcey’s simplistic misrepresentation of materialism is not an argument.

                      ***********But in order to conclude that “everything ultimately stems from the material,” you must ignore the insights of modern physics, which clearly demonstrate that an immaterial conscious observer is necessary to collapse the wave function in order for a material thing to take physical form. Before an observation is made by an immaterial conscious observer, material things do not exist in any form besides a possibility wave. The famed physicist Eugene Wigner, for example, pointed out that materialism (the matter-comes-first view) is completely incompatible with modern physics.

                      Further, you must ignore the insights of modern biology, which demonstrate that DNA (the language of life) uses abstract, symbolic representation to codify and store information for an organism to grow and develop. This is further evidence that the mental precedes and produces the physical.******************

                      You present a few more points meant to bolster your argument, some of which are variations of previous arguments. For example, you say that matter is not in the “category” of things that can have truth value. You enlist a quote from Barr to support this statement. The quote starts off with a faulty premise in that he says neurons are abstract concepts. There’s no need for me to refute that statement, as you’ve already done it for me many times. For example, in the very same post, you expand on a Feser quote and say, “a neuron is a purely non-rational physical thing.” [bold mine] I’ll let your refutation of Barr stand.

                      ***********No, you’ve got this confused. Barr says that the abstract concept of “neuron” must be a physical thing if materialism is true. “Neuron” is an abstract concept necessary to serve as a representation for a physical object. But, under materialism, there cannot exist abstract concepts because only material things exist.**************

                      You conclude your quote of Barr with the $64,000 question, “How can one pattern of neuronal impulses be more true or less true than another pattern of neuronal impulses?” I’ve already answered it, and in fact you’ve answered it yourself, with your Colson-Pearcey quote, but it won’t hurt to repeat it to be sure the point is clear. Some “neuronal impulses,” firing within and among other groups of neurons, produce what we call brain states. Some brain states produce what we call thoughts or beliefs, or propositions. Some thoughts we call true if they map closely enough to reality. The brain state we call a thought is not the same as the sensory impulses we call a toothache, or the autonomic impulses (and chemical reactions) we call digestion. (Additionally, the thought “I have a toothache” is not the same as the sensory impulses causing us to perceive pain in that tooth.)

                      **************But, once again, you must ignore the insights of modern physics which clearly demonstrate that mind comes first, and that matter is a construct of consciousness. As Max Planck, the Nobel Prize winning physicist who founded quantum physics put it:

                      “As a man who has devoted his whole life to the most clear headed science, to the study of matter, I can tell you as a result of my research about atoms this much: There is no matter as such. All matter originates and exists only by virtue of a force which brings the particle of an atom to vibration and holds this most minute solar system of the atom together. We must assume behind this force the existence of a conscious and intelligent mind. This mind is the matrix of all matter.”

                      And the problem with mind depending on biochemistry is clear, as I have stated before. How can chemicals (such as brain chemicals) be true or false, any more than the chair you are sitting in can be true or false? Material things do not belong to a category of things which can be said to have truth value. This is the same category error that you continue to commit.

                      Further, modern science has demonstrated that you have it exactly backwards. Minds do not depend upon brains. Rather, brains depend upon minds. UCLA Professor of Research Psychology Jeffrey M. Schwartz comments on how physical activity within the brain follows mental focus, and how mental states cause brain states, and even alter brain structure in his book The Mind and the Brain:

                      “Modern neuroscience is now demonstrating what [William] James suspected more than a century ago: that attention is a mental state (with physically describable brain state correlates) that allows us, moment by moment, to “choose and sculpt how our ever-changing minds will work, [to] choose who we will be the next moment in a very real sense…. Those choices are left embossed in physical form on our material selves.” If James was speaking metaphorically, he was also speaking with almost eerie prescience. The seemingly simple act of “paying attention” produces real and powerful physical changes in the brain. In fact, Stapp’s work suggests that there is no fully defined brain state until attention is focused. That physical activity within the brain follows the focus of attention offers the clearest explanation to date of how my hypothesized mental force can alter brain activity. The choice made by a patient—or, indeed, anyone—causes one physical brain state to be activated rather than another. A century after the birth of quantum mechanics, it may at last be time to take seriously its most unsettling idea: that the observer and the way he directs his attention are intrinsic and unavoidable parts of reality.”

                      “There, I propose that the time has come for science to confront the serious implications of the fact that directed, willed mental activity can clearly and systematically alter brain function; that the exertion of willful effort generates a physical force that has the power to change how the brain works and even its physical structure. The result is directed neuroplasticity. The cause is what I call directed mental force. Mainstream philosophical and scientific discourse may remain strongly biased toward a materialist perspective. Yet the simple fact is that the materialism of classical physics offers no intuitively meaningful way of explaining the critical role played by the will in the brain changes seen in OCD patients.”

                      Assisted by University of California, Berkeley physicist Henry Stapp, Schwartz notes how his research into OCD patients dovetails very neatly with what we have learned from modern physics….matter does not exist in the absence of an immaterial conscious observer.

                      “Correct it may be, but at its core quantum physics departs from classical physics in a very discomfiting way. Integral to quantum physics is the fundamental role played by the observer in choosing which of a plenitude of possible realities will leave the realm of the possible and become actual. For at its core, quantum physics challenges the ontology that permeated the scientific enterprise for centuries, the premise that a real world—independent of human choice and interference—is out there, uninfluenced by our observation of it. Quantum physics makes the seemingly preposterous claim (actually, more than claim, since it has been upheld in countless experiments) that there is no “is” until an observer makes an observation.”

                      NON CREDENTI, PLEASE PAY PARTICULAR ATTENTION TO THE LAST SENTENCE IN THE ABOVE CITATION FROM STAPP AND SCHWARTZ:

                      Schwartz and Stapp continue:

                      “But as soon as an observer performs a measurement—detecting an electron landing on a plate, say—the wave function seems to undergo an abrupt change: the location of the particle it describes is now almost definite. The particle is no longer the old amalgam of probabilities spread over a large region. Instead, if the observer sees the electron in this tiny region, then only that part of the wave function representing the small region where observation has found it survives. Every other probability for the electron’s position has vanished. Before the observation, the system had a range of possibilities; afterward, it has a single actuality. This is the infamous collapse of the wave function.”

                      Non-Credenti, your view that “minds depend on [material] brains” is in direct conflict with the insights of modern science. You have it exactly backwards. Atheism must deny modern science in order to preserve itself from disintegrating.***************

                      You also quote Jinn as he makes an argument against determinism and the ability of naturalists to have valid thoughts. Unfortunately, his argument depends on some unproven assumptions and faulty understanding. For example:

                      – He incorrectly states that free will is incompatible with atheism. He trusts his readers have never heard of compatibilism.
                      -He assumes free will.
                      -He claims that free will is incompatible with atheism
                      -He doesn’t disclose that Dennett is a famous compatibilist (this isn’t illogical as much as it is dishonest).

                      ***********Please explain how compatibilism can save you from the point that atheism necessitates that our thoughts are the product of natural forces beyond our control. You have thrown the term “compatibilism” out there, but you have not explained what it means or how it can negate the point the atheism implies that are thoughts are the product of natural forces beyond our control. With a cursory examination of compatibilism, it seems to be just a form of determinism. If my cursory analysis is wrong, please explain.

                      How can free will be compatible with atheism? Please explain. Recall that above quote from Bo Jinn:

                      “Free will is incompatible with atheism for the reason that nondeterministic action would imply that there is something primary over matter and energy. Any naturalistic worldview would necessitate that human beings are an inextricable part of the same natural laws which created them. If atheism is true, then everything what we are and, furthermore, everything that we believe has to be the end result of consciousness responding to material stimuli without a say in the matter. The same way fire is doused by water and smoke rises from a burning log, everything about us has to be determined by the same physical rules of cause and effect that govern our material selves. If that were not the case, then we would be led to the conclusion that there is something at work controlling our material bodies- something that is separate and distinct from the body itself. Ergo; free will would necessitates that there is something primary over the matter which constitutes us, and that is a doorway to a very big problem for atheism.”********

                      -Similarly, he draws the unfounded conclusion that if our reasoning is invalid (itself incorrect, see below) that all our beliefs are invalid. This is wrong because invalid reasoning can lead to true beliefs.

                      ************But how could you know that the stance that “invalid reasoning can lead to true beliefs” is true if you do not have control over your reasoning faculties? If your reasoning faculties are governed by mindless and non-rational natural forces outside of your control, then how could you weigh the evidence and make a volitional decision to accept the above conclusion? For something to be deemed to be the product of reason, there must be a process of reasoning behind it….not mindless and non-rational natural processes which produce determined beliefs.**********

                  • Scott Youngren says:

                    Bob,

                    As I said to Non-Credenti, there is no way to ground matter in reason. The difficulty with grounding matter in reason is very easy to see. Matter does not belong to a category of things which can be said to have truth value. A neuronal impulse (or a pattern of neuronal impulses) can be neither true nor false, just as the chair I am sitting in cannot be true or false. Therefore, grounding reason in matter (and material things like neurons) is a category error. University of Delaware physicist Stephen Barr elaborates in Modern Physics and Ancient Faith:

                    “Cognitive scientists talk about neurons, for example. But ‘neuron’ itself is an abstract concept that arose from the researches of biologists. For the materialist, then, even this concept of ‘neuron’ is nothing but a neurological creation; it also is a pattern of neurons firing in someone’s brain. If this sounds like a vicious circle, it is. We explain certain biological phenomena using the abstract concept ‘neuron,’ and then we proceed to explain the abstract concept ‘neuron’ as a biological phenomenon—indeed, a biological phenomenon produced by the activity of neurons. What we are observing here is the snake eating its own tail, or rather its own head. The very theory which says that theories are neurons firing is itself naught but neurons firing.”

                    “…Why should anyone believe the materialist, then? If ideas are just patterns of nerve impulses, then how can one say that any idea (including the idea of materialism itself) is superior to any other? One pattern of nerve impulses cannot be truer or less true than any other pattern, any more than a toothache can be truer or less true than another toothache.”

                    Again, grounding reason in matter is an open-and-shut category error because material things such as neurons and neuronal impulses (and chairs) do not belong to a category of things which can be said to have truth value. Please answer the question that Barr alludes to above: How can one pattern of neuronal impulses be more true or less true than another pattern of neuronal impulses, any more than one toothache can be more or less true than another toothache?

      • Non Credenti says:

        This is a continuation of my responses to your post of December 8, continuing after I explained atheist grounding.

        Next you say that my explanation of Mary’s brain physiology “is self-defeating” because my theories are “nothing but the product of an irrational physical process that evolved…” Firstly, I think the more appropriate term is “non rational” not irrational. Irrationality implies the opposite of rationality, while non rationality implies simply the lack of rationality. A rock is more correctly said to be non rational, not irrational, whereas we call irrational something that could or should be exhibiting rationality, but is not. Regardless, your conclusion seems to be that, on naturalism, beliefs must be irrational, since they arise from “an irrational physical process,” and you’re inferring that the irrational cannot give rise to the rational. This is the fallacy of composition, in which it is inferred that the whole must have some property because some of the part(s) have that property (or cannot have a property because the parts do not have that property). A simple illustration should suffice to show the error. “Rivets and sheet metal and engines cannot fly, therefore an airplane made of rivets and sheet metal and engines cannot fly” is obviously wrong; it tries to take the properties of the parts and ascribe them to the whole. The error works the other way, as well, and is called the fallacy of division, which infers that what is true of the whole must be true of the parts. “A watermelon’s sweet flesh is red, therefore, the fructose molecules that make up part of the watermelon’s flesh are also red.”

        ” If your worldview is true, then your ideas about how our [reasoning] evolved are natural phenomena in the brain. How can natural phenomena in the brain be either true or false?”

        They can’t, at least not in the way you’re referring to them. But the beliefs that arise from those natural phenomena can either comport with reality (truth) or not. (We end up with a strange scenario where we might have an accurate picture of reality, even though we can never KNOW it, but that only becomes a problem if we require absolute knowledge, rather than provisional knowledge.) Here, you’re ignoring emergent properties, which a complex system has, but which the individual sub-systems do not have. (Sodium does not taste salty. Chloride does not taste salty. Sodium chloride is salt, and tastes salty.)

        You repeat this error of overlooking emergence when you quote Goswami’s claim that the physical and chemical properties of molecules making up the brain don’t have a property called “survivability” and from there trying to conclude that the organism that results from these molecules cannot either. It’s another fallacy of composition to say that the whole (organism) cannot “attempt to survive” because individual molecules don’t. You also miss the fact that survivability is not a property unto itself. Different physical, mental, or behavioral traits can increase survivability in certain contexts. A polar bear’s thick fur coat and layer of fat is a survival trait in the arctic, it’s likely fatal in the tropics.

        Your quote of Goswami here is doubly strange, because in addition to his obvious blunder in missing emergence, you’re quoting a man who is a New Age huckster. He uses the same bad logic you quote to lead to the conclusion that prana, chi, and ki are real and can affect the world. Are you a New Ager, or have you become one because of reading Goswami’s works? (You have read his works, right? You’re not just blindly quoting him because some apologist quoted him in a book, right?) If you reject his New Age-y conclusions, how did you pick which of his arguments were compelling, and which weren’t? After all, they’re the same arguments. Fortunately you can safely discard his arguments, since he ignores emergence, and mangles ‘survivability’ to boot.

        • Scott Youngren says:

          —————–Next you say that my explanation of Mary’s brain physiology “is self-defeating” because my theories are “nothing but the product of an irrational physical process that evolved…” Firstly, I think the more appropriate term is “non rational” not irrational. Irrationality implies the opposite of rationality, while non rationality implies simply the lack of rationality. A rock is more correctly said to be non rational, not irrational, whereas we call irrational something that could or should be exhibiting rationality, but is not. Regardless, your conclusion seems to be that, on naturalism, beliefs must be irrational, since they arise from “an irrational physical process,” and you’re inferring that the irrational cannot give rise to the rational. This is the fallacy of composition, in which it is inferred that the whole must have some property because some of the part(s) have that property (or cannot have a property because the parts do not have that property). A simple illustration should suffice to show the error. “Rivets and sheet metal and engines cannot fly, therefore an airplane made of rivets and sheet metal and engines cannot fly” is obviously wrong; it tries to take the properties of the parts and ascribe them to the whole. The error works the other way, as well, and is called the fallacy of division, which infers that what is true of the whole must be true of the parts. “A watermelon’s sweet flesh is red, therefore, the fructose molecules that make up part of the watermelon’s flesh are also red.”———————

          Dictionary.com defines irrational as, “Without the faculty of reason; deprived of reason.” This definition only implies the lack of rationality. This is no fallacy of composition. Rather, this is a category mistake on your part. A quick Google search defines category mistake as, “The error of assigning to something a quality or action that can properly be assigned to things only of another category, for example, treating abstract concepts as though they had a physical location.”

          You airplane example takes more simple material things (rivets, sheet metal) and makes a more complex material thing (an airplane). But this analogy is completely invalid because consciousness is not merely a complex material thing. The existence and nature of matter are independent of the existence of consciousness. Your view that consciousness is really just complex matter is flatly wrong.

          Philosopher J.P. Moreland elaborates on this crucial point:

          …if J. P. Moreland is five feet and eight inches tall, but Eileen Spiek’s youngest son is six feet tall, then they are not the same thing. Further, if J. P. Moreland is five feet eight and Eileen Spiek’s youngest son is five feet eight, but it would be possible for J. P. to be five feet nine while Eileen’s youngest son were five feet ten, then they are not the same thing either.

          What does this have to do with the mind/body problem? Simply this: Physicalists are committed to the claim that alleged mental entities—substances, properties, events/states—are really identical to physical entities, such as brain states, properties of the brain, overt bodily behavior, and dispositions to behave (for example, pain is just the tendency to shout “Ouch!” when stuck by a pin, instead of pain being a certain mental feel of hurtfulness). If physicalism is true, then everything true of the brain (and its properties, states, and dispositions) is true of the mind (and its properties, states, and dispositions) and vice versa. If we can find one thing true, or even possibly true, of the mind and not of the brain, or vice versa, then dualism is established. Then the mind or its properties and states is not the brain or its properties and states.

          It may be that for every mental activity, a neurophysiologist can find a physical activity in the brain with which it is correlated. But just because A causes B (or vice versa), or just because A and B are constantly correlated with each other, that does not mean that A is identical to B. Sunlight may cause me to sneeze, but it’s clear that the sunlight is not the same thing as my sneezing. Something is trilateral (three sided) if and only if it is triangular (three angled). But trilaterality (the property of having three sides) is not identical to triangularity (the property of having three angles), even though they are constantly conjoined. Therefore, and this is critical, strict physicalism cannot be established by showing that mental states and brain states are interdependent on, causally related, or constantly conjoined with each other in an embodied person. Physicalism needs identity to make its case, and if something is true, or possibly true of a mental substance, property, or event that is not true or possibly true of a physical substance, property, or event, then strict physicalism is false.

          ….mental states are in no sense physical since they possess five features not owned by physical states:

          (1) there is a raw qualitative feel or a “what it is like” to having a mental state such as a pain (e.g., we can easily tell a pain from a feeling of joy, since the two experiences are qualitatively different);

          (2) many mental states have *intentionality—of-ness or about-ness directed towards an object (e.g., I can have a thought about a cat or of a lake);

          (3) mental states are inner, private, and immediate to the subject having them;

          (4) mental states require a subjective ontology—that is, mental states are necessarily owned by the first-person subjects who have them (only I can possess my thoughts; no one else can);

          (5) mental states fail to have crucial features (e.g., spatial extension, location, being composed of parts) that characterize physical states and, in general, cannot be described using physical language (my thoughts have no physical dimensions, no physical location, and aren’t made of simpler building blocks).

          Non Credenti, #5 in the above list requires particular emphasis because it brings to light the category mistake you commit when you confuse physical things with mental states (subjective, first person experience). A mental state is not composed of smaller parts.

          ——-How can natural phenomena in the brain be either true or false? They can’t, at least not in the way you’re referring to them. But the beliefs that arise from those natural phenomena can either comport with reality (truth) or not. (We end up with a strange scenario where we might have an accurate picture of reality, even though we can never KNOW it, but that only becomes a problem if we require absolute knowledge, rather than provisional knowledge.) Here, you’re ignoring emergent properties, which a complex system has, but which the individual sub-systems do not have. (Sodium does not taste salty. Chloride does not taste salty. Sodium chloride is salt, and tastes salty.)——————

          No, this is the same category mistake as above. Taste is a first-person, subjectively felt experience that does not have many of the above listed features, such as #5. The subjective experience of taste does not have spatial extension, location, and is not composed of parts. The experience of taste has no physical dimensions, no physical location, and isn’t made of simpler building blocks.

          However, sodium, chloride, and sodium chloride all have spatial extension, location, and are composed of parts. They all have physical dimensions, physical location, and are all made of simpler building blocks.

          Sodium, chloride, and sodium chloride are not owned by first-person subjects…etc, etc…

          Your reference to “emergent properties” is a perfect example of how atheists confuse an observation, on one hand, with an explanation, on the other hand. Yes, consciousness emerges in humans. Good job. Now please explain why this is so.

          Your suggestion that first-person, subjective mental experiences can be composed of material building blocks is an open-and-shut category mistake. The definition of category mistake that I got from Google, again:

          “The error of assigning to something a quality or action that can properly be assigned to things only of another category, for example, treating abstract concepts as though they had a physical location.”


          ————You repeat this error of overlooking emergence when you quote Goswami’s claim that the physical and chemical properties of molecules making up the brain don’t have a property called “survivability” and from there trying to conclude that the organism that results from these molecules cannot either. It’s another fallacy of composition to say that the whole (organism) cannot “attempt to survive” because individual molecules don’t. You also miss the fact that survivability is not a property unto itself. Different physical, mental, or behavioral traits can increase survivability in certain contexts. A polar bear’s thick fur coat and layer of fat is a survival trait in the arctic, it’s likely fatal in the tropics.——————

          Yes, consciousness, which involves subjectively felt first-person experiences, emerges in humans. Good job. Now please explain why this is so. “Emergence” is a perfect example of an observation that atheists confuse with an explanation.

          First-person, subjective experience is not composed of material parts. This is an open-and-shut category error.

          ————Your quote of Goswami here is doubly strange, because in addition to his obvious blunder in missing emergence, you’re quoting a man who is a New Age huckster. He uses the same bad logic you quote to lead to the conclusion that prana, chi,and ki are real and can affect the world. Are you a New Ager, or have you become one because of reading Goswami’s works? (You have read his works, right? You’re not just blindly quoting him because some apologist quoted him in a book, right?) If you reject his New Age-y conclusions, how did you pick which of his arguments were compelling, and which weren’t? After all, they’re thesame arguments. Fortunately you can safely discard his arguments, since he ignores emergence, and mangles ‘survivability’ to boot.—————

          Yes, I agree that Goswami is a new age huckster…at least when it comes to some of his beliefs. But to suggest that we can discard his solidly reasoned arguments because we don’t agree with other arguments that he makes is a non sequitur. And to dismiss everything that he says because we feel that he is a new age huckster would be to commit the Ad Hominem logical fallacy.

          I very deliberately cite a people with a variety of world views in my essays. I do this so that readers do not develop the perception that a Christian bias warps the arguments. In fact, I frequently cite atheists. For example, in my essay titled OK….I want numbers. What is the probability that the universe is the result of chance?, I cite the Cambridge University astrophysicist and mathematician (and atheist) Fred Hoyle:

          “A common sense interpretation of the facts suggests that a superintellect has monkeyed with physics, as well as with chemistry and biology, and that there are no blind forces worth speaking about in nature. The numbers one calculates from the facts seem to me so overwhelming as to put this conclusion almost beyond question.”

          Viewed from the lens of his atheist worldview, Hoyle has not choice but to conclude that this “super intellect” is space aliens.

      • Non Credenti says:

        My posts are appearing in a strange order, so if anybody is coming upon this response first, you can probably ignore it; it’s just cleaning up a few loose ends. My post of Dec. 11 on grounding is the only thing I think is worth the time reading.

        Quoting Bo Jinn, you say that “at the very last instance” truth / value judgments will “always require a judgment from a personal agent.” If by that you mean we just have to accept our base assumptions as givens, that they cannot be justified, then I agree. But how is that any different for the theist? If you hear a voice from the heavens, or feel some mysterious gut feeling telling you the earth is 6,000 years old, or to drown your children in a lake, you, too, must use your personal judgment to evaluate the message—Is it real? Is it really from God. Is this really what God wants me to do, or is it a sick test?

        “If human minds are the only personal agents in existence, it follows that all truth judgments based on reason are completely relative…, if atheism is true, then human reasoning has no validity at all, because valid reasoning implies a standard of truth that can be reasoned toward and a sufficient reason for believing that human reasoning works in the first place. That guarantee is non-existent in an atheist world.”

        Jinn thinks he’s making a grand point with his extreme “no validity at all” claims, but that’s what sinks his argument. If my “standard of truth” is “the facts (or truth) of reality,” then by reasoning, I’m simply trying to arrive at the most accurate map of reality I can. The success of that map in making predictions is “sufficient reason” to believe human reasoning works—at least to a degree. I can’t make any claims about how accurate the map is, but its continued success as a predictor of (the perceived) world surely seems sufficient to trust it provisionally. His next sentence does the expected equivocation, where “sufficient reason for believing” morphs into the requirement to be a “guarantee… in an atheist world.” Again, he sinks himself. If he requires a guarantee, then he must also provide a guarantee for his own account of reason, and he cannot do that. At best, he will use the same senses, memory, and reasoning (well….) as the atheist and proclaim that it’s not perfect but it seems to be successful enough at navigating the world that it would be irrational to abandon it.

        You continue by quoting Colson and Pearcey, who try to say we shouldn’t credit normal brain chemistry because extreme examples of abnormal brain chemistry aren’t normally credited. The absurdity of that claim shows either the magnificent ignorance of these apologists in areas of science and logic, or their astounding dishonesty in taking advantage of the unfortunate ignorance of their target audience. “Hey, you wouldn’t trust a computer that has a million volts running through it, so you shouldn’t trust one that has the designed 5 volts, either!”

        Next is an analogy from Pearcey that might actually be worse than the one she worked up with Colson. ”Materialism reduces thinking to biochemical processes in the brain, akin to the chemical reactions in digestion. But digestion is not something that can be true or false. It is just a biological fact. If thinking is reduced to brain processes, then our ideas are not true or false either.” “Hey, a moped is just a means of transportation, akin to a space shuttle, so let’s take my moped to the moon!” Thinking and digestion are vastly different processes. That they have some general thing in common doesn’t mean you can ignore the specific ways in which they’re different; namely that thinking happens among billions of neurons with trillions of interconnections and produces ideas while digestion happens in completely different cells and produces feces. This again ignores that beliefs are emergent.

        Then you circle back to BIV, and say if I didn’t mean it as an ontological question, fine. Why would I mean it that way? I and Bill Mitten(?) understand that it is showing a problem with our concept of knowledge, which is what your essay was also about. When you tell someone they’re confused, and it turns out the confusion is yours, the adult thing to do is admit your error, and maybe offer an apology, not keep doubling down, as you do when you bring up Plato’s Theaetetus, I guess in an effort to claim it also has something to do with ontology. Theaetetus is also about epistemology (just read it and it’s obvious). The Stanford Encyclopedia of Philosophy calls it “arguably Plato’s greatest work on epistemology.”

        Finally, you claim that I’ve “selectively ignored” your points in two other essays. In fact, I responded multiple times to your first essay, in the comments under that post. The tone of responses was becoming a bit too aggressive, so I stopped responding, but I’ll give a short recap of my general response to that essay: You don’t understand QM; you link to a YouTube video from a person who does not understand (or misrepresents) QM; you quote those with fringe beliefs who “strongly support” your interpretation, while ignoring the scientific consensus, which does not support your interpretation; when you do quote people who actually work in the field, you ignore those who would explain and support the consensus; and you use quotes from 100 years ago in a field that has undergone major changes in just the past five years.

        • Scott Youngren says:

          Non Credenti,

          I have bolded your comments and responded below:

          ——-Quoting Bo Jinn, you say that “at the very last instance” truth / value judgments will “always require a judgment from a personal agent.” If by that you mean we just have to accept our base assumptions as givens, that they cannot be justified, then I agree. But how is that any different for the theist? If you hear a voice from the heavens, or feel some mysterious gut feeling telling you the earth is 6,000 years old, or to drown your children in a lake, you, too, must use your personal judgment to evaluate the message—Is it real? Is it really from God. Is this really what God wants me to do, or is it a sick test?

          Jinn thinks he’s making a grand point with his extreme “no validity at all” claims, but that’s what sinks his argument. If my “standard of truth” is “the facts (or truth) of reality,” then by reasoning, I’m simply trying to arrive at the most accurate map of reality I can. The success of that map in making predictions is “sufficient reason” to believe human reasoning works—at least to a degree. I can’t make any claims about how accurate the map is, but its continued success as a predictor of (the perceived) world surely seems sufficient to trust it provisionally. His next sentence does the expected equivocation, where “sufficient reason for believing” morphs into the requirement to be a “guarantee… in an atheist world.” Again, he sinks himself. If he requires a guarantee, then he must also provide a guarantee for his own account of reason, and he cannot do that. At best, he will use the same senses, memory, and reasoning (well….) as the atheist and proclaim that it’s not perfect but it seems to be successful enough at navigating the world that it would be irrational to abandon it.———-

          Non Credenti, you tried to argue that irrational natural processes can lead us to truth. (Irrational is defined as “Without the faculty of reason; deprived of reason,” at Dictionary.com) But your atheism catches you in a trap, because if evolution led us to our beliefs, then this must include belief in God. But, according to atheism, belief in God is false. Therefore, on the atheist worldview, evolution causes people to have false beliefs. On atheism, we have no way to trust our beliefs because evolution has led many many people to a false belief in God (false, according to atheism, that is).

          And you commit a straw-man fallacy when you cite a 6000 year old earth because only a fringe minority of Christians believe this. Further, apparently evolution went wrong when it caused some people to believe in a 6000 year old earth. This is another example of how evolution leads to false beliefs, and therefore we cannot trust our reasoning faculties if atheism is true.

          This is the self-defeating incoherence in which atheism is caught.

          Atheism destroys our ability to trust our reason, but does not build it back up.

          How is it any different for the theist? Yes, both theists and atheists have to take their base assumptions as givens. But this does not relieve us from explaining WHY our rational faculties are reliable. You seem to think that the fact we have to take our base assumptions as givens leads to the conclusion that a worldview does not need to explain WHY we can trust our reasoning faculties to lead us to truth. But this is a complete non sequitur. In order for a worldview to be coherent, it must explain WHY we can trust our base assumption that our reason is reliable. But atheism does not provide any explanation for why we should be able to reliably take our base assumptions (such as our ability to reason accurately) as givens. Bo Jinn continues:

          ….for the sake of clarity, we shall spell out the problem of determinism, starting with why determinism and atheism are, by and large, inseparable. We have said that determinism is the denial of the human capacity for free will. Free will is incompatible with atheism for the reason that nondeterministic action would imply that there is something primary over matter and energy. Any naturalistic worldview would necessitate that human beings are an inextricable part of the same natural laws which created them. If atheism is true, then everything what we are and, furthermore, everything that we believe has to be the end result of consciousness responding to material stimuli without a say in the matter. The same way fire is doused by water and smoke rises from a burning log, everything about us has to be determined by the same physical rules of cause and effect that govern our material selves. If that were not the case, then we would be led to the conclusion that there is something at work controlling our material bodies- something that is separate and distinct from the body itself. Ergo; free will would necessitates that there is something primary over the matter which constitutes us, and that is a doorway to a very big problem for atheism.

          Simply stated; free will automatically implies the existence of an immaterial, enduring self, since that enduring substance is guiding the body, and is therefore primary over it. If the self transcends the material human corpus that would mean that conscious experience is itself fundamental, and not simply a waste product of determined materialistic processes. And if consciousness is fundamental, it would mean that it had to have come from somewhere, something or (perish the thought) someone that is beyond mere matter and energy. Obviously the transcendental implications of this line of reasoning are anything but atheistic. That is why the vast majority of more refined atheists, including Dennett, Harris and Dawkins, are all determinists.

          But what does this necessary implication of the atheist worldview say about our beliefs? Are not beliefs themselves the same result of this determined process? The possibility of having beliefs that are true or false necessitates that they were freely chosen. However, on atheism, that possibility does not exist. If all human action is determined, that would include the act of forming beliefs itself. If beliefs are determined, then they have no truth value. Atheism is one such belief, so atheism also loses its value and atheism becomes self-defeating once again.

          Premise 1: Naturalism (atheism) implies that there is no free will.

          Premise 2: Reasoning must be free in order to be valid.

          Premise 3: Therefore, if free will does not exist, then reasoning is invalid.

          Premise 4: If reasoning is invalid, then all our beliefs are invalid.

          Conclusion: If all beliefs are invalid, naturalism (atheism) is invalid. If one commits oneself to the belief in determinism, that all beliefs can be reduced to uncontrolled material processes, then the possibility of maintaining a valid belief is negated, since the belief is an inevitable consequent of an irrational process with no interest in producing truth. [“Irrational”, is defined by Dictionary.com as, “Without the faculty of reason; deprived of reason.”]

          That also goes for statements of science. On the other hand if one accepts the existence of free will then that opens up a doorway to something superseding the natural universe and atheism is thrown into equal doubt.

          Again, on atheism, we know that we cannot trust evolution to produce true beliefs, because evolution produced belief in God among many many people. Are you going to argue that theists freely chose their belief in God?

          —-You continue by quoting Colson and Pearcey, who try to say we shouldn’t credit normal brain chemistry because extreme examples of abnormal brain chemistry aren’t normally credited. The absurdity of that claim shows either the magnificent ignorance of these apologists in areas of science and logic, or their astounding dishonesty in taking advantage of the unfortunate ignorance of their target audience. “Hey, you wouldn’t trust a computer that has a million volts running through it, so you shouldn’t trust one that has the designed 5 volts, either!”——-

          That they have some general thing in common doesn’t mean you can ignore the specific ways in which they’re different; namely that thinking happens among billions of neurons with trillions of interconnections and produces ideas while digestion happens in completely different cells and produces feces. This again ignores that beliefs are emergent.
          —–

          So having a large enough number of neurons causes consciousness? Emergent? Yes, beliefs emerge in human minds. Good job. Now please explain why this is so. Stating that beliefs emerge (“are emergent”) in human minds is an observation that you confuse with an explanation.

          Unless you can enlighten us with your knowledge of science, your argument is entirely rhetorical. Why don’t you enlighten us and explain how brain chemistry and electricity can bring about consciousness…first person subjective experience.

          UCLA Research Professor of Psychiatry Jeffrey M. Schwartz explains why it is absurd to attribute consciousness to brain physiology in The Mind and the Brain:

          There is a difference between a programmed, deterministic mechanical response and the mental process we call consciousness. Consciousness is more than perceiving and knowing; it is knowing that you know.

          If it seems ridiculous even to consider why a handful of wires and transistors fails to generate subjective perceptions, then ask the same question about neurons outside the brain. Why is it that no neurons other than those in a brain are capable of giving the owner of that brain a qualitative, subjective sensation—an inner awareness? The activity of neurons in our fingertips that distinguish hot from cold, for example, is not associated in and of itself with conscious perception. But the activity of neurons in the brain, upstream of the fingertips’ sensory neurons, is. If the connection linking the fingers to the brain through the spinal cord is severed, all sensation in those fingers is lost. What is it about the brain that has granted to its own neurons the almost magical power to create a felt, subjective experience from bursts of electrochemical activity little different from that transpiring downstream, back in the fingertips? This represents one of the central mysteries of how matter (meat?) generates mind.

          In this book, I describe experimental data that support an alternative, offering evidence that the brain truly is the child of the mind.

          How? Through the mental act of focusing attention, mental effort becomes directed mental force. “[T]he effort to attend,” James believed, may well be a true and genuine “original force.” Modern neuroscience is now demonstrating what James suspected more than a century ago: that attention is a mental state (with physically describable brain state correlates) that allows us, moment by moment, to “choose and sculpt how our ever-changing minds will work, [to] choose who we will be the next moment in a very real sense…. Those choices are left embossed in physical form on our material selves.” If James was speaking metaphorically, he was also speaking with almost eerie prescience. For it is now clear that the attentional state of the brain produces physical change in its structure and future functioning. The seemingly simple act of “paying attention” produces real and powerful physical changes in the brain. In fact, [U of C, Berkeley physicist Henry] Stapp’s work suggests that there is no fully defined brain state until attention is focused. That physical activity within the brain follows the focus of attention offers the clearest explanation to date of how my hypothesized mental force can alter brain activity. The choice made by a patient—or, indeed, anyone—causes one physical brain state to be activated rather than another. A century after the birth of quantum mechanics, it may at last be time to take seriously its most unsettling idea: that the observer and the way he directs his attention are intrinsic and unavoidable parts of reality.

          As research with OCD patients (which Schwartz cites) demonstrates, willed mental effort can change how the brain works and even it’s physical structure. It is impossible to label consciousness the product of brain physiology when consciousness can ALTER brain physiology.

          This dovetails neatly with what modern physics tells us about how consciousness precedes matter. In the famous double slit experiment, particles do not take on material form until an observation is made by a conscious observer. Rather, particles only exist in a “possibility wave” until a conscious observer makes an observation.

          Schwartz (assisted by U of C, Berkeley physicist Henry Stapp) continues:

          “Correct it may be, but at its core quantum physics departs from classical physics in a very discomfiting way. Integral to quantum physics is the fundamental role played by the observer in choosing which of a plenitude of possible realities will leave the realm of the possible and become actual. For at its core, quantum physics challenges the ontology that permeated the scientific enterprise for centuries, the premise that a real world—independent of human choice and interference—is out there, uninfluenced by our observation of it. Quantum physics makes the seemingly preposterous claim (actually, more than claim, since it has been upheld in countless experiments) that there is no “is” until an observer makes an observation. Quantum phenomena seem to be called into existence by the very questions we ask nature, existing until then in an undefined fuzzy state. This feature of the quantum world led the American physicist John Archibald Wheeler to say that the world comes into being through our knowledge of it—or, as Wheeler put it, we get “its from bits” (bits of knowledge).”

          “Eugene Wigner, as I hinted earlier, followed the new realizations to their inevitable conclusion. “The laws of quantum mechanics cannot be formulated…without recourse to the concept of consciousness,” he wrote in 1961.

          “It seems inconsistent,” Wigner said in 1969, “to explain the state of mind of [an] observer…in terms of concepts, such as positions of atoms, which have to be explained, then, in terms of the content of consciousness.” If the positions of atoms (and thus, for our purposes, the state and arrangement of neurons, since neurons are only collections of zillions of atoms) have no unambiguous existence independent of the consciousness of an observer, Wigner asked, then how can that very consciousness depend on those same atoms? “The extreme materialistic point of view…is clearly absurd and…is also in conflict with the tenets of quantum mechanics,” he concluded.


          ——Then you circle back to BIV, and say if I didn’t mean it as an ontological question, fine. Why would I mean it that way? ——

          You didn’t mean it in an ontological way. I was merely pointing out that BIV also calls attention to the ontological question of whether or not an external physical world exists. You are trying to beat this dead horse in order to create a distraction.

          —–Finally, you claim that I’ve “selectively ignored” your points in two other essays. In fact, I responded multiple times to your first essay, in the comments under that post. The tone of responses was becoming a bit too aggressive, so I stopped responding, but I’ll give a short recap of my general response to that essay: You don’t understand QM; you link to a YouTube video from a person who does not understand (or misrepresents) QM; you quote those with fringe beliefs who “strongly support” your interpretation, while ignoring the scientific consensus, which does not support your interpretation; when you do quote people who actually work in the field, you ignore those who would explain and support the consensus; and you use quotes from 100 years ago in a field that has undergone major changes in just the past five years.———

          Non Credenti, this is smoke and mirrors in the purest sense. I said that quantum physics shows that an immaterial conscious observer is required to collapse a possibility wave into material actuality.

          Physicist Amit Goswami writes,

          “Now do you see why consciousness, to effect collapse, must be nonmaterial? A material consciousness arising in the brain is only a possibility wave. A possibility wave acting on a possibility wave just makes a bigger possibility wave. No actuality ever comes out of such an interaction (von Neumann 1955).”

          “You may not have noticed, but we can see paradox in the observer effect in another way. The observer chooses, out of the quantum possibilities presented by the object, the actual event of experience. But before the collapse of the possibilities, the observer himself (or herself) consists of possibilities and is not manifest. So we can posit the paradox as a circularity: An observer is needed for collapsing the quantum possibility wave of an object; but collapse is needed for manifesting the observer. More succinctly, no collapse without an observer; but no observer without a collapse. If we stay in the material level, the paradox is unsolvable. The consciousness solution works only because we posit that consciousness collapses the possibility waves of both the observer (that is, his or her brain) and the object simultaneously from the transcendent reality of the ground of being that consciousness represents.”

          Again, you think that truth emerges from consensus opinion. But this commits the logical fallacy of Appeal to Authority. The truth emerges from the most coherent logical argument. Further, you merely assume and assert that there is a consensus supporting your stance, but you do not provide any evidence to support your assertion. This is the logical fallacy of Argument By Assertion.

          You say that I do not understand QM. Well, then apparently the famous physicist Eugene Wigner (cited above), and the physicists I cite below also do not understand QM!!! Why don’t you explain to us, for example, where the physicists Eugene Wigner and Richard Conn Henry went wrong. HOW CAN THE WAVE FUNCTION COLLAPSE INTO ACTUALITY WITHOUT AN IMMATERIAL CONSCIOUS OBSERVER???

          I AM VERY EXTREMELY CURIOUS TO HEAR YOUR REPLY. I AM ALSO VERY EXTREMELY CURIOUS TO HEAR ABOUT THIS “NEW SCIENCE” WHICH HAS EMERGED IN THE LAST 5 YEARS!

          The primacy of consciousness is what the founder of quantum physics (Max Planck) was referring to when he said:

          “I regard consciousness as fundamental. I regard matter as derivative from consciousness. We cannot get behind consciousness. Everything that we talk about, everything that we regard as existing, postulates consciousness.”

          and it is what the great physicist Sir Arthur Eddington was referring to when he said…

          “The idea of a universal mind or Logos would be, I think, a fairly plausible inference from the present state of scientific theory.” [“Logos” is defined as “the word of God, or principle of divine reason and creative order.”]

          ….and it is what the knighted mathematician, physicist and astronomer Sir James Jeans was referring to (in his book The Mysterious Universe) when he wrote…

          “There is a wide measure of agreement which, on the physical side of science approaches almost unanimity, that the stream of knowledge is heading towards a non-mechanical reality; the universe begins to look more like a great thought than a great machine. Mind no longer appears as an accidental intruder into the realm of matter. We are beginning to suspect that we ought rather to hail mind as the creator and governor of the realm of matter.” (italics added)

          Physicist Richard Conn Henry explains why people (such as atheist biologists) cling to materialism/naturalism despite the fact that it has been completely discredited by modern physics:

          “Why do people cling with such ferocity to belief in a mind-independent reality? It is surely because if there is no such reality, then ultimately (as far as we can know) mind alone exists. And if mind is not a product of real matter, but rather is the creator of the illusion of material reality (which has, in fact, despite the materialists, been known to be the case since the discovery of quantum mechanics in 1925), then a theistic view of our existence becomes the only rational alternative to solipsism.” [“Solipsism” is defined as “the view or theory that the self is all that can be known to exist.”]

          Please note that Henry’s citation is not from 100 years ago, since he is still a practicing Professor of Physics.

          Further, you conveniently ignored a question from my previous comment: WHY DO MATERIAL THINGS SO CONSISTENTLY FOLLOW NATURAL LAWS SUCH AS THE LAWS OF PHYSICS AND THERMODYNAMICS?? (feel free to substitute the term “law” for whatever term you choose, such as “regularity,” etc..) WHY IS THERE NOT JUST CHAOS?? Chaos is what we should expect a priori, as Einstein put it.

          Not to be rude, but I have to use a counter in order to keep track of the times that an atheist avoids a question that he cannot coherently answer:

          THE COUNTER IS NOW ON ONE (1) FOR THE NUMBER OF TIMES YOU HAVE IGNORED THE ABOVE QUESTION.

  23. Thoranhaxmaul says:

    Can I try? *ahem*:

    All christians believe in Geocentricism. Geocentricism is the belief that Dark Age astronomers were absolutely correct about everything and that the Biblical model of the Universe is literally true. However, if there was a firmament above the clouds, and water above the firmament, and God is outside the Firmament, the God would drown!

    Therefore, geocentricism says God is dead. Therefore, you can’t bea Christian and a Christian and a Christian at the same time.

    Because this one little area of Christian mythology that not all Christians agree with and that I completely missed the point of, ESPECIALLY the part I clearly got way off, I have arbitrarily decided that the “God can drown” part must be the most important thing in Christianity.

    Since Christians by *actual* definition do not agree that god can drown and I’m obsessed with that part for some reason, i have decided that Christians contradict themselves.

    • Scott Youngren says:

      Thoranhaxmaul,

      Your argument is an absolutely textbook example of a straw-man logical fallacy. A copy and paste from this post about “straw man fallacy”:

      “A straw man is a common form of argument and is an informal fallacy based on giving the impression of refuting an opponent’s argument, while actually refuting an argument that was not advanced by that opponent.”

      Who do you think you are fooling by saying that, “All christians believe in Geocentricism”? This is a profoundly ridiculous statement, and therefore your argument is a particularly bad straw man argument. Where did you get this exceptionally ridiculous idea?!

      God can drown? God is a non-material entity who obviously doesn’t have lungs.

      I have seen a lot of really bad arguments for atheism, but this one is exceptional.

      Scott

      • Thoranhaxmaul says:

        Wow, you have no understanding of sarcasm or irony. OF COURSE it’s a strawman! I was making fun of YOUR straw-man you hypocrite!

        • Scott Youngren says:

          Thoranhaxmaul,

          Wow! That’s a lot of angry rhetoric and name calling. There is definitely some powerful emotion behind your atheism. Could it be that the existence of God is offensive to you because you don’t like being subject to a higher moral authority?

          A primary psychological motivator of atheism is the need to be free from having to answer to a higher moral power. If there is no God, there is also no right nor wrong, no good nor evil, and we can do whatever we want. As famed Russian novelist Fyodor Dostoevsky put it, “If there is no God, everything is permitted.” In a stroke of commendable honesty, the English novelist and philosopher Aldous Huxley admitted to the psychological motives behind his atheism (and nihilism):

          “I had motives for not wanting the world to have meaning; consequently assumed it had none, and was able without any difficulty to find satisfying reasons for this assumption. The philosopher who finds no meaning in the world is not concerned exclusively with a problem of pure metaphysics; he is also concerned to prove there is no valid reason why he personally should not do as he wants to do. For myself, as no doubt for most of my contemporaries, the philosophy of meaninglessness was essentially an instrument of liberation. The liberation we desired was simultaneously liberation from a certain political and economic system, and liberation from a certain system of morality. We objected to the morality because it interfered with our sexual freedom.”

          Prominent atheist philosopher Thomas Nagel is equally commendable for his honesty regarding the psychological motivations behind his atheism:

          “I want atheism to be true and am made uneasy by the fact that some of the most intelligent and well-informed people I know are religious believers. It isn’t just that I don’t believe in God and, naturally, hope that I’m right in my belief. It’s that I hope there is no God! I don’t want there to be a God; I don’t want the universe to be like that… My guess is that this cosmic authority problem is not a rare condition and that it is responsible for much of the scientism and reductionism of our time. One of the tendencies it supports is the ludicrous overuse of evolutionary biology to explain everything about life, including everything about the human mind.”

          Judging from the angry rhetoric and name calling which you use, Thoranhaxmaul, I am guessing that you have a similar “cosmic authority problem.”

    • V says:

      I have not read such a laughable depiction of Christianity or God in… oh I think ever.
      Omnipotent, omniscient, omnipresent God can no more be physically affected by his creation than Walt Disney could have been killed by the car driven by Goofy in Motor Mania.

  24. Thoranhaxmaul says:

    The entire basis for your argument is based on a misunderstanding of Evolution that doesn’t even make sense.

    You believe that evolution just stops the very second an animal gets to the point that it has the absolute bare minimum number of muscles/claws/whatever necesary for survival. Because… Economy? You think Evolution is controlled by someone who spends money on genetic mutations and so just stops once the animal is at the bare minimum to save money?

    You know nothing about Evolution OR competition in general. When you design a ra e car, you do not design it for petrol efficiency to get you from A to B. You make it as fast as possible because you’re competing with other car designers trying to do the same thing.

    And, since “Evolution just stops at this point for no reason” is the entire basis of everything else you said, it’s all bullshit.

    • Scott Youngren says:

      Thoranhaxmaul,

      The argument in this essay has nothing whatsoever to do with whether evolution stops or continues.

      Please respond to the main points of the essay. I will summarize below:

      1) Darwinian evolution selects for survival value, not for truth.

      2) Therefore, a false belief (such as, “I shouldn’t eat this plant because doing so will turn me into a werewolf”) provides just as much survival value as a true belief (such as, “I shouldn’t eat this plant because it is poisonous”).

      3) This state of affairs leaves us with no reason to believe that evolution has provided us with reasoning faculties which lead us to truth.

      4) Therefore, we cannot trust the reasoning faculties which lead to atheist/naturalist beliefs…such as the belief that our reasoning faculties evolved from mindless natural processes.

      5) This leaves us with the unavoidable conclusion that there is no reason to accept the naturalist account of evolution as true or reliable.

      Please note that Charles Darwin himself (in addition to many contemporary Darwinists) have noticed this huge problem. Darwin himself admitted this huge problem when he wrote:

      “With me the horrid doubt always arises whether the convictions of man’s mind, which has been developed from the mind of the lower animals, are of any value or at all trustworthy. Would anyone trust in the convictions of a monkey’s mind, if there are any convictions in such a mind?”

  25. Himanshi says:

    The blog elegantly dissects the inherent contradictions within atheistic naturalism. It sheds light on the paradox of relying on reason within a worldview that denies its validity, offering compelling arguments against the self-defeating nature of naturalism. The author skillfully navigates complex philosophical territory, inviting readers to critically examine their beliefs.

Leave a Reply

Your email address will not be published. Required fields are marked *